1st series [1] [2] [3] [4] [5] [6] [7] [8] [9] [10] [11] [12] [13] [14] [15] [16] [17] [18] [19] [20] [21] [22] [23] [24]  2nd series [1] [2] [3] [4] [5] [6] [7] [8] [9] [10] [11] [12] [13] [14] [15] [16] [17] [18] [19] [20] [21] [22] [23] [24] [25] [26] [27] [28] [29] [30] [31] [32] [33] [34] [35] [36] [37] [38] [39] [40] [41] [42] [43] [44] [45] [46] [47] [48] [49]

  View the latest questions and answers at askaphilosopher.org

Ask a Philosopher: Questions and Answers 12 (1st series)

Here are some of the questions that you asked a philosopher from June 2001 — August 2001:

  1. Objective idealism
  2. Vedanta and the meaning of life
  3. What is destiny and does it exist?
  4. A problem with God's omnipotence
  5. Human rights in a post-structuralist universe
  6. Non-causal events
  7. Validity of competing moral standards
  8. What use is the soul?
  9. Puzzle about war and the male birth rate
  10. Problem with the idealism of Berkeley
  11. Philosophy and the reason for our existence
  12. Our mission in the world
  13. I'm looking for a philosophical question that hasn't been explored
  14. Reincarnation in Hinduism and Buddhism
  15. Who you are is because of what you are
  16. What is health? what is disease?
  17. Did God create man, or did man create God?
  18. Our fear of death and dying
  19. Why is the day, day and the night, night?
  20. Love of wisdom and love of existence
  21. Auteur film theory
  22. Love and destiny
  23. Why we 'love' slavery
  24. "Art upsets, science reassures"
  25. The philosophy of Boethius
  26. Does everyone have a price?
  27. Definition of an experiment
  28. Problem with the theory of evolution
  29. Physical laws, logical necessity and counterfactuals
  30. The perfect lifestyle
  31. Existentialism made easy
  32. Amoralism, subjectivism and relativism
  33. The universe, infinity and near death experiences
  34. Problem of evil revisited
  35. Paradox of the surprise examination
  36. Explaining déjà vu
  37. Is morality dependent on religion? what is truth?
  38. Yin and Yang
  39. How we can reach happiness
  40. What is man?
  41. Which came first: language or the ability to categorize?
  42. Does Karma exist?
  43. So what if I don't believe in God?
  44. How we can believe in things that are not proved
  45. Philosophy is just a vast illusion
  46. Why a human being is not purely physical
  47. Purpose of human beings in an infinite universe
  48. Why do we get pangs of conscience?
  49. Injustice of the trial and execution of Socrates
  50. Health versus happiness
  51. Comparing moral goods
  52. Philosophers on the freedom of religion
  53. Darwin and Huxley's theories of evolution
  54. Differing concepts and conceptions of God
  55. Nature of thought and consciousness
  56. Meaning of "ptsosis kneuton"
  57. Is technology a double edged sword?
  58. Is a "multicultural society" self-contradictory?
  59. Idea of rebirth in Buddhism
  60. Why Krishnamurti thinks that charity is "selfishness"
  61. Affirmative action and professional sport
  62. Socrates on the definition of justice
  63. How 'disorder' can be 'order'
  64. Hume on the notion of causation
  65. Can religion be objectively proved?
  66. How Epicurus would feel about pornography
  67. Philosophy and high speed machining
  68. I've been asked to explain philosophy and religion and I don't know how
  69. Great story about the meaning of life
  70. Healing with magnetic energy
  71. Do knowledge and culture advance?
  72. Kant's view of free speech
  73. What jobs does a training in philosophy help you to do?
  74. Never jump to conclusions
  75. One certainty Descartes didn't consider
  76. Sartre and original choice
  77. Aristotle and the death penalty, abortion, minorities and assisted suicide
  78. Was Spinoza an atheist?
  79. Existentialist response to determinism
  80. Richard Rorty's views on truth
  81. Samuel Alexander and the evolution of the universe
  82. Warning about scientology
  83. Worries about the scientific account of perception
  84. Tarski's theory of truth without the logic
  85. Order, balance and harmony
  86. Berkeley on abstract ideas
  87. Excellent book to read on biographies of philosophers
  88. Why philosophers get better with age
  89. The power of so-called experts
  90. Christians and the doctrine of predestination
  91. How idealism and anti-realism explain our ability to 'push an object'
  92. What we have learned from the development of computers
  93. Gaston Bachelard's Poetics of Space
  94. Is there a coherent determinist theory of punishment?
  95. Is 'music' innate or a product of culture?
  96. Philosophical considerations on prayer

Rapoz asked:

What is objective idealism? (I understand that it is related to the world view of Plato.) What arguments are there in favour of this position and what arguments are against it?

Short overview:

Idealism, in terms of metaphysics, is the philosophical view that the mind or spirit constitutes the fundamental reality. It has taken several distinct but related forms. Among them are Objective and Subjective idealism. Objective idealism accepts common sense Realism (the view that material objects exist) but rejects Naturalism (according to which the mind and spiritual values have emerged from material things), whereas subjective idealism denies that material objects exist independently of human perception and thus stands opposed to both realism and naturalism.

Detailed elaboration:

As we experience ourselves as subjects (mind, consciousness) and objects (body, matter), it is no wonder that questions like 'How are mind and matter related?', 'Which is primary, consciousness or matter?', 'Which, mind or matter, is the source of the other?' are fundamental to all philosophy.

There are many philosophical positions trying to answer these questions. The spectrum of answers reaches from the extreme spiritual to the extreme material position and is usually divided into materialism and idealism.

Materialism holds to the primacy of matter, idealism to the primacy of consciousness.

Positions trying to avoid duality of matter and consciousness, are called monistic. They escape having to explain how mind and matter interact and therefore reduce one to another.

A justification of the idealist position is that that we only know for certain that our experience exists, while we never can be sure that matter exists. Hence, to explain mind by matter would be to explain the certain by the uncertain, which is a flawed form of explanation. This sounds quite clear, but how should we think of physical objects?

According to idealism a physical object is a cluster of properties such as color, size, weight, and texture, but there is no reason to think that those properties are caused by some non-mental stuff called matter. To treat an abstract concept as if it were something having physical reality is to reify that concept. Just because a noun "oddness" can be constructed from the adjective odd, doesn't mean that oddness really exists, there are only odd numbers. Likewise there is no good reason to think that matter exists. There are only objects with physical properties.

Plato can be said to be the earliest representative of metaphysical objective idealism. Dissenting from the view of Heraclitus that everything is in a state of flux and flow, he formulated, in the interest of ethics, his doctrine of eternal unchanging ideas. These ideas exist objectively in a supersensuous world and form the background and basis of the ever-changing phenomenal world. Reality is not inherent in the individual object, as, for instance, a horse or a tree, but in the general idea of horse or tree. The highest idea is the idea of the Good — a self-realizing end. According to Plato, transitory and imperfect matter does exist by participating in eternal and perfect ideas or "forms". Matter can be perceived by our senses, while the forms are recognized by our souls.

So Plato's worldview was dualistic, therefore not truly idealistic.

Modern idealism tries to escape this dualistic worldview. I will pick out two kinds of modern metaphysical idealism: subjective idealism and objective idealism.

Subjective idealism denies the existence of objective reality altogether, except perhaps as illusory, as for instance in the views of Berkeley. Objective idealism, such as the system of Schelling, recognizes the existence of objective worlds while regarding the ideal world as the primary production and paramount: the external world has a relative and temporary reality.

An example for subjective idealism is Berkeley's theistic idealism. Berkeley said that it is God who causes us to experience physical objects by His directly willing us to experience matter avoiding the extra, unnecessary step of creating matter.

But isn't belief in God not even more problematic than belief in matter? Not at all, Berkeley would reply: We do not know what matter is like, but we do know what minds are like. Therefore we know what God is like, as it is a supreme mind. We may not understand God fully, as he is infinite and we are only finite, but God is still a mind.

How about common experience? According to Berkeley it is again God, who makes different people experience the same thing at the same time. So-called common perceptions is like copies of the same movie running in different theatres at the same time. God causes each of us to have such similar experiences that we can communicate about them, just as if we were really in the same situation.

In contrast to Berkeley's subjective idealism was objective idealism. Objective idealism is the view that the world out there is Mind communicating with our human minds. It is formulated by the three German successors of Kant. These were F.W.J. Schelling (aesthetic idealism), J.G. Fichte (moral idealism), and G.W.F. Hegel (dialectical idealism). Differences between subjective and objective idealism were not always clear-cut, however. For instance, Fichte's idealism was later called subjective in contrast to Schelling's objective variety, while Hegel's became known as absolute idealism. The term Objective Idealism was only sometimes used by Schelling, while the term Subjective Idealism was used by both, Schelling and Hegel, to put their own ideas in contrast to Fichte's position.

While Schelling's Objective idealism remained insignificant, the objective idealist with most influence is probably G. W. F. Hegel. Hegel agreed with Berkeley, that there is no such thing as matter in the materialist's sense, and that spirit is the essence and whole of reality. But he objected to the idea that God is separated from the world. Therefore reality is not God and the minds that God creates, but a single, absolute, all-inclusive mind, which Hegel referred to as "The Absolute Spirit" or simply "The Absolute". The Absolute Spirit is all of reality, no time, space, relation or event ever exists or occurs outside of the Absolute. As the Absolute also contains all possibilities in itself, it is not static, but constantly changing and progressing.

How the do we relate to the Absolute?

Finite individuals like human beings, planets and even galaxies are not separate beings, but part of something larger. Our relation to the Absolute is similar to the relation of cells or organs to the whole body. Like the cells, that constitute an organism continually emerge, make their contribution, die and are replaced, we as human beings come and go, while the Absolute continues.

Hegel had no problem in considering an objective world beyond any particular subjective mind. But this objective world itself had to be understood as conceptually informed, as it were — it was objectified spirit.

A general objection to idealism is that it is implausible to think that there can be an analytic reduction of the physical to the mental. Hegel's system of objective idealism is under suspicion for substituting the Absolute for God, which doesn't make anything clearer in the end. And if we are forms that the Absolute is taking, it means that the Absolute gets headaches, all kinds of diseases and even thinks of committing suicide sometimes. Why should the Absolute inflict such things upon itself? If it does, or can't help doing so, is it worthy of being called "The Absolute"?

Simone Klein

back

Alex asked:

I have two questions for you:

1. What is the real meaning of life and how can we reach happiness? I searched for answers in Christianity, Buddhism and philosophy but I didn't find any answers at all.

But at the first question it's clear that like ALL things, the human civilization will end sometime. So then why is human kind in such a hurry for progress and stuff like this? Especially when we know that things like science and technology will destroy us because without nature we can't survive and we will destroy nature with all these things.

2. And at the second question it's clear that, like Buddha said, all life is suffering. When you have what to eat and drink, you think for a place to call home, when you have a home, you think for education, friends etc, when you have these, you think for love, children, wealth and stuff like this and when you have all this things the thoughts of death, God, meaning of life are troubling and worrying your soul. So how is it possible to reach happiness? Please answer me and I will be very grateful to you.

I will present a certain viewpoint that is derived from a faith tradition called Vedanta.

Man is essentially not what he/ she assumes him/ herself to be. In essence, he/ she is spirit. This spirit is in essence the very opposite of what man assumes himself/ herself to be.

Just as a speck in the eye is not natural to it and therefore one is compelled to discard it, similarly until man finds his true nature he will continue to be dissatisfied.

The Vedas consider the ability to discern this without being distracted by the desire to enjoy and find out that the enjoyment did not have the capacity to fulfill that desire, as the essential quality of the student who can gain the knowledge it seeks to impart. Vedas intend to help the student intuit and discover his/ her true nature. This endeavor is termed the utmost importance and is considered the highest calling and termed the highest end.

There are many different traditions that derive from the Vedas. They are broadly referred to as Hindu. The Hindus themselves refer to it as 'Sanatana Dharma' or the eternal quest since it is inherent in the very nature of being.

Gopi Mavankal

back

Mairon asked:

After we think, in a philosophy paper we only write one interesting question:

Does DESTINY exist?

We've been thinking, what is the thing that rules our lives and I didn't find a certain answer to that question, but that doesn't mean there are not possible answers — it's just that I'm not sure about them. One of those was that a God rules people but who can prove that he exists? Nobody, nobody in the present time or 2000 years ago.

And Fernando asked:

I'm studying High School at ITESM and taking a course on philosophy and some questions came to my mind:

Is everything in this life meant to be?

Does destiny control our lives or is there something that people can do to change what's happening to them and, if they could, is it because they want or is it predestined? In the case that everything was "written" before we came to Earth, does life then make sense?

And Homero asked:

Are we owners of our destiny? Does destiny exist?

At the age of twenty-one I discovered philosophy. From that time, I have known, more or less continuously and with only occasional lapses of doubt, that it was my destiny to be a philosopher. — How can a philosopher believe in such an ambiguous, questionable and misleading concept?

I am not interested in the standard, question-begging explanations of where the alleged false belief in destiny comes from. Of how and why we fall under the illusion that there are things we are destined to do, or the illusion that we possess a destiny. "There must be a reason why the world is the way it is, it can't just be an accident." Or, "There must be a reason why I am here, in the world, it can't just be an accident." So the explanation proceeds, we are led to invent a reason that exists 'out there' — perhaps a reason that God knows — all the while totally unaware that 'the reason out there' is merely a creature of our own imagination.

A more contemporary, but no less question-begging explanation is the idea that we are story-telling creatures, that we feel impelled to construct a coherent narrative that makes sense of the events and the decisions in our lives. As in the previous explanation, the sense of destiny is supposedly revealed as nothing but an illusion, an invention, a prop. The fact that you or I might find it difficult or impossible to live without that prop does not make it any less an illusion.

Both styles of explanation may be described as reductive: There is no such thing, in reality, as destiny. The belief in destiny has a cause. But the description of that cause does not involve the concept of destiny. In the same way, the belief, in the Middle Ages, that there were such things as witches who possessed supernatural powers derived from the Devil had a cause. Understanding that cause does not require that we believe in the actual existence of witches. The belief in destiny is false, just as the belief in witches is false.

The common assumption behind reductive explanations of the notion of destiny, I would argue, is a concept of belief which is altogether too rigid and simplistic.

I have heard it said that the logical difference between the concept of belief and the concept of desire is that in the case of belief, our intention and aim is to mirror the world. If the world is different from the way our belief represents it as being, then the belief is wrong, not the world. By contrast, with desire, our intention and aim is to change the world, to make it conform to our representation. If the world does not conform to our representation, then it is the world that is wrong, not our desire.

The belief in destiny — or, better, the sense of destiny — is not a 'belief' in this sense. Nor, on the other hand, is it a mere intention that I form, "I will act as if there were a destiny for me". Adopting the language of existentialism, to live with a sense of destiny is a way of being in the world. Just as to live without a sense of destiny is a way of being in the world. It is a choice that is 'mine' yet which I do not make, nor is it made for me.

My destiny is a real, objective feature of my world — of the world as it spreads out from this unique point in time and space — yet I did not find my destiny there, for nothing that can be found in the world could ever justify the belief that it possesses that feature.

Geoffrey Klempner

back

Matthew asked:

The God Thing

I have a problem with the idea of God as an omnipotent being. Does it make any sense? For surely the whole essence of a being (or at least a major part of it) is to have a point of view. And what is a point of view? It is a very particular way of looking at the world consisting of desires, hopes, likes, dislikes, fears etc. And if this is so then surely any sort of being cannot be omnipotent, for the very idea of point of view which is so essential to the idea of being is one which necessitates privation of experience?

Does this make any sense?

The Lord knoweth the thoughts of the wise, that they are vain. (I Corinthians: 3:20)

Yes, Matthew it makes excellent sense. To be properly described as "omnipotent" a God would have to be capable of doing absolutely anything at all. This has traditionally led philosophers to have a bit of fun by trying to think of "fallacies of omnipotence" — the things God simply couldn't, logically, do. These include, "Could God make another God more powerful than Herself?", "Could God make a stone too heavy for God to lift?", "Could God annihilate Himself?", and so on. The point is that, whether or not God has done these things, it is thought to be enough to have conceived of some impossible act for it to be shown that there are some things which God cannot do, therefore either an omnipotent God is a fiction, or the real God isn't omnipotent. Unfortunately, entertaining though these fallacies are, they only show that God cannot be illogical. And the idea of God being bound by the rules of logic is not something with which many believers seem to have difficulty.

However, the way you put the problem, centering it on the idea of 'point of view' is much more interesting. I think, though, the difficulties of personal experience are more applicable to the problem of God's omniscience than of his omnipotence.

Omniscience means "all knowing". In the traditional view of most monotheistic religions, God knows all things. Most especially, and central to much religious understanding, is the idea that God knows what I know. In other words, that God knows what each person is thinking, feeling, intending, understanding. As it says in The Book of Chronicles (28:9) "the Lord searcheth all hearts, and understandeth all the imaginations of the mind", and in the Koran (Sura 6) "Allah knows your secret thoughts." This is a vital centre of belief, for without accurate knowledge of His people's thoughts and intentions, then He cannot stand in judgement over them without risk of error. But, as you have begun to see, this cannot be the case.

I know what I know, and I know that all my perceptions and beliefs are clouded by the limitations of my knowledge. Say, for instance, that I can't understand Arabic. If that is so, then if you show me a word in that language, then I simply cannot see, no matter how hard I look, anything other than a shape on paper, albeit a rather decorative one. On the other hand, someone who does know Arabic will find it equally impossible to see an Arabic word without putting a particular meaning on to it.

So, if God were able to somehow able to access the contents of my mind, what He saw there could never be the world as I see it — it would always be clouded by God's own particular knowledge. If nothing else, His appreciation of my mind would be different to my own understanding in that He would know my mind at the same time as being aware of the contents of other minds. Now I have no such knowledge of other minds, so it cannot reasonably be said that God would know my thoughts as I know them. And what could 'knowing my mind' mean other than being able to think as I think? The best that God might be able to do is to read minds, to understand selves, in much the same way as you are reading a part of my mind, my self, by reading these words. No matter what care I take in writing, it is inevitable that our understanding of individual concepts will differ, because our reading of them is made up of our individual likes and desires and hopes and fears. And that is not omniscience.

I'm afraid the concept of God is far from logical, so that, any argument about the nature of His abilities will tend to lack logic as well. What we know of God we know by faith, emotion and tradition. As to whether some privation of experience is necessary to Being, by which I presume rational being, I am afraid I cannot know, for I am not omniscient.

You might care to have a look at:

http://www.str.org/free/reflections/philosophy/cangodma.htm
http://www.errantskeptics.org/Omnipotence.htm

Glyn Hughes


If I understand what you mean when you write that the whole essence of a Being is to have a point of view, I am afraid I can't agree with you. After all, lots of things "have being" or exist, but do not have points of view since they don't have minds, and there cannot be a point of view without a mind. Desks or stars "have being" but don't have minds, and so, don't have points of view. In any case, it seems to me that omnipotence concerns the power of God, and has nothing to do with his point of view.

Kenneth Stern

back

Jon asked:

Can human rights be substantiated within a post-structuralist universe?

Postmodernism developed within the tradition of continental philosophy and with the consequent disintegration of the self and the identification of distinct language games or narratives as governing our ethical and political lives, the continental tradition struggles to solve the problems it has raised for itself. You might be interested in Charles Altieri's book Canons and Consequences in which he outlines the problems the postmodernist J-F Lyotard faces in defining justice. If justice cannot be defined, neither can rights.

However, Altieri believes that the non-postmodern analytical philosopher John Rawls' account of justice as the result of a hypothetical initial position which determines basic rights is compatible with postmodernism since it abstracts from the concept of the self to involve "self and others", and it is non-historical and non-political. You have probably read Rawls' A Theory of Justice and know of the original position, in which the assessment of distributive rules and the principle of basic rights is fixed by abstraction from oneself and replaced with the consideration of oneself from a subjective and objective point of view, as a social individual. Reflection on the principles of rights and distribution is free from political and historical assumptions and the definition provides criteria for upholding rights in actual life.

The postmodernist has denied the subject/ object relation, the possibility of truth and determinacy of language and has difficulty establishing rights if he cannot define them, although Habermas holds that we even though we cannot abstract from language, we can abstract from norms and culture. By means of dialogue, rights can be established by consensus, but this collapses into relativism unless it is understood as ideal consensus. Then the problem arises that a society with an ideal consensus is a society with no conflict or areas for argument and dialogue comes to an end. This is more problematic than Rawls' idea of determining very basic principles.

Rachel Browne

back

Tim asked:

Is it possible for an event to be non-causal? If so, what are the implications of non-causal events?

According to quantum mechanics, there are uncaused events (such as the radioactive decay of an atom, for example, or the precise way in which the QM probability wave function collapses). But some people didn't like this idea ("God doesn't play dice with the Universe" Albert Einstein said), so they posited what were called 'hidden variables' which would be unable to be measured but which would cause quantum events. However, John Bell thought up an experiment that would distinguish whether an event depended on hidden variables or not, and Alain Aspect found a way to do it. The results showed that (on the assumptions made), there are no hidden variables.

Since all experiments rest on assumptions, we can look at them to see if there is a way out of the conclusion. One of the assumptions here is that causes precede effects. If that is false, then we can have backwards causation in time and the Aspect experiment does not show that there are uncaused events. Some people take this seriously, but others think backwards causation is logically suspect. In a way, you have to choose which you think is more impossible: backwards causation or uncaused events.

I would think that the scientific consensus at the moment is that some events are uncaused.

As to the implications, there are many. One I think is important is that it becomes not just practically but also theoretically impossible to predict the future in the way that La Place's demon could. The demon would know the exact state of the world at some instant, and then, using the strict causality of physical laws, be able to calculate the state of the world at some future time. If some events are uncaused, this is no longer possible.

Some people think that uncaused events can explain free will, because they allow us out of the sort of lock-step determinism that La Place argued for. I disagree. I want free will to mean that I choose what to do for my own reasons, not that science can't predict what I will choose because some uncaused events take place in my brain. If these events are uncaused, then they are not caused by me, and it isn't my decision any more.

Tim Sprod


Do you mean by "non-causal", "uncaused" or do you mean "not causing anything"? The problem, as I see it, is that no one really knows what to make of causation. Hume dealt with it in a very stunning and frustrating way (take a look at his Treatise if you haven't), and most philosophers, as far as I can tell, spend a lot of time listing types of causes and effects and taking cause more or less on intuitive grounds. I'll take your meaning to be "uncaused".

Here's an example: suppose that there were a machine that could "read" the positions of all the atoms, say, in a piece of matter (which are moving around or vibrating or whatever), and project holographs of them, magnified a few million times, into the room. So the hologram is caused by the lasers, which is caused by the reading mechanism, etc. Now clearly we don't want to say that the images of the atoms or quarks or whatever in that picture are causing each other's motion, right? Why not? Because our physical theories say they aren't; those are our physical intuitions. Let's take it one step further; suppose (and I'm extrapolating from an example of Tye's here) that this "piece of matter" is someone's living brain. Is the hologram conscious? Why not? Because it's "just pictures"? But they're interacting just like the elements in the brain are, so why can't they be conscious? Believe it or not, I've asked many people this question, and no one can really answer it; we just have an intuition that the hologram can't be conscious, because of "causality".

So to get back to your question, now that I've muddled the waters of causality somewhat (and I've only talked about linear causality here...). in quantum physics, the answer to your question is pretty unequivocally "yes". "Vacuum fluctuations" and "virtual particles" are the results of the uncertainty principle, roughly speaking. That is, given that there is a lowest level of energy below which nature cannot go, one filled with a kind of undifferentiated wave-function (which is a real entity, by the way) which is (effectively, but actually it's in many states simultaneously and changing the likelihood of sets of these) fluctuating very rapidly, the spontaneous production of particles and energy takes place all the time. The vacuum is not a vacuum, in other words, but there isn't anything quite there either, except occasionally. Are these events "caused"? Well, there's no simple and direct cause we know of. There's also a phenomenon known as "tunneling", where a particle is quite suddenly and spontaneously somewhere where it basically can't be, because it has a very low probability of being there. How does that happen? It just happens, with some finite probability. Is it caused? Well, it can be pushed in that direction, but no one can say that it will happen with certainty.

So the implications of these events are quite profound, actually. Whole electronic industries are built on the tunneling effect, and huge (and well-verified, for the most part) theories make use of virtual particles and vacuum fluctuations to explain, for example, the interactions of the quarks comprising other particles.

Steven Ravett Brown


If you mean by a "non-causal event" an event that has no effect then I don't see why it should not be possible. I agree with David Hume when he says that it is possible for an event not to have a cause, so why shouldn't it be possible for an event not to have any effect? Of course, whether it is true or even probable is another matter. I suppose that an event which had no effects would simply be some isolated happening; lonely as a cloud.

Kenneth Stern

back

John asked:

Supposing there are such things as competing basic moral standards, are they equally valid?

I think validity and moral standard go hand in hand. If the validity of any moral standard were in question by the proponent of that standard, then there is no standard at all. Now if we consider whether two standards are equivalent even when opposed, then the answer is Yes.

If we consider whether there is any such thing as Universality or commonality in moral standards then it boils down to asking is there a Universality and commonality in nature itself. Just as we are impelled towards a theory that combines all forces in physics similarly if we believe there is an essential unity in the very nature of things and beings then there has to be a universal element in morality.

I think Jesus captured that element when he stated "Do unto others what you would have them do unto you".

From a relative standpoint there can be opposition but if there is an absolute standpoint, then from that standpoint it should be possible to resolve or understand the difference observed in the relative standpoint.

Gopi Mavankal

back

Ignacio asked:

What is the purpose of the soul in our body? — if it happens to exist?

I like this way of approaching the mind-body problem. We all-too easily assume that we just know what difference it would make whether we have a soul, a substantial, non-physical self or not. But do we know this? So what if I don't have a soul? or so what if I do? What difference does it make?

Consider the following thought experiment. A mischievous demon has been watching me as I write these words. Thousands of miles away, across the Atlantic, Ignacio wonders, as he sips his breakfast coffee, whether the question he has submitted to Ask a Philosopher will be answered this week. Then the mischievous demon decides to play a nasty trick. The demon takes Geoffrey's soul and places it in Ignacio's body, and at the same time takes Ignacio's soul and places it in Geoffrey's body. However, at the moment when Ignacio and Geoffrey's souls are swapped over, all the memories in Ignacio's soul are erased and replaced with Geoffrey's memories, and all the memories in Geoffrey's soul are erased and replaced with Ignacio's memories.

'Geoffrey' continues to write his answer to Ignacio, 'Ignacio' continues to sip his coffee. Neither is aware that anything has happened. — The philosopher who first conceived this thought experiment was John Locke, in the section on 'Personal Identity', in his book, An Essay Concerning Human Understanding (1690). Locke imagined the soul of a prince and a pauper swapped while they lay asleep. But it should be obvious that it makes no difference whether the victims of the prank are awake or asleep at the time.

But has anything happened, in reality? Before we can say that anything has happened, whether Geoffrey or Ignacio are aware of it or not, we need to produce some definition of a 'soul' other than, 'the bearer of memories and the sense of self'. Furthermore, the definition must not be purely negative, such as, 'the part of the person that is non-physical'. Until such a definition can be produced, we do not know whether we mean anything at all when we talk about 'the soul', or whether, on the contrary, we are just uttering meaningless sounds.

Here is another thought experiment. While the soul of Geoffrey continues to think about the answer he is writing, and the soul of Ignacio continues to enjoy the taste of the coffee he is sipping, a second soul of Geoffrey is thinking exactly similar thoughts and feeling exactly similar feelings as the first soul of Geoffrey, while a second soul of Ignacio is feeling exactly similar feelings and thinking exactly similar thoughts as the first soul of Ignacio. But why stop at two? Perhaps Geoffrey and Ignacio each have a dozen souls, or hundreds of souls, or millions.

Or, on second thoughts, for all you or I know, every second that passes, Geoffrey's soul is annihilated, and replaced with an exactly similar soul. Every second that passes, Ignacio's soul is annihilated, and replaced with an exactly similar soul. — These thought experiments were first conceived by the philosopher Immanuel Kant in the section on the 'Paralogisms of Transcendental Psychology' in his book, Critique of Pure Reason (1781).

As in the case of the imagined case of Geoffrey and Ignacio's soul-swap, the question is not simply whether we can know whether or not you and I each have hundreds of souls, or an endless succession of souls, but whether in fact in talking this way any coherent thought has been expressed. To accept that the concept of a soul has no useful purpose, when seen in these terms, is to admit that we don't know what we are talking about when we talk about 'the soul'. It is to admit that when we speak those words, we might as well be uttering meaningless sounds. — That is the challenge posed by these thought experiments.

Geoffrey Klempner

back

Fabien asked:

I have recently heard that during a war the proportion of male births increases. Statistics show a more than sensible augmentation of the male birth rate — as if we had some "social compensating" organ. That would mean we are less individuals than we thought. If this is true which are the actual explanations or theories about it?

Aha! The supposed problem is this; if a population becomes embroiled in a war, many males will be killed, and so the ratio of males to females will fall. Yet, mysteriously, go and examine the same population a few years later and the ratio is back to normal. How can we possibly account for the apparent change in the birth ratio of males to females?

There are two famous explanations. The first is that God has intervened to alter the normal course of nature and correct the gender imbalance. By this, he not only restores His order, which foolish mankind had upset, but demonstrates His very real existence and His willingness to intervene in human affairs. I have heard exactly this explanation put forward by adherents of the Particular Brethren.

The other explanation is that during the earlier phases of the human female's monthly fertile period, the likelihood of conceiving males is equal to the likelihood of conceiving a female. But, later in the cycle, males are more likely to be conceived. As a female can only become pregnant once in a particular cycle, if a couple are having sex at fairly regular intervals it is likely that conception will occur earlier in the cycle. If, however, men suddenly appear on the scene, after being absent for some while, as happens when the menfolk return from war, then the chance of conception occurring later in the cycle is increased, resulting in more male births. The work of the geneticist Koltzoff is usually cited to support this one.

Convinced? I do hope not, because, with a little analytic application, we will see that the problem does not, in fact exist. And we don't even need to look at any empirical statistics.

It is often difficult to examine clearly those problems which are full of fiddly little numbers. So let's imagine an extreme case, and see what would happen there. Say that every man on earth, except one, was killed. That would leave 1 man to about 3000 million women, a spectacularly extreme sex ratio. Whether that one man was particularly sexually active or not, all the offspring of his unions would appear, as they always do, roughly in the proportion of 1 male to 1 female baby. Go and have a look at the population, a year later and we'd find that some babies had been born. Let us assume that he had been busy and sired 100 children, 50 males and 50 females. The sex ratio over the entire population would then have gone from 1:3000M to about 50:3000M, a fifty-fold increase in the proportion of males. This looks impressive, until you realise that to get there the proportion of male to female births was never anything other what we ordinarily expect.

As if that was not enough to damn the proposition, I'm afraid the "social compensation organ" couldn't exist either. For what use would be extra infant boys to an excess of women short of mates? Those boys would not become available as mates for 15 to 25 years, by which time the women would likely be to old to make use of them.

So, now we've done the analysis, let's, like good philosophers, check our results against the facts. I have the figures for Wales to hand, a small country which suffered great losses in the two world wars. The male-to-female birth ratio is typically taken to be 1.07:1, though it varies quite a lot. True, the ratio in Wales before the first world war was about 1.037:1, and after it rose slightly to 1.053:1, but even that was still very much on the low side. But before and after the second world war it remained the same at about 1.05:1. In fact the greatest male surplus was around war-free 1930 when it reached 1.147:1, and it has had upward 'blips' again in 1975 and 1996.

So, there's no need to invoke Godly intervention. I'm afraid the fertility-cycle-conception thing turns out to be nonsense as well, though that doesn't seem to have stopped some less-than-honest clinics trying to sell gender-selection services on the back of it, usually accompanied by an ingenious twisting of Koltzoff's discoveries.

So, are we less individual than we thought? You and I are as individual as we think we are, but when we're considering whole populations then individuality, of necessity, is lost. After all, we are only humans.

Glyn Hughes

back

Brian asked:

I have been reading Berkeley's Principles of Human Knowledge.

He seems to say:

  1. Only God can create ideas
  2. Man can copy and re-create ideas but cannot create from nothing
  3. There are two types of mind, created and uncreated (God) and two type of ideas: ideas of sense, and ideas of the imagination which only come from God
  4. God has no ideas of sense

So where does Man get his ideas of sense from?

I am sure that I have mis-understood the text but in what respect?

Firstly, your points. Man does not copy ideas. Sense experience is a given, and by means of the will ideas of sense can be used in imaginings, so there is a distinction between ideas of sense and those of imagination, the latter being weaker. But the main distinction between ideas is between the simple sense experience, such as visual qualia, and ideas as "things". Berkeley uses the word "ideas" for "things" because a thing is not matter but a construct of simple sense experience ideas belonging to different senses. A tree only becomes a thing, or complex idea, when we have visual as well as tactual experience of it.

God is not "created and uncreated". He is infinite, or eternal and "uncreated", as you can see in the The Principles no. 92. Man is finite and created.

As to your question, man gets his ideas from God even though God himself has no sense-experience. God's way of perceiving is described as "understanding" and in contrast to this our own perceptions are incomplete and appear as sense experience. God imprints or excites these experiences in us by means of his will. We know that God is the cause of ideas because we know that the will is a causal force as we ourselves can "excite ideas to the mind at pleasure". However, when we receive a passive idea, and are not using our wills to imagine something, so another will must be responsible — God. God is the causal force and law of nature, responsible for all ideas and change. It is difficult to rebut this argument when we don't have an idea of cause as other than laws of nature.

A C Grayling has written an excellent book on Berkeley which you might find helpful.

Rachel Browne

back

Gerardo asked:

I have two questions:

  1. Why do we need to know a reason for our existence?

  2. How do philosophers get their questions answered?

1. Well, I don't think we really do need to know (the) reason for our existence. And probably there isn't one to know. Of course, scientists know the causes for the existence of many things, and so do you.

2. The general answer is, by thinking about them. Take a look at 1 above. That's an example of trying to answer a philosophical question. Of course, sometimes, there is no answer. We cannot assume there always is an answer to every question. Moreover, sometimes philosophers only think they have asked sensible questions when they really have not. The question, why do we exist may be one of those. Philosophers have learned to examine the question before trying to answer it.

Kenneth Stern

back

Erika asked:

How do we discover our mission in the world?

There are two assumptions that I can see underlying this question: that we do have a mission in the world; and that it is the sort of thing that we can discover.

Firstly, it can be argued that we do not have a mission in the world. Some argue that the world is ultimately meaningless, a chance occurrence, and that we are also creatures of chance, with no mission. The idea of a mission seems to imply that somebody or something has set the mission. If there is no such somebody (e.g. God) then there will be no mission.

This brings me to the second point. For a mission to be discovered, it would seem that it has to be there already, waiting for us to find it. But there is an alternative. We can set our own mission for ourselves. We invent it, not discover it. This makes more sense to me. I don't mean to say that we just make it up out of nothing. It arises out of our desires and aims, all in the community(ies) in which we live. Social institutions, our upbringing, our chosen circles of friends and so on all come into play. Within this, however, we can make choices about what is most important to us and those we care about, and this leads to us setting up our mission.

Tim Sprod

back

Aaron asked:

I was recently considering writing a philosophy paper for my own enjoyment, but was unable to find a topic that few people had already explored. Has there been much research into the question of whether or not the concept of God is self-contradictory? Can you shed any advice, opinions, and/ or references to aid me?

You know, philosophy is commonly regarded as something that we do after we've done "real" things, finished with our "real" career, and then we'll sit down and write our contemplations in our old age, or our spare time. — No, it's not like that.

Let's say your question above was, "I was recently considering writing a paper in theoretical physics for my own enjoyment, but was unable to find a topic that few people had already explored." Would you seriously write that? Ask that? Why is it, then, that philosophy is different? There are three thousand years of philosophy, in contrast to about two hundred in physics, to read and understand. In the Western philosophical tradition. If you read Sanskrit, you'd need to add another, oh, four thousand or so years of Indian philosophy to that. I won't go into Chinese or Japanese or Arabic (not to mention various mystical traditions). So, to respond to your question, there are probably hundreds of questions that no one has explored, thousands. But in order to understand enough to ask them, much less even suggest reasonable answers to them, you need to spend a little time reading some philosophy. Like about 5-10 years, to start.

Then, if you're brilliant, you can ask (and maybe suggest an answer to) something like, "Are bleen and grue valid categories, really?" To what am I referring? If you don't know, then you're missing basic — yes, basic, as in something all philosophers know about — background.

I cannot even begin to answer the last question, in light of the first. References? Start with Plato and go from there. Aristotle, the Greeks (actually some of the pre-Socratics would be appropriate also); then the Medievals — Thomism, Duns Scotus, Bacon. I've jumped over about 1000 years there, but hey ... and keep going.

Steven Ravett Brown

back

Hathanhliem asked:

Please send to me the sources of Reincarnation in Hinduism and Buddhism.

Refer to the section on "Coming of age in the universe", in the chapter on Hinduism, in the book The World's Religions by Dr. Huston Smith, ISBN 0-06-250799-0. Dr. Smith explains the concept very adeptly.

If you want specific references to the same concept in the Hindu scriptures, the book by Barbara Powell, Windows into the Infinite, a guide to the Hindu Scriptures is useful (ISBN 0-87573-071-X). Dr. Powell gives a translation of the source material and cites the original sources. Her book is also useful to understand the types of books in the tradition.

Gopi Mavankal

back

Charmaine asked:

In statement form:
"Who you are is because of what you are."

In question form:

Who are you, because what are you?"

My question is does this make sense and is there a difference between the statement form and the question form?

Although it was not immediately clear to me, your question does make sense but I think the only difference between the statement and the question is the form!

When you say "who you are is what you are" the implication is that there are facts about you which define you, such as what you do, the nature of your physical and psychological make-up and, perhaps, the history of your experiences. Although true as a statement of fact, this not necessarily true since you can imagine yourself with another life, a different body and psychological type, and with a different history of experiences.

So the statement asserts that the factual definition of a person is true and the question assumes this definition and is asking about the nature of psychological and actual facts.

Rachel Browne

back

Julio asked:

I am a physician, and I am interested in thinking about health, disease and healing. Philosophically thinking, what is health? What is disease? What is "to heal"? I am thinking that the only way we can think about healing is in a metaphysical way, I mean, in the level of the being.

I know this is not only a question, but an issue. Do you have any philosopher you could recommend me in order to think more about this matter?

Here's a resource for you, if you can access the American Philosophical Association publications. They publish The APA Newsletter on Philosophy and Medicine. I'm a member, so I can access it. You might have to ask a member for it. A website for them is: http://www.apa.udel.edu/apa/index.html.

In addition, there are people like Oliver Sachs, whom you know about, I assume. But as far as questions like yours above, dealing with issues on that level of abstraction are concerned: if you're talking about mental health, I think you can find a lot of resources in the field of clinical psychology. If you're talking about physical health — on that level — it sounds as if you might want to do some reading in, perhaps, evolutionary biology, and even in phenomenology. I'm thinking of Leder's book, for example, The Absent Body and some of Merleau-Ponty's The Phenomenology of Perception.

Although there is a great deal of literature on the themes above in alternative medicines and in various Eastern medicines, I'm hesitant to recommend them, because the treatments are largely based on the religious or philosophical principles rather than on science (i.e., controlled experiments), so whatever biases and metaphysical leanings a particular system has is incorporated into its medicine (and its definition of health and illness).

Another possible resource would be to look up some of the various programs now being taught in medical ethics, a hot field in philosophy right now. I'm sure you could find something there along the lines you're looking for.

Steven Ravett Brown

back

Nydia asked:

Did God create man or did man create God?

It is, I think, really important to make the distinction between the concept of God, and God. (Just as we should carefully distinguish between the concept of a tiger and tigers.) Concepts are in the mind, but what they are concepts of are not in the mind. All concepts are created by human beings. But what they are concepts of are not created by human beings. Think of the tiger example. So, the concept of God was, like all other concepts, created by human beings. But I don't think it is possible for God himself (if God exists) to have been created by human beings.

Kenneth Stern

back

Susana asked:

What I want to know is the expectation before we die. I'm used to fear dying, but I think that is a philosophic question. If i die what is next? Why do we die? It's because of the same we are born, or I would never have other life?

We don't know what is next when we die. We die because we are biological organisms. If there is an after-life, there is more cause for fear than if there is nothingness. What is an after-life like, and will you like it? What if you hate it and it goes on forever? If there is no after-life and you simply cease to exist, there is no cause to fear death since when you are dead you will have no mental or emotional life.

Reasons to fear the dying process are pain and leaving those you love and who love you. This is what we fear when we think about dying in the future. But when we have this fear we are not yet in the process of dying. That is a future person and we do not know how we will feel. Death is sometimes welcomed when it actually comes.

Rachel Browne

back

Bertha, Mariana, Arlette, Carlos, and Bere asked:

Why is the day, day and the night, night?

There is no logical reason why the day should be day, or the night, night.

Ever since the invention of curtains and artificial lighting, human beings have had the free choice over their 'day' and their 'night'. It is my free choice if I decide to write philosophy until 6 am, then burrow beneath the bed covers as the sun comes up.

Admittedly, it is convenient if one can count on one's friends or work colleagues being awake at the same time. Just think of the potential pitfalls of picking up the telephone, if you didn't know whether it was 'day' or 'night' for the person at the other end of the line.

However, I believe there would be potentially great advantages to be had from giving up the convention that day should be day, and night, night for everyone living in the same time zone. Workers would no longer require to be paid extra wages for 'night work'. A house would require only half the number of bedrooms, allowing the extra space to be used for more useful purposes.

Best of all, you could watch 'late night TV' all day long.

Geoffrey Klempner

back

Ana asked:

If the philosophy is the love of wisdom, what sense has love in existence?

Do you mean love of existence? If so, the love of wisdom is the love of existence if it is understood as an impassioned pursuit of knowledge of mankind and the world. If by love in existence, you mean the joy of being alive in this best of all possible worlds, then this differs from the love of wisdom which involves rational activity as opposed to delighting in the pleasures of nature. But the two are not mutually exclusive! It is wise to recognise the pleasures of nature and of being alive which is a form of knowledge.

Rachel Browne

back

Anton asked:

What is the philosophical status of auteur film theory today?

Auteur film theory is a rather broad school. To the French Cahiers du Cinema critics, who first developed the politique des auteurs that later became mistranslated as Auteur Theory, it was merely a way of looking at the stylistic similarities between the works of the same director. It soon became inextricably linked with evaluative judgements. There was a clear pattern of authorship in the films of Hawks, Hitchcock et al and this therefore justified valorising these directors above others. It became a way of justifying the approbation of many popular Hollywood films rather than just the output of 'arty' Continental film directors.

To these French writers Auteurism was a loose, contradictory, open critical approach; the idea of personal authorship in the cinema. The fact that Barthes, Foucault and Derrida were at the same time challenging the very existence of an author and querying the preferring of authorial meaning was contradictory but not a problem. Anglo-American writers turned it into a much more precise formal theory. Andrew Sarris was the first of many to seize on Auteurism and construct a formal theory based aesthetic evaluation of film directors (to justify his personal view that Hollywood films were superior to Continental films). You can see here the loose relaxed Continental philosophy inflecting the French writers while the more demanding, definition seeking, theoretically based Anglo-American Analytical Philosophy school feared contradiction and demanded more close analysis and certainty among Anglo-Saxon academic institutions.

In relation to its current philosophical status, it sits embedded within film aesthetics and is an example of the debate about whether artists or their art should be the unit of aesthetic analysis and valorisation. Should aesthetics be considering the author Hitchcock or the film Psycho (in the same way other arts agonize over whether it is Beethoven or The Moonlight Sonata, Van Gough or Sunflowers, etc.) Film writers like Bazin plead for the film/ text, as, in philosophy writers like Scruton argue for the piece of music in his The Aesthetics of Music. Truffaut argues for film authors just as in literature Harold Bloom argues for poet/ authors.

This debate is often reduced to statements like "there is no art without artists" versus "it is art that defines an artist" — a variant on the "chicken and the egg argument" — and it seems equally pointless. The argument is peculiar to Anglo-American Aesthetics. Continental aesthetics is much more flexible. Why should it be one or the other? Why not both? The concept of an artist is in any case no more than a critical construct. It is not the flesh and blood Hitchcock that we analyse but 'Hitchcock' — a perception created from his films. Maybe the artist and his art are indivisible in aesthetic terms.

There is also a problem with Auteurism and all authorial aesthetic analysis as to where to draw the line. Is everything that Hitchcock/ Beethoven/ Van Gough produced a work of art? What about Van Gough's rough sketches?, Hitchcock's home movies?, Shakespeare's laundry lists? Auteurism struggles to produce a coherent answer.

That being said Auteurism, in the form of authorial stylistic and aesthetic analysis, is now fundamental to most film aesthetics, just as authorial analysis is basic to most art aesthetics. The Spring 2001 issue of the Internet film magazine Screening the Past is entitled 'Auteurism 2001' and contains a number of articles carrying on the Auteurism reinterpretation. No one denies that there are authorial styles identifiable within films. However the danger is that authors with the most individual personal imprints (e.g. Tarantino) are excessively valorised because they have a strong intrusive style that can be clearly identified, discussed and analysed, compared to more subtle authorial styles which integrate cinematography into the narrative (e.g. Ang Lee). As Pauline Kael has noted a skunk smells more than a rose but that does not make it better.

Andrew Browne

back

Karla asked:

Love: Is it causality, or is it destiny?

You pose this question as an either/or, but I don't think that it is. Of course, it depends on what you are taking 'destiny' to be, but it seems to me that causality and destiny amount to the same thing.

This is why. I take causality to mean the position that everything that happens has a determinate cause. So, falling in love is just the effect of a lot of previous events, all of which jointly cause the events that come to be described as 'falling in love'. Given the state of events at some time in the past, before the lovers have met, then long chains of causes and effects bring about the falling in love.

So what is destiny? I take it to be the position that the lovers were destined — they could not avoid — falling in love, even before they met. If the sort of strict causality I outlined above is true, then it is also true that the lovers were destined to fall in love. Causality and destiny are the same.

Of course, you can think of destiny as being something different — e.g. a god or gods deciding to bring something about and hence interfering with causal chains. Then they are not the same. But under my interpretation, both causality and destiny are consequences of determinism — the view that things could not have turned out differently than they have, because people cannot make free decisions between alternatives. That in itself is a tricky philosophical problem.

Tim Sprod

back

Seaton asked:

"They" had slavery around here once, and they liked it.

Whilst working at my job as a firefighter, many years ago, I coined this phrase. I spoke it in a response to my fellow crewmembers about the poor managers we had to cope with, as opposed to being supported and aided in my efforts on a day-to-day basis fighting fires, responding to first-aid calls, administrative and tedious tasks, etc. etc. It 'caught-on' by many, and discussions eventually evolved into the need to re-activate the long-gone Union, the International Association of Firefighters, Local 526.

Here's another example of what I mean:

Yesterday, my daughter returned home from her summer job at a local fast-food restaurant. In a fit of rage, she informed me that the manager (one of several supervisors, actually) told her coworker and herself that "I just wanted to see if you could do it."

"It," what the supervisor was referring was to the fact that instead of 3 persons being scheduled to work for the initial 4 hours of operations, 2 were scheduled. Yes, most of their tasks were done---but not without some extraordinary-efforts "demanded" from the employees. Energy expended by the employees was more than commonly given. Instead of comfortably accomplishing all the tasks with 3 persons within the time limits, 2 were worked overly-hard and nearly all tasks were accomplished and nearly all were satisfactorily done. Results? Both employees ready to quit.

Why only 2 employees instead of 3? Because of the capriciousness of the Supervisor? Or...."They had slavery around here once, and they liked it"?

This question intrigues me, but I don't really know what kind of answer you're looking for. Philosophy gives very general answers to abstract questions. Here's one of those: slavery is a very good economic system, for the slave owners. Think about it: what do you pay a slave? What kind of housing, food, etc.? Slavery is cheap. As long as you can keep revolts down, and you don't care about slaves as human beings, you're doing fine. How to keep revolts down? Well, why not educate a class of people to believe that suffering is good; that if you suffer in "this life" you will be rewarded in the "next"? A great philosophy for slaves, right?

Here's another answer: the supervisor was a sadist and enjoyed making people suffer.

Here's another answer: the supervisor was under extreme pressure from his boss to make the restaurant profitable, and was trying to cut costs, at the price of suffering. Was it worth it? Well, would there be more suffering if the restaurant went out of business, and the two remaining employees, and the supervisor, were out of jobs?

Here's another answer: when people do work which requires little skill, they are replaceable, since anyone can do the work. So unskilled labor, or labor requiring just a little education, has long lines of people just waiting to fill a few jobs, and employers can do just about what they want. If the workers don't like it, they'll find ones who will from a large pool of unskilled people. The answer? Educate yourself in some skill, preferably one you enjoy, and use that to earn a living. You will not be replaceable (or as replaceable) and so you'll be able to dictate your own work situation to a much greater extent. You have done that, as a firefighter. In the meantime, while you're educating yourself, you have no choice but to join the pool of unskilled labor (but that's not all or nothing; you can get better jobs as your education or skills increase).

I can tell you, from experience, that the latter works; I learned computer programming while working as a word processor, and the jump in status and pay was huge.

Steven Ravett Brown


It took me a while to cotton on to what you are saying here. There seems to be a direct analogy with something that is sometimes remarked about war. We all deplore the fact that wars take place. We investigate the economic, political or religious causes of war, and how these conditions might be changed. We look forward to a utopian time when there will be no more wars. Yet there is a major flaw, it is said, in this approach. It ignores the simple, brutal fact that men love war. Many of those who go out to war discover, sometimes to their horror, that they enjoy fighting and killing. Seasoned soldiers interviewed about their experiences in battle have described how they get a sexual high. My impression from what I have read is that this is common knowledge within the armed forces, however shocking it may seem to civilians who have fortunately never had to pick up a gun.

The claim made is that this is an unalterable fact about human nature. I am not going to dispute that claim here. If the claim were true, then one plausible explanation might be that it is part of our evolutionary inheritance. A capacity for aggression, in certain species, is necessary for survival. It follows that a race of intelligent beings who did not share our evolutionary inheritance — Martians, say — might never experience the pleasure human beings experience from mortal conflict.

I am not totally convinced by this, mainly because it leaves out the crucial issue of the thrill of killing. Natural aggression might lead us to fight, but must that fight be to the death? At any rate, the claim seems less plausible when transposed to the issue of slavery. Animals fight and kill, they wage 'war'. But to make a slave of another individual, as opposed to merely exerting one's force to bring about changes in their behaviour, requires a development of self-consciousness that only exists in the human world. The impulse to make slaves cannot, therefore, be part of our evolutionary inheritance.

The classic analysis of our love of slavery is in Hegel's famous section 'Lordship and Bondage' in his book The Phenomenology of Mind (1807). The section is difficult and obscure, but given your interest I would say that it is something you must read. Hegel argued that the dialectic of the relation between self and other plays a pivotal role in the development of our sense of self, and that the impulse to seek to make a slave of the other, is not contingent but universal. (In our terms, Martians would naturally succumb to the impulse just as we do.) Slavery is a solution to the problem of how to reckon with the existence of the other, but a failed solution, one that, as Hegel demonstrates, is necessarily self-frustrating.

Though you give examples from the world of work, our most immediate experience of this impulse is in personal relationships. It is universal. It is not an unfortunate 'deviation' that some persons suffer from while others remain immune. Those who would emphatically deny this, have merely failed to recognize the impulse for what it is.

Geoffrey Klempner

back

Alex asked:

I read somewhere that, "Art upsets, science reassures." — Can you explain that please? Can't it be the other way round?

Not knowing who said this or why, I can only guess at what it means. It could be that art presents us with the unknown in the sense that our responses, our aesthetic ideas and our concept of art itself are not reducible to facts. Even if we take art as consoling, the how and why is not reducible to solid facts either. Science reassures since it enables to confirm and increase our knowledge and extend our powers over the world.

However, this seems absurdly theoretical. Art doesn't in fact upset us, but does console us and provides us with an enormous of pleasure. So you can change this around since science can be held to upset us because it in fact leads us to less natural forms of behaviour or some such thing. If you want to change this around, you can stick to facts. It is less plausible to say that science upsets us because it arouses in us fears that things may not be as they seem, because for most persons this is not the case. No normal person has such fears! This sort of thing is only considered in the philosophy class room when we consider the difference between our sense perceptions and what the physicists claim there is. However, if statement "art upsets, science reassures" is understood as a philosophical claim, this abstract, theoretical fear might lead us to deny its truth.

Rachel Browne

back

Richard asked:

I would appreciate it if you could lead me to a one or two page summary of the philosophy of Boethius. I hope this request is realistic.

I am giving a course on 'The Western Mind' and I believe that some discussion of Boethius would be appropriate.

What is wonderful about Boethius (c.480-254), even more than what he thought, is that he thought. Here is an excerpt from a lecture of mine on Boethius:

Boethius was an aristocrat and a scholar and a high government minister of Theodoric the Ostrogothic King who ruled Italy between 493-524; but Theodoric was a Byzantine (as opposed to Roman) Emperor. Latin and Greek cultures were separated long before their final divorce in 1054. Boethius was implicated in a plot to overthrow Theodoric — although Boethius asserts his innocence in The Consolation of Philosophy, which he wrote while awaiting execution. In all probability Boethius was innocent as he protested. In any case, he was arrested, tried, sentenced to death and sent into exile in Pavia to await execution. On the dread day ropes were twisted around his neck until his eyes popped out, then he was finished off with a bludgeon. Awaiting execution Boethius wrote the Consolation.

The Consolation is one of the greatest books of Western tradition and probably the single most universally appealing book of the whole Middle Ages. The Consolation was translated by Alfred the Great into Anglo-Saxon around 890 and into English by Chaucer and Elizabeth I. It was translated into all known languages at a time when translating books into the vernacular was extremely rare indeed. The Consolation was the favourite reading of three of the greatest literary giants of all time: Dante, Boccaccio and Chaucer. The Consolation was an international best-seller for over a thousand years. No single book by a single author has ever known such renown or longevity — perhaps Augustine's Confessions. But compared the Confessions, the Consolation has even broader appeal. Until a couple of hundred years ago, C.S. Lewis tells us, it would have been hard to find any educated person in any European country who did not know and love the Consolation. To acquire a taste for this book, Lewis also says, is almost to become naturalised to the Middle Ages.

On top of this sublime achievement, Boethius was more than anyone responsible for giving Aristotle to the Middle Ages. And not just Aristotle, but an Aristotelian way of doing theology which is known as Scholasticism — although Boethius died 600 years before his method of doing theology was to really be developed (in the High Middle Ages). In addition, Boethius bequeathed to posterity the main intellectual problem of the High Middle Ages: the problem of universals. Not only did Boethius bequeath the main problem which was to tax the best thinkers for more than two centuries five hundred years after his death, not only did he bequeath the method of treating the problem, he also made the translations of the main texts of Aristotle which were to be used as a basis for what would develop into rationalism and science. At a time when few scholars knew Greek and Latin equally well, Boethius translated key arithmetical and astronomical works (e.g.. Euclid's Elements) which were to have immense influence on the development of science as well as works on musical composition which were to have crucial significance for the development of Western art. The so-called Quadrivium of Medieval education was founded by Boethius. Cassidodorus, a contemporary, wrote in 507: "By your translations Latin readers now have Pythagorus' music, Ptolemy's astronomy, Nicomachius' arithmetic, Plato's theology, Aristotle's logic, and Archimedes' mechanics" (Variae i, 45, 4).

It was Aristotle who first started using letters as variables in the construction of his logic. Boethius' two monographs on Aristotelian logic transmit this arithmetical lexicography to the Middle Ages. It is important to note that Boethius was a traditional theological authority of the highest order and that by default he validated what would undermine the continuity of that order. For he validated innovation in refining and improving Aristotle's logic. This gave an authoritative basis for continuing such a practice. Not everyone's innovations would have the tact and prudence of Boethius'. Abelard's logic is precocious, academic and subversive.

End of excerpt. All this seems to me more crucial than what Boethius thought. He was not an original thinker in our sense, but like all original thinkers of his day, traditional. He was a Christian Platonist to put it in two words. That means that he combines the sense that we are all puppets in Plato's cave (i.e. living in illusion, including self illusion — sin) with the idea of one God, like Plato's one light of truth, goodness and beauty. But man, in the image and likeness of God, can know logic, which in a Dark Age, is akin to light and will carry him to the light by enlightening him from within. This light is the light that was in the beginning with God (see intro to John's Gospel). Boethius is Christological in the patristic sense in which Christ is 'logos' or word by which the world is made. This logos has its logic, which is that of reason as Aristotle learned it from Plato (he does not read these two in opposition). It is the literary quality of his Christian Platonism in the Consolation, which is so perfect.

Matthew Del Nevo
http://www.sicetnon.com

back

Jordan asked:

My daughter's boyfriend smugly stated that "Everyone has a price" and that of course he "would kill someone for the right price, as would anyone if they were being truly honest!"

I was caught somewhat by surprise, and realized that this boy was proud of his personal revelation and felt that my wife and I were simply not being honest or rational enough to face the facts.

I feel strongly that the world would not work well if everyone held his views and that I would not enjoy my life if I lived it based on this model. But beyond Biblical or Dickensian arguments, I couldn't buttress my views on his own terms.

What should I have said?

You can't argue against a nihilist in moral terms, because he can just deny what you're saying and it is in the nature of morality that this can be done. A point you could have made is that morality is based on the recognition of the reality of the subjectivity of others and if he can't see this then there is a major part of reality which is passing him by. But he can deny this too, saying "that's your reality". As far as I can see, all you can do is shift your ground and attack him on his logic. That is, he is holding a belief about others based upon his own case — that he has a price — and it an invalid inference to move from one's own case to that of others. Of course, he might reply this is true of you too, but at least there'll be a stalemate and he can't claim to be right.

Rachel Browne


First, what is a "price"? Can you compare money, for example, and someone's life? How much, in money, is a life worth, and how do you even compare the two? Would a "price" be the saving of other's lives? Is it worth killing one person if you can guarantee saving the lives of two others?

So I think the first thing to do is establish that his question is basically incoherent: he hasn't defined his terms and probably hasn't even thought about the implications of, for example, comparing life and money.

Second, that kind of comparison, even if you can do it, implies that there is something quantitative involved; something that can be added or at least compared as to magnitude. The "worth" of a "life" is a quantity, in this view, that can be laid next to another quantity, like money or other lives, and the two compared as to size or weight. Well, there are philosophers who believe that: the utilitarians (based on Bentham, but carried much further now than his rather naive approach), and they have had very elaborate schemes to assign "magnitudes" or something like them to "the good" and then compare those magnitudes. Now if this guy were sophisticated he could espouse something like that and muster reasonable defenses. But "price" then would be a much broader term, involving some kind of measurement of happiness. There are very complex game-theoretic approaches based on this kind of assumption.

On the other hand, there are probably more philosophers who think that approach is bunk, basically, and that you cannot compare "goodness" or "life" or "price" like dollars or hamburgers... that you have to either go by some kind of intuition as to which of two (or more) courses is better, or get unanimous consent or something like it. You might take a look at Rawls' Theory of Justice on that one. His position is a pretty sophisticated methodology for arriving at consensual agreement.

Steven Ravett Brown

back

Cord-Christian asked:

I would like to get some ideas for this one (I'm learning English at the moment, so I have some problems to tackle the question correctly).

What counts as an experiment? Can experiments be undertaken in human sciences or only in 'normal' sciences? Are there some necessary conditions for an activity to be an experiment, e.g. hypotheses, data, manipulation of variables, observations, generalizations and expectations of outcome?

I cannot do better than repeat the entry from the Oxford Dictionary of Philosophy edited by Simon Blackburn:

A controlled manipulation of events, designed to produce observations that confirm or disconfirm one or more rival theories or hypotheses. To experiment is to put questions to nature, and the experimental method is contrasted with the passive acceptance of whatever observations happen along. The method is characteristic of modern natural science. However, a discipline (such as history) may be pursued with greater or less objectivity and success without being able to avail itself of the experimental method.

Kenneth Stern

back

Matthew asked:

The evolution thing

Well, I don't really know all that much about evolutionary theory but something just occurred to me. For example, of what possible advantage is it to an organism to have a slightly opposable thumb? Surely it is no advantage, and if so then how could even more opposable thumbed creatures hope to evolve given that the slightly opposable thumbed creature will be no more successful than its counterparts?

So surely at least some of the developments we see in organisms must be due to sudden radical change rather than a gradual process as envisaged by evolution? And this will have the character more of luck than adaptation. i.e. an organism luckily develops something (through mutation or whatever) that just happens to be beneficial to it.

Is this at all right?

I'm afraid the idea that radical, evolutionarily advantageous changes might develop 'by luck' is extremely weak. As a solution to the problem you have posed, it is a desperate last resort. If there is going to be any useful explanation, we have to stick with small changes brought about by genetic mutation. So the challenge is to demonstrate how these small changes — for example, a slightly opposable thumb — might confer some evolutionary advantage.

Actually, the example you have chosen is not that difficult. The stock examples are the evolution of the eye, or a bird's wing. We shall come to those in a moment. Let's look at your case. The first point to make is that an evolutionary advantage can be measured in very small percentages. If a 'slightly opposed thumb' gives just one tenth of one per cent increase in the chance of successfully performing a given task, then given the scale that we are working on — thousands of generations, millions of individuals — that will work its way through. The proportion of individuals with that trait will increase slowly but inexorably. Then we can run the some process through again, with a further slight modification, and so on.

Richard Dawkins, in a Royal Academy lecture series for children a few years ago, brilliantly took up the challenge of the wing and the eye. What good is a tiny fluffy protuberance that might develop into a wing? Well, an animal that lived in trees might have a fraction of a per cent less chance of dying as a result of falling out of the tree because the protuberance slightly breaks its fall, or because it slightly increases the animal's wind resistance. An eye might first start of as a few slightly light sensitive cells. The chance mutation of a narrow ridge of skin around the patch would cast a slight shadow, which would give the ability to distinguish very crudely between different positions of the light source. You can work out the rest from there.

As Dawkins graphically described in his lectures, evolutionary theorists have not been content to sit in armchairs and speculate. The power of computers allows the possibility of constructing detailed, testable hypotheses. How would a given variation work out after a thousand generations? or ten thousand? or a hundred thousand? The test can be run through in a few seconds.

When we try to apply these theoretical ideas to the massive complexity of the world of living things, with or without the aid of computer models, the imagination balks. We are only able to see, to understand, a very small part of the picture. From that we infer to the whole. It follows that we cannot prove that the inference is correct. But no theory in science is ever proved once and for all. The only claim made is that the theory of evolution by natural selection is, all things considered, the best explanation.

Geoffrey Klempner

back

Ravi asked:

What is the nature of necessity involved in physical laws? In what way it is distinct from logical necessity? What role do counterfactuals play in distinguishing law statements from accidental generalizations?

Laws of nature are governed by natural necessity and we come to know such laws by observation and confirmation in particular instances. When natural law is understood as causal, one event is supposed to be necessary and sufficient for another, although the predictability which this implies is called into question by quantum physics. However, at an ordinary macro-physical level, if water is heated to 100C it will be boiling. This is necessary because it would not be boiling without having reached this temperature and sufficient because the water's being at this temperature guarantees that it is boiling. We understand this sort of necessity as exceptionless and lawlike, given the proper conditions.

Logical necessity, as it relates to counterfactual situations, is that which we cannot imagine to be otherwise, or that which is true in all possible worlds. It is not logically necessary that when water is at 100C that it is boiling. This is a law of nature, determined by the actual world. An example of logical necessity understood as what we cannot imagine to be otherwise, is that an object cannot be red and green all over. This is because we cannot perceive two different colours at the same time nor imagine what it would be to have a sensation of something's being red and green at the same time. This is not just true for secondary qualities, but also primary qualities. An object cannot be both round and square: We cannot imagine what such a counterfactual state of affairs would be. This is an a priori approach to identifying logical necessity.

However, Kripke has argued that the identity of names determined by reference, when considered in terms of counterfactual situations, is a natural necessity. It is arguable whether the identity of Hesperus and Phosphorus — the 'evening star' and the 'morning star' — is logically necessary. If logical necessity is what we cannot imagine to be otherwise, it is not logically necessary because it is conceivable that each name identifies a different star. However, for Kripke, a counterfactual state of affairs is something we posit within our language and when we refer to Venus we mean our Venus.

The laws of nature can be held to be contingent, i.e. it is possible that water doesn't boil at 100C. However, on Kripke's view of natural kinds such a state of affairs wouldn't involve what we call water, but rather something that appears to be 'water', but isn't. I think the role of a counterfactual is to test our conceptual commitments. We won't give up our commitment to water's boiling point when we consider a possible world. If we consider the state of affairs in which water boils at 35C we suppose that it is not water. If, in fact, we found water was boiling at 35C we would still take the stuff to be water if it was H2O and wonder what would explain this change in natural law. Counterfactuals highlight our commitment to essentialism for natural kinds insofar as we take water to be necessarily H20 and the contingency of the laws of nature which seem to be exceptionless and lawlike, and so we take them to be necessary in this sense, but they could change. Our notion of causation is our way of understanding the world but we have no grasp of the significance of causation or whether the world could change.

Kripke's view of counterfactuals and necessity is not subscribed to by everyone. There is a lot of debate about counterfactuals and the nature of possible worlds. Since you have been interested in counterfactuals for some time, I wonder if you have read An Introduction to Philosophical Logic by A C Grayling and Causation and Conditionals edited by E Sosa.

Rachel Browne

back

Eduardo asked:

Which is the perfect lifestyle?

I think this is the wrong question. I think the question would be better as "What is the perfect lifestyle for me?" If we agree that different people have different likes, preferences, values and commitments, then no one lifestyle will suit all.

But even my new version seems wrong to me. It assumes that we can rank lifestyles best to worst. But if we agree (and I think there are good arguments that we must) that each person's likes, preferences, values and commitments do not form a perfectly coherent whole (so that some preferences are somewhat at odds with others — e.g. I want to be a sports star, and I want to travel extensively while I'm young), then while we can say that some lifestyles are better for me than others, we will come to a conclusion that, amongst the better ones, this lifestyle (training every day) is better in this way (making me a sports star), while that lifestyle (setting off overseas on an open ticket) is better in some other way (travelling), and there does not seem to be any way of saying that one is absolutely better than the other.

So now the question becomes, for me, "What would be a good lifestyle for me — one I would be happy with?" And while asking other people is a perfectly good way to go about answering this, there will not be a single answer for all of us, or even for me alone, and I will have to make my choice as best I can amongst the alternatives. When I do, some possibilities open up and others close down. As long as I end up happy, it doesn't matter that some of my preferences were never fully met.

Tim Sprod

back

Melanie asked:

I have to give a class presentation on existentialism. Can anyone please explain it to me without all those big text words? The philosophy book is so hard to understand.

Existentialism is not a philosophy. It is something different philosophers, both modern and ancient have in common. The appellation "existential" for such philosophies is modern, to distinguish such philosophers or philosophies from the dominant schools of modernism, rationalism on one hand and empiricism on the other.

To return to your query: what is it that these philosophers/philosophies have in common? It is a concrete sense of our existence. Now this concrete sense may be understood in our ordinary way as experience. Therefore, an existential philosophy is about experience, either it starts with experience or refers back to it; in either case, experience runs a check on it. But some existentialists find "experience" too psychological as a category, too subjective, therefore, too liable to be hijacked by the modes of rationalism, and its concomitant, empiricism. Existential experience is not so much subjective and individual for these philosophers, but world-historical, what they would call "essential", an idea which hearkens back to the possibility of experience of the eternal in Christian philosophy, much of which, in the early days, and always in the Orthodox world, related to experience. As Paul said, my experience is not "my" experience, "but Christ's in me". In the age of subjectivity we can't think of our experience this way very easily or naturally, but some existentialists still try and think experience outside the category of the ego. The most famous and successful examples are that of Hegel and Heidegger.

Matthew Del Nevo
http://www.sicetnon.com

back

Lindsey asked:

Can you tell me what Amoralism, Subjectivism, and Relativism all have in common? What similar relationship/ characteristic do they all share in their claims? And what is the flaw in their similar claims?

One thing that amoralism, subjectivism and relativism have in common is the denial of the objectivity of moral value. The amoralist thinks morality is something you are into or not, the subjectivist takes morality as a matter of personal sentiment and the relativist takes morality as culturally valid but not objective or universal. These moral attitudes arise in part from the problematic position of moral objectivity in its attempt to establish moral truth or a moral reality which transcends ordinary fact.

However, it is not clear cut. A weak relativist might admit that while values are cultural in fact, there is a possibility of agreement if cultural differences can be settled in some way. However, this person would not strictly be a relativist if he thought that there was the possibility of ultimate agreement on moral value amongst humanity in general. On the other hand, if a relativist is not simply a subjectivist, he must recognise the claims of culture which is a move towards, not objectivity, but at least intersubjectivity.

It is moral objectivity — or intersubjectivity — which the three positions do not recognise. The three positions fail to acknowledge the reality of our recognition of the value others as actually having importance in our deliberations towards action, or as just mattering because we care about them. The flaw is that the three positions are not moral positions at all. Rather, they deny the actuality of morality or reduce morality to sentiment or cultural codes.

Rachel Browne

back

Greg asked:

I have several questions:

  1. If the universe is expanding, what is it expanding into? There must be an edge because it is expanding into something.

  2. Are there an infinite amount of numbers between 2 and 3? For instance 2.3, 2.5, 2.6, 2.6564745645.....?

  3. How seriously do philosophers take near death experiences? I think the common themes from each culture says something.

  4. Who let the dogs out?

1. Let's say that you were a 2-dimensional creature living in the surface of a sphere. There was no way you could see off of that surface: light just moved in the surface. So that sphere is your universe, and you can go around and around in it; there's no edge, but it's finite. Ok... now, the sphere, let us say, is expanding. Well, as far as you are concerned, the universe is getting bigger, but it's not expanding into anything. It's just expanding. Is our 4th-dimensional (or maybe 11th, if string theory is correct) expanding "into" anything? Well, we'll probably never know ... and if we did, then the problem would be just pushed back to the next level, wouldn't it? But if there's no way we can get off, or even see off, then we can speculate all we want, but that's all it will be.

2. Yes. You're talking about the "real numbers", and there are an infinite number of them. In fact, a mathematician named Cantor proved that there are, first, a countably (i.e., you can count them, one by one) infinite number of integers (the whole numbers: 1, 2, 3... you just can't finish counting them... that's why they're infinite); second, a countably infinite number of rational numbers (i.e., all fractions); but there are an uncountably infinite number of real numbers: numbers like pi, square root of 3, and on and on. If you're trying to count them, you can't ever get from the first to the second, because there are an infinite number in-between any two. In fact, that latter infinity (of reals) is just the second order of infinity (the first Cantor called "aleph-null", the second "aleph-one"); there are an infinite number of orders of infinity (I don't know if anyone has proven whether the number of orders is countable or not). Aleph-two is the number of functions, for example.

3. From my point of view, the validity of near-death experiences (in the literature, NDEs) is not a philosophical question or issue, but a medical and psychological one. Are they real? Yes. Do they indicate what people want them to indicate, i.e., that there is "life after death"? Well, you have to look at the literature for that. My take on it is that they do not; that they are interesting, and due to somewhat rare conditions in the brain (oxygen starvation and massive release of particular neurotransmitters — glutamate, mostly) when it and the body are under great stress — like, dying. It is, of course, extremely difficult to do controlled experiments here (and a little ghoulish, don't you think?), but it's been tried, and the results seem to be that we don't really float around, etc., because people can't really see into closed drawers, etc. When, with the people who were saved, and what they seem to have seen is investigated afterwards, it seems to have been just inferences. The accuracy is mere chance. But this is still an open field. The uniformity of the experiences, by the way, is probably due to the uniformity of the physiological events.

4. Socrates.

Steven Ravett Brown

back

Paulo asked:

I've never heard anything but specious remarks about the problem of evil. Any Comment?

My dictionary defines "specious" as "deceptive" or "inauthentic" although with a "pleasing appearance." I wish you had given instances of these remarks you call "specious," I would have not had to reply in a vacuum. Perhaps you mean only that replies to the question as to how evil is consistent with an all good and all powerful god do not seem satisfactory to you; but neither have they seemed so to many philosophers starting with David Hume.

Perhaps you should ask a more specific question.

Kenneth Stern

back

Rose asked:

I am doing an essay on the surprise examination paradox. I know what it is, and how the argument goes, but I would like to know: is it a genuine paradox? why or why not? what has 'vagueness' got to do with it? what is the answer?!

Where should my essay take me, and where can I find decent arguments on the web?

A teacher tells her class: "You will have a logic exam at 10 am one day next week. I'm not telling you which day. It will be a surprise." One bright student reasons as follows. "Teacher can't wait until Friday to give the exam, because by then we will know that it has to be on Friday, so it won't be a surprise. However, if Friday is ruled out, then by the same reasoning the exam can't be on Thursday either, because by then we will know that the exam has to be on that day. The same reasoning proves that the exam can't be on Wednesday, Tuesday or Monday. Therefore, our teacher can't give us a surprise logic exam!"

Confident that there will be no logic exam, the bright student doesn't bother to revise. The following Wednesday, the teacher hands out examination scripts. Boy, was the student surprised!

I know that some philosophers make a lot of this paradox. (Richard Montague in Formal Philosophy is one who takes the paradox seriously. I don't recommend that you attempt to read that inpenetrable book.) In my view it is not, in fact, a genuine paradox.

First, let's get rid of the magician's hand waving. The specious plausibility of the paradox derives from the fact that the students are given a choice of five days. Watch what happens, however, if we reduce the number of days to just two. "You will have a logic exam at 10 am either tomorrow or the day after tomorrow. I'm not telling you which day. It will be a surprise."

Now things are a lot clearer. It is true that the teacher is still able to surprise her class. Tomorrow 10 am comes, and the teacher either gives the exam or doesn't give the exam. Either way, the students 'are surprised'. They didn't know whether they would get the exam on that day or not. However, it is also clear that when the teacher said, "It will be a surprise," she could not have meant, "Whenever I give you the exam, you will be surprised." This is something she knows can't be true. They can be surprised by the exam's not being given tomorrow. They can't be surprised by the exam's being given the day after tomorrow.

In my view, therefore, the surprise examination paradox comes into the same category as the infamous barber paradox. Suppose I tell you that there is a barber in Sheffield who shaves all and only those Sheffielders who do not shave themselves. Does the barber shave himself? Answering either "yes" or "no" leads to a contradiction. If he does, he doesn't. If he doesn't, he does. This is not a genuine paradox because the response is simply to say that the statement, "There is a barber in Sheffield who shaves all and only those Sheffielders who do not shave themselves" cannot be true. The statement is self-contradictory. End of story. We should say the same thing about the surprise examination paradox. The statement, "You will be given an exam one day next week and whichever day it is you will be surprised" cannot be true. Only on the assumption of its truth could the bright, but easily suckered student have reasoned as he did. That reasoning is what shows the teacher's statement to be self-contradictory.

I don't know where you would find material on the surprise examination on the web. Try a search for the exact phrase "surprise examination paradox" with Alltheweb. I have found that particular search engine to be extremely accurate with text string searches. If there is just one page on the web with that phrase, Alltheweb will find it.

Geoffrey Klempner

back

Magdalena asked:

If simultaneity of times in different dimensions or spaces is not verified, how can one explain "deja vu"?

When we experience déjà vu, we feel something which is happening has happened before and I'm not sure if this involves different time dimensions, since it essentially involves memory, which is of the past. I like the theory that déjà vu is concurrent memory and consciousness of a single situation. It is thought that the brain races forward and acquires information before it enters consciousness, so informationally, in the brain, there is a memory. When we then become conscious of the situation, the brain matches this with what has become a memory as far as the brain is concerned — since it rushed forward. We then have both a memory and consciousness of a current situation.

Rachel Browne

back

Rebecca asked:

Two questions:

1. Is morality dependent upon religion?

2. What is truth?

1. No, morality is not dependent upon religion. Plato (through Socrates) argues quite convincingly in the dialogue Euthyphro that — to put it in modern terms — things are not right (morally correct) because a god says they are right, but rather that a god will say those things are right because they are already right. So if there is an independent way to say whether something is right (morally correct) or not, then god is not necessary. Just what it is that makes things morally correct is more difficult to say.

My favorite example of this is the Abraham and Isaac story from the Bible (especially as told by Bob Dylan in the first verse of Highway 61:

God said to Abraham "kill me a son", Abe said "man, you must be putting me on". God said "Abe", Abe said "What?", God said "You can do what you want, Abe, but the next time you see me coming you better run". Abe said "Where do you want this killing done?" God said "Down on highway 61".

So, why did Abe think God was putting him on? Because to kill an innocent son is immoral, no matter whether God tells you to or not. Even (especially?) if He threatens you.

2. If I knew the answer to, 'What is truth?', you would be reading it in my bestselling book which would have made me the most famous philosopher of all time.

Seriously, there are many views about what truth is. Briefly and roughly, the correspondence view says it is whatever beliefs corresponds to reality. The coherence view says it is whatever set of beliefs coheres (fits) best with all other beliefs and experiences. The consensus view says it is whatever beliefs are held in common amongst those who have jointly tried to find it. The pragmatic view says it is whatever beliefs turn out to be the most useful to have.

There are many versions of each, and they are not all mutually exclusive either. There are also some other theories on truth I haven't mentioned.

Tim Sprod

back

TK asked:

In the Yin/Yang philosophy it states that opposites are necessary in order to create the universe. If this is the case, since one polar is considered to be "good", the opposite polar being "bad" or in the case presented "evil", is evil a necessity in order to create balance in the universe?

The Yin/Yang philosophy, as you call it, is about balance, not opposites. It is the Taoist philosophy in fact. Good and evil are not postulates let alone abstractions, they are what is the case if we examine our bodies or the world around us (i.e.. our experience and that of our families). There is a circular relationship between the two, rather than an oppositional relationship, in this philosophy. Good health becomes bad health if it is too good, e.g.. the growth of cells becomes cancer after a certain point. Beauty becomes ugly and so on. Cold burns us, heat eats us away like cold. The truth lies between, in the balance. The Yin/Yang philosophy, as you call it, is an avoidance of extremes. Its main postulate is a middle way.

Matthew Del Nevo
http://www.sicetnon.com

back

Cynthia asked:

Does happiness exist?
If the answer is yes, how can we reach it?

Of course happiness exists! There's a John Wayne film in which a character asked John Wayne "What is the secret of happiness?" and John Wayne replied "One thing". The questioner asked "What thing?" I think Wayne just replied "one thing!" OK, you might not think this is funny.

If there is nothing good in your life it is difficult to be happy. But if you have just one really good thing it's easy. The "one thing" will not be the same for everyone.

Rachel Browne

back

Christian asked:

What is man?

Man is a human being who does not bear children. Emotionally, he is the weaker of the human species and because he does not have to be prepared for childbirth, he cannot easily bear pain. He is more vulnerable than women and to hide this fact he has instigated the myth that man is the stronger sex which gains spurious support from the fact that he is, on average, physically bigger.

However, if by "man" you mean mankind, then man is a biological organism with a brain capacity that can sustain thought and language and he believes himself so superior that he can use the world's resources as he wishes without having to bear the consequences.

Rachel Browne


If you are asking 'what is the essence of humanity', then that is far from an easy question to answer. Many philosophers, Sartre among them, have argued that the question is incoherent, and cannot be answered. But one thinker who did his best to find out what it is that makes humans different, 'the science of human specialty' he called it, was Jacob Bronowski. He concluded, among other things, that humans not only have a unique capacity to make comparisons between concepts, but that this process of comparison is something which we are forced to do by the way our minds are structured. I think there is considerable merit in this view, which has echoes inæFreud.

It may well be the case that we are stuck with this process of comparative reasoning. One thing which rather supports the view is the way in which many thinkers, even those with skill in philosophy, often seem to observe only the differences between things, and miss entirely the similarities. For instance, it is not uncommon for people to assert that there are considerable differences between human males and females, when, in fact, the differences are negligible while the similarities between the sexes are all but overwhelming. Even such differences as there are, for instance that males are usually more skilled at spacial and analytic reasoning, while females are better at facial recognition, are only matters of degree and of average. It is just as probable to find a female with greater than average male skill in philosophy as is it is to find a male who is shorter than the average female. Such concentration on difference might even lead a human to think that they were significantly different to non-human animals — on their collective attitude to the world's resources, for instance.

Now that you have two answers, you can use this faculty of comparison on them. But don't forget to look for the similarities, not just the differences!

back

Leonardo asked:

What came first in human species: language or the capacity to classify (categories)?

I would go for the latter. I think animals must have some such capacity. It is the development of the ability to classify conceptually that leads to language, in my view. Now the question becomes a much harder one: What came first in human species: language or the capacity to conceptualise? I don't know the answer to that one.

Tim Sprod

back

Dana asked:

Does karma exist? In other words, is the universe interested in revenge?

If you are a Buddhist, that is taking refuge in the Buddha, Dharma and Sangha, yes karma exists, but if you are not on the Way, no it doesn't, then you are oblivious to it. You have to be on the path to perceive karma. That does not mean karma is relative. But absolute or relative karma does not presuppose something called a "universe" as you suppose. In the Chinese translation of the Heart Sutra, the character for "space" includes the closest possible and the furthest away imaginable. There is no end, no boundary. The Chinese character for time covers the first beginning and the last ending. There is no starting point and no finishing. In short, "the universe" as a thing, let alone a thing "out there", is empty (kong), your idea of it is empty. Your idea of it is itself karmic. The Buddhist idea is that everything that exists in any one moment of time, from the nearest to the furthest is so because karma. It is not a result of karma, it is karma. Karma is thisness. Karma is nowness. The old explanation to westerners of karma in terms of a chronological sequence of cause and effect was helpful once, perhaps, but now is a distraction.

Matthew Del Nevo
http://www.sicetnon.com

back

Blake asked:

So what if I don't believe in God?

Does that make me a sinner or no longer a soldier of "God's army"? ... Coz I don't really give a f***!! ... It's all bullshit anyway!!

You might not believe in God but you do f*** care or you wouldn't be so concerned as to write in this language or even to express yourself at all and ask a question! This sounds like a case of rebellion rather than disbelief! But you needn't worry about it because surely the best God loves and forgives everyone.

Rachel Browne


You sound like someone in the throes of a crisis, and I sympathize. Ask yourself, which god are you rejecting? Kali? Thor? Zeus? Buddha? Yahweh? Allah? Shall I go on...and on? Humanity's history is full of various gods, goddesses, demi-gods, and so on, all of whom have had (and still in many cases have) zealous followers. These followers state categorically that their god is the only true god, and all the rest are false. It must be nice to have the key to the universe. As for myself, I don't claim to have that key, and I doubt that any being, past or present, has. I gather that I have my choice of hells: the Christian hell, the Mohammedan hell, the Norse hell, the Greek hell...they're all waiting for me, since I'm an atheist. How do you think they'll divide my time in the afterlife (which I don't believe in)? But wait...only one of those is the real afterlife/ Shall I flip a coin?

Steven Ravett Brown

back

José asked:

Why do people believe in something that don't know, that is not proved?

Why if we are alive, are we unhappy or forced to be unhappy?

Belief in what we do not know could be driven by our desire to salve feelings of fear or anxiety, examples being faith in God and superstition, although uncertainty may also be involved. Lying behind belief in God could be fear of death, of non-existence, or of losing contact with loved ones. But perhaps faith in an unproven God just stems from the need to believe in the un-worldly or other-worldly because this world is not perceived as good, just or worthwhile.

As for your second question, we are unhappy because we are alive. Problems come with life. Good aspects of being alive are sometimes overridden by the bad and, then, why should we not be unhappy?

Rachel Browne

back

William asked:

Is it worthwhile studying philosophy, since all that philosophers talk about will one day be adequately handled by the particular sciences? It seems, then, that philosophy is just a vast illusion, dealing not with reality but with mere concepts.

There is a large and highly questionable assumption underlying this question: that everything can be reduced to science. This view is called "scientism". I think the assumption is wrong. I would recommend finding the magazine Philosophy Now issue 27 (June/ July 2000) and reading the debate on this topic. You could also look for New Scientist nos. 1832 and 1833 (1 and 8 August 1992) for two articles by Mary Midgley and Peter Atkin on this topic.

And who says concepts are not reality? They are part of my reality!

Tim Sprod


One good reason for studying philosophy is to get some training in critical thinking. I say that because one aspect of learning critical thinking is learning to recognize fallacies like the fallacy of begging the question. You beg the question in an argument when you assume something that needs at least as much proof as your conclusion. I think you have begged the question when you assume that all that philosophers talk about will one day be adequately handled the particular sciences. Should we simply assume that is true, or have you some argument for that premise? For example, how is whether capital punishment is right do be decided by science? Or how is whether there is a God to be decided by science? Just to cite two philosophical questions.

Another thing you might learn from critical thinking is to clarify what you say so that you can determine whether what you say is true. You cannot determine whether what you say is true unless you understand it. You write that philosophy is not about reality, but about "mere" concepts. Even if you are right about that, and it is not clear you are, how "mere" are concepts. It is our concepts that slice up the world so that we can understand and investigate the world. These concepts are our tools for thinking about reality. And, like any tools, we would like them sharp and clean. Otherwise our thinking about reality will be confused. It is one of the most important jobs of philosophy to clarify and sharpen these concepts. So, one thing you might learn by studying philosophy is that concepts are not "mere." And another thing to learn is how to employ these concepts to answer questions not in the province of science.

Kenneth Stern

back

David asked:

Can you give me some reasons for believing that a human being is not a merely purely physical thing?

Three reasons: Consciousness, qualia, intentionality (I am quoting the famous physicalist — more precisely, central state materialist — David Armstrong here, who identifies these as the three major threats to his views).

Consciousness I guess you know. Qualia are the actual experiences of sense perceptions (such as the "redness" of red). Intentionality is the way that our thoughts are 'about' things, whereas brain states don't seem to be about things in the same way at all.

Tim Sprod


One reason (although not one I would agree with) is that in the physical world, objects follow the laws of cause and effect whereas human beings have free will, meaning they can go against their previous experiences. If you hit a golf ball, you know it will go flying, whereas if you hit a person their reaction is completely free — they may hit you back, they may do a Zulu rain dance, you never know.

Problems with this are:

1. You may not agree with that definition of the physical world; quantum indeterminacy does actually show that the physical world does not always follow the laws of cause and effect.

2. Do humans even have free will? Humans might be deterministic "biological machines" with the illusion of free will, hence conforming to the laws of cause and effect.

Peter Clarkson


We can give try to give reasons for believing that another human being is not simply a physical thing. The argument from analogy is an attempt to provide reasons although it not universally accepted. Basically, the argument is that we know from our case that when we are in a conscious state such as being in pain that this is accompanied by particular physical behaviour and when we see this behaviour in others it is associated in our minds with the conscious state of pain. This is criticised on the ground that we can only know that pain behaviour is evidence of a pain sensation if it is evidence for the truth of another's pain sensation, because we can only know something if it is true. Since we can't know if it is true whether or not another is in pain on the basis of an association from our own case, we do not know that behaviour is evidence for the truth of the occurrence of a sensation. You can read about this argument, and the criticisms, at length in The Philosophy of Mind by V C Chappell.

However, the idea of reasons for belief is essentially connected with truth and objective knowledge but mental life is subjective and our knowledge of it is based on inter-subjectivity as opposed by intentionality. An intentional state such as belief is an attitude to clear cut conceptual content and that another person is conscious and has a mental life is far from clear cut! Levinas makes this point in relation to ethics. When we are the intentional state of perceiving another, we perceive a pose and a countenance, but this is not what leads us to suppose another is more than physical. Levinas suggests that we know others are conscious not for reasons but because of our response to them. We do not even have to look at a person's countenance to open the door for him: Our response is an ethical awareness. If we start from a Cartesian point of view of an isolated ego, we may need reasons to believe that others are more than physical, but our awareness of ourselves — from the very beginning — is an awareness of ourselves as one amongst others and we do not respond to or treat others as physical objects.

Rachel Browne

back

Héctor asked:

Is the Universe in which we live really infinite and still growing? Then what is the purpose of human beings on this complex universe?

Well, let's look at it another way; let's turn the question around. Suppose there were a purpose, imposed, let us say, by some superintelligent being(s), and suppose that human beings were part of that purpose. So what? Why should we take their (or "its") purpose as our purpose? Because they say so? Because it's "their" universe?

Let me put it another way. Suppose, again, there were such beings, who have designed the universe. Fine. Now, what gives them their purpose? Another being(s)? And what gives that next set theirs? And why should any of them, including us, pay any attention to any of the others' purposes?

You can see the problems here. There are ethical questions ("why should we be concerned with, and certainly why should we be constrained by, another being's purpose?"); and there is the regress problem ("where does the originator of our purpose get its purpose?"). I do not know, actually, of any philosopher treating these issues at length and systematically (Aquinas barely touches them, but of course he was extremely biased); maybe I'll have to write that book.

What I'm getting at is very simple (which is probably why no one's written a book). Ultimately, somewhere, something is going to have to say, in effect, "The buck stops here; we're going to take responsibility and jump-start purpose." Well, so... go do it.

Steven Ravett Brown

back

Josh asked:

Why do I seem miserable and at odds with my own self when I fail to act virtuously or unselfishly?

As you notice, a moral failure is distinctive because it involves conscience and a concern about what others think of us. Also we want to think well of ourselves. There is not always a conflict between the self and others, but it is often the case and because it makes us feel bad when we fail to act unselfishly, moral thinking must have a very strong hold. The question is how and what sort of hold. Supposedly we recognise moral principles and values because we are both rational and social. Because we are rational we know what is good and understand the expectations put upon our behaviour by others and because we are social we have emotional commitment to others insofar as we care about their feelings.

It might be thought that there is an internal conflict and desire sometimes wins out over recognition of duty, but conscience is both rational and emotional, and also the emotions cannot be detached from the rational self. We always act for a reason, whether it is in our favour or another's but we can't barely act for a reason because we need a motivating force in the form of desire. When we act selfishly we do so because there are reasons and it is what we want. Sometimes this isn't so bad. You can't be virtuous all the time.

As for an explanation of the grip morality has on us, my own tendency is to go for a psychoanalytical explanation in terms of development of conscience by means of the internalisation of moral rules at a very young age. The problem is that I'm not sure how this can account for moral failure in a particular situation. However, the development of the self by means of interactions with others must contribute to the weight others have in our considerations.

Rachel Browne

back

Daniel asked:

I'm a grade eleven student who has a history question on one of the greatest philosophers that ever lived. I'm speaking of the Greek philosopher Socrates. I'm writing a 2000 word essay on how Socrates' death sentence of drinking hemlock on the charges of corrupting the young and not believing in the gods, was the biggest injustice that ancient Greece has ever seen. I've been reading books by Plato like the Republic, the Apology and Crito. I also searched the web and found few sites on my subject. I was wondering if you could help me with my research by sending me information on my subject or tell me where I can find info. I was also wondering if you could help me with a philosophical definition on the word "justice" because I was thinking of putting it in my essay.

Rawls in his book A Theory of Justice (Harvard University Press 1995) defines justice as "fairness". He states, "The intuitive idea is that since everyone's well-being depends upon a scheme of cooperation... the division of advantages should be such as to draw forth the the willing cooperation of everyone taking part in it..." (p. 15). And so forth, for about 583 pages. That's merely the introduction to a modern and very famous treatise on justice, if you want to wade through it.

Now let's take Socrates. What he was doing, very deliberately (as you have seen, having read the Apology), was being as intransigent as possible, without actually being violent. He was very consciously flouting some of the most deeply-held beliefs of his society, i.e., that the gods were the ultimate authority, and that their precepts (as interpreted, of course, by the priests) must be unquestioningly obeyed, by sacrificing his life (which he clearly foresaw would happen, given the laws of Athens) in order to make himself a martyr to the cause of reason (rationality, the dialectic) as the ultimate authority. And in the Apology, you see that he does not defend himself; in fact, he uses his "defense" to attack his accusers, and even more outrageously, all Athens. So in a sense there is no injustice at all in his death; Socrates knew the penalties for teachings against the state; has set it up, planned for it, and expected it from the beginning.

Now one can ask whether his society's viewpoints and laws were just. Well, by their (the Athenians') lights, they satisfied the definition above. They (i.e., most of the people of Athens) would have stoutly maintained that Athens' well-being depended on people obeying the laws of the state and of the gods, and indeed, given the vote against Socrates, the "willing cooperation" of at least the majority (if not of "everyone") was "drawn forth". Were they correct? Well, what if the priesthood had been overturned and the basis of law and education in Athens became the dialectic... what would have happened? Revolution? A just society? There's no way to know, is there. We would like to think that a real utopia would have prevailed... a nice thought, but who could maintain that with any certainty?

So, was Socrates' death unjust? From our point of view? Well, what is the position on the relative weight of some modern god's laws vs. rationality, for most people? I'm afraid that in the States, at least, the majority would side with the Athenians. But we do not (at this particular point in time) put people to death for disagreeing with the religious norm, fortunately. Thus the question becomes one of the degree of punishment. Remember, however, that Socrates could have accepted banishment; his friends were ready and set to get him out... but he refused. That would have been a very hard situation for him, at his age (ancient, for that time), with a wife and son in Athens... but he did have friends in another city who would have sheltered him, and perhaps his family would have joined him. He wanted to die, to make a point.

All in all, then, this becomes a somewhat more complex question, does it not? I actually think, myself, that the laws of Athens were unjust, if only because the penalty for disobeying them was too severe. But that was a much more brutal time than this (they just had less brutal weapons available than we do), and the penalty of death was a common one for illegal acts. In fact, Socrates was treated very well: he wasn't tortured, and his death was deliberately painless... unusual, for those days. I'm just making the point that times and customs differ, and to put your argument through you have not merely to define justice, but show that your definition holds universally... well, Rawls gives it almost 600 pages, and you've got 2000 words. I'd try to step carefully here, maybe take a somewhat more restricted topic... but good luck.

Steven Ravett Brown

back

Susan asked:

What is more important: health or happiness?

Health is a necessary condition for happiness. A condition of something is less important than what it is a condition for. But, that does not mean, that it is not important. So, if you think happiness is important, you will think health is important.

Kenneth Stern


According to Aristotle (in the Nicomachean Ethics), one cannot have happiness (eudaimonia) without good fortune. Good fortune includes having good health. I guess this implies that happiness is more important, in that it goes beyond health.

Tim Sprod

back

Alan asked:

Which is the greater good:
1. To give a homeless child a home
2. To have a suffering pet destroyed

Neither is good without qualification. It wouldn't be good to put a child into an uncaring home, although he might be better off than being on the streets. If the good is that which minimising suffering this is better than destroying a sick pet since while this might be good for the pet I have never met anyone who has had their pet put down and felt good about. In fact, it seems that having a pet put down is more distressing than letting it die naturally.

Good is always for someone. In the case of the child it is clearly good for him. In the case of the pet, you are assuming it is for the pet's good and I agree with you that we consider what is good for pets, although it is held by some that we owe no moral duty to animals. Those who take this attitude see our feeling for pets as sentimental attachment in which case whether or not to have a pet put out of it's misery is not a moral issue, which I think is absurd. If our feelings for pets are not misplaced emotions and we really care for them then it could be a moral issue, although the morality here is not one of duty. We care about our pets; they are part of the family and we believe they feel they belong to us. There is a mutual relationship of devotion, care and dependence which makes the attachment more than mere sentiment. At least this is so for dogs! It is difficult to think in terms of moral relations with canaries and gerbils not because there is no care and dependence but because there is no reciprocality of feeling.

Rachel Browne


The former since human beings are more important than animals, and the suffering of a homeless child is greater than the suffering of an animal.

Kenneth Stern

back

Neil asked:

I'm a high school student who has recently become interested in philosophy and am thinking of pursuing the field in university. Recently, in my political science class, an argument developed over the freedom of state and religion. I found the topic very interesting and was wondering if you knew any classical philosophers that have written works on the subject. It would be greatly appreciated if you could help me pursue this topic.

the obvious first place to start is with John Locke's Second Treatise of Government.

Tim Sprod

back

Laura asked:

What are the main differences between Huxley's and Darwin's theories of evolution?

Interesting that I came across this, because there is a wonderful discussion group that you might be interested in, the Evolutionary Psychology group, and below is a recent posting (I'll leave the sender and recipient off):

Many of the proponents of Evolutionary Psychology are not really Darwinians, because they are closer to Thomas Huxley's position than to Charles Darwin's. I have argued this in various places (such as in my book Darwinian Natural Right), and I have been reminded of this in noticing that Frans de Waal (in his new book, The Ape and the Sushi Master) argues the same point.

Darwin believed that morality could be explained as an expression of a natural moral sense rooted in human biological nature. We know from his early notebooks that Darwin thought that a biological account of morality would have to be a crucial part of his theory. But it was not until 1871 that he finally published his evolutionary theory of morality in The Descent of Man.

I'm not sure what it means to "really be a Darwinian," but I do agree with him that many people in the evolutionary psychology camp (and many evolutionary biologists of all stripes) deny that morality can be explained as an expression of a natural moral sense rooted in human biological nature. This is a shame, because (a) they are most likely wrong, and (b) since people act morally very often, one cannot possibly believe that evolutionary thinking can contribute significantly to explaining human behavior, unless you also believe that morality has a basis in our biology?

I suspect that the Huxley-type evolutionary psychologists focus so much on immoral behavior because they don't believe they can explain moral behavior (vis: evolutionary psychologists are great at explaining rape, murder, neglect by step-parents, and selfish behavior but they can't deal with altruism, love, empathy, sympathy, guilt, shame, revenge-seeking, and the like).

The fact is, we are getting a lot better at explaining pro-social behaviors, and with them, biologically based pro-social morality.

http://groups.yahoo.com/group/evolutionary-psychology

I highly recommend this group if you're interested in this area.

Steven Ravett Brown

back

Shamil asked:

Is the all's God one or a lot, and differs according to men who believe?

If there is a God then he is the same (supposing monotheism is true) But there are several concepts of God, and many conceptions of God. First of all you must not confuse God (if he exists) with the idea or concept of God. They are very different. Just as the concept of a giraffe is different from a giraffe. The giraffe is flesh and blood. The concept of a giraffe is (at least on one view) something in the mind. A concept may exist without what it is a concept of (its object) existing. The concept of a unicorn exists. Unicorns do not exist. We can also distinguish between concepts and conceptions under a particular concept. There is a particular concept of God. This is the concept Jews, Christians, and Moslems all share. It is of a transcendental God, distinct from his creation. But Judaism, Christianity, and Islam, all have different conceptions under this same concept of God. For instance, most Christians identify this God with Jesus Christ. No Jews do. And no Moslems do. But there are other concepts (not conceptions) of God. There is the God of Spinoza for whom God is not transcendental but immanent, and who is identical with Nature, his creation. This concept (remember we are not talking of God, but the concept of God) is very different from the concept shared by Jews, Christians, and Moslems.

Clearly, those who have different concepts and conceptions of God do not worship the same God. Everyone is an atheist relative to all the other religions, so even if you worship a particular concept of God, relative to all the many other religions, you are an atheist. Of course, if you worship some God under a certain concept, you are not an atheist in that sense.

Kenneth Stern

back

Sunil asked:

Hello, how are you ? I hope you are fine!

Can you tell me:

  1. How does thought work?
  2. Is there any truth to sayings "thought creates reality" and "we are what we think"?
  3. Why are different people conscious of different things? how does consciousness work?

Hope you are fine! And I hope the following is the sort of thing you want to know.

It is generally held that the relation between thoughts is a function of the conceptual content. For example, if I think it will rain this afternoon it will be because I know the skies are clouded and I believe these are rain clouds. These thoughts, which are immaterial, will have an effect on my behaviour, such as picking up an umbrella. So a problem of how thought works, is how the immaterial acts upon the physical. We can try to give up the notion of cause, which is physical, or we can subscribe to a theory such as Donald Davidson's which identifies mental events with physical events so that for any thought there is a neural event in the brain, but this seems not to give enough force to the mental as it appears to us. We think it is our decision, not a neural event, which makes us pick up the umbrella.

Another problem is that, from our point of view, we don't seem to go through a process such as judging the nature of the clouds and consciously forming a belief these are rain clouds. We just have a quick look. But if it is simply the concluding thought that is causally efficient, the reasoning would have no causal effect, but my belief about rain clouds must have some bearing. The problem arises because we take relations between thoughts as based on clear cut conceptual relations and we think thought is linguistic. This has led artificial intelligence researchers to the quasi-scientific view that intelligence is unambiguously coded and that if a physical thing can manipulate codes, it can think. Since we believe thinking involves consciousness, this is highly counter-intuitive, because we also strongly believe that only biological organisms with brains are conscious. Furthermore, neuroscience has it that the language areas of the brain are not responsible for thinking so it is wrong to take thinking as a syntactic computational operation, as language itself is. Thought is prior to language, and although we have come to think linguistically it doesn't mean that being able to manipulate codes is sufficient for thought.

The example of the short train of thought above actually makes thinking look partially unconscious: We are only really conscious of our "having a look" and then picking up the umbrella. According to the neuroscientist Edelman in his recent book Consciousness: How matter becomes imagination, the stronger our knowledge becomes, the more it becomes unconscious, so eventually we don't have to wonder about the nature of the clouds. This is clear in the case of typing, for instance. As we learn, we are conscious of our fingers on the keyboards, but when we are proficient we are aware of what we write on the screen. The unconscious, then, has the function of providing us with an ability for intelligent development: We don't have to consciously think about everything and there is an enormous amount of activity going on in the brain at all times. My brain is guiding my fingers and thinking of what I am writing and I am perceptually conscious of what is around me.

It is difficult to say how consciousness works. The artificial intelligence view supposes that information is coded and can be programmed which assumes that as we perceive and learn language the brain learns new codes and signals and there be will be a determinate physical state for any particular thought or sensation. Edelman, on the other hand, says that the brain is active. Instead of being computational, the brain is creative, dynamic, associative and constantly changing. Essentially, a neural process is highly informational. A conscious neural process is associative and selectional, so that a thought is not isolated, and it can move in many directions. There is a "functional cluster" of relevant connections within the brain which changes all the time. It is difficult to summarise Edelman's theory, but the basic idea of a dynamic cluster reflects what thought and perception is like for us. We are never in a simple conscious state, such as "experiencing red". Rather, the mind is active and we perceive an enormous amount of visual information at any one time as well as experiencing bodily feelings and thoughts. Consciousness works because of our brain activities, though why and how our brain activities have the effect of subjective experience, we cannot really say.

It has long been held that "thought creates reality". Kant, for instance, argued that the way in which we perceive and understand what is given to us in experience moulds what we take reality to be. Actually, for Kant, "reality" itself is a concept introduced by our way of understanding, as is cause and effect. Concepts of the understanding, such as reality and causation, are needed for us to make sense of our experience. Our idea of the world as spatial and temporal, on the other hand, is held by Kant not to be imposed by our understanding, but is introduced by the very nature of experience. Our perceptual equipment must determine how reality appears to us. Because of our shared equipment, there are limits to the different sorts of things we can be conscious of. However, you cannot be conscious of things you don't know about. For instance, our grasp of concepts, such as clouds, say differs. You won't be conscious of rain clouds unless you know about rain clouds. Interestingly — for me anyway! — Edelman's claim is that no two informational states at the brain level can be the same. All your experiences of clouds have been different from mine so that when you think of clouds your informational neural process will be different. If you perceive differently, or more than another person, such as in listening to music, you are bringing knowledge and experience to bear.

I hope this is what you want. To find out more about the philosophical problems of thought and consciousness, there are accessible writers such as Colin McGinn (The Subjective View), Daniel Dennett (The Intentional Stance) and John Searle (Intentionality). The mind is generally described as intentional or relational between a subject and content which you might want to find out more about. You could, of course, send a new question, differently phrased!

Rachel Browne

back

Christine asked:

What is the phrase "Ptsosis Kneuton" in reference to? Someone told me if I read Plato "I would know what it was." Can you give me some general information?

Try this site:

http://www.users.globalnet.co.uk/~loxias/search.htm

Steven Ravett Brown

back

Sam asked:

Is technology a double edged sword? Do it's vices outweigh its virtues?

Technology has vices and virtues, but how could we measure whether one outweighs the other?

Heidegger has written a paper called "The Question concerning Technology" (see "Basic Writings" edited by D F Krell) in which he claims that there is nothing bad about technology in itself but only in man's exploitation of the energies of nature. Man is not subservient to technology, which is a means to an end, but is subservient to the challenge, which he seems unable to avoid, to create technological advances. However, we are often not aware of our commitment to technology. The forester who cuts down trees may believe he is working with nature, but really he is subservient to man's desire for paper and ultimately magazines. Heidegger annoyingly speaks in a language which he creates for himself, but I think that what he is saying — and this is a vast simplification — is that when we recognise a danger to mankind there becomes a possibility of seeing how we can be saved. The creativity inherent in technology and art come from the same source and since art, but not science, fosters mankind's ability for reflection on his own nature, the saving power lies in art and, if we can see this, we can avoid a total entrenchment in technology. Because the source of art and technology is the same, we are not in danger as long as we continue to be in touch with the truth and maintain our ability to recognise how our understanding of the way we are can be falsified by ourselves. Our creative urge is not essentially directed to technology and we do not really want to exploit nature for our own ends. The artistic creative urge is a part of us and should not be smothered.

Of course, this is abstract and philosophical and was written before computer technology became so much a part of our lives. I would say that, in fact, we are subservient to technology. As soon as computers break down no-one can do their work! The internet has fostered a desire for immediate action and communication. It has now been realised that we are waiting for two whole days for cheques to clear the bank when this isn't necessary. Really, technology is feeding mankind's worst features of greed and impatience. But then it has it's good side too.

Rachel Browne


It is a double-edged sword since it has some problem: pollution and the like. However, it is clear that the advantages of technology far outweigh its disadvantages. Consider the world 300 years ago, and the world now. It is clear that this is a better world, and this is mostly due to the scientific revolution and the technological revolution. How would you like to have surgery without anesthetic?

Kenneth Stern

back

Peter asked:

This is not really a philosophy question, or not at the level I intend it anyway, but philosophers are good with definitions and meanings so I'll ask it.

If 'society' implies, or depends on, a shared culture (culture in its widest sense), as I suspect we all feel it must, and if 'multicultural' suggests a multiplicity of cultures, as I suspect we all feel it must, then is not 'multicultural society' an oxymoron? Has political correctness, or a well meaning attempt to be inclusive, led us into meaningless doublespeak and another example of the use of grammatical tricks to save us facing deeper issues?

More generally is it reasonable to say that a key task in philosophy, a key element in any definition of philosophy, is the rooting out of conceptual and grammatical conflicts of this kind, if such it be?

A key task for philosophy is rooting out and exposing fallacious arguments: yours is an instructive example.

"If 'society' implies, or depends on, a shared culture (culture in its widest sense), as I suspect we all feel it must...." For there to be such a thing as a society, there must be something that the members of that society have in common, in virtue of which they are able to identify themselves as members of that society. That is all you can reasonably claim to be self-evident, or, as you put it, something 'we all feel' to be necessary. But whether that shared factor is a common nature, or interests, or purpose — or, as you claim, culture — is far from being self-evident. It is a question that bears close examination.

In order to show that the argument you have advanced is fallacious, however, it is not necessary to give a full and complete answer to that question. That would require a book. It is sufficient that we can point out possible ways in which diverse cultural groups could perceive themselves as belonging to, having a stake in, a single all-inclusive society. One of the most important and obvious ways is through inter-faith and inter-cultural dialogue. (An argument of exactly similar form to the one that you give could be used to 'prove' that the very idea of an inter-denominational religious service is a conceptual confusion. That ought to make you pause.) Another way is through political action on a local level. Think of what would be involved, for example, in a community consisting of diverse cultural groups setting up a Neighbourhood Watch scheme to combat burglary and street crime, or getting together to tackle problems with drugs and prostitution.

There are many persons of different ideological persuasions who believe in the possibility of a multi-cultural society as an article of political faith. I would include myself in that large and diverse group. I am not saying that we should not accept the challenge to provide arguments in support of that belief. It is healthy that we should debate the issue, and not allow it to become unthinkingly held political dogma.

Geoffrey Klempner

back

Deshal asked:

Hello, I was just wondering something about Buddhist philosophy and in particular rebirth. How could rebirth take place and yet we see a continuous increase in human population, particularly the sudden surge in the last 100 years or so?

Rebirth is a religious idea, that means it addresses a fundamental philosophical question of existence. Actually the idea of rebirth addresses a nexus different questions. It is a synthetic or integrative idea. The idea of rebirth is certainly not descriptive, as your question seems to suggest, nor is it 'explanatory' in the theoretical sense (that theory purports to explain things, which it only does for a certain kind of mentality). In other words rebirth is not a demographic concept, nor can it be said to 'take place' in the everyday literal sense of these words as you use them, as if about some 'thing'.

I am faced with a difficulty of saying something (explaining?!) rebirth. Rebirth presupposes every other Buddhist concept and can only be properly understood together with them. The abstraction of rebirth into the Western context conceives it in terms of our psychology and religious bent (which is basically monotheistic). In Buddhism, however, (this is going to be very rudimentary) everything is reborn. That does not mean every thing. It means every deed or action will give rise to other deeds and actions elsewhere. In this way everything is reborn. And even the first deed or action is itself a consequence of prior actions, it is a rebirth, including your question and my answer. You are not the source of your question and "I" am not the source of my answer. Scary hey.

Buddhists experience no suffering and no rebirth (they are one and the same) once they are trained in the Path that leads to them.

Matthew Del Nevo
http://www.sicetnon.com

back

MG asked:

"'Be selfish and stay selfish' is my message. Wanting enlightenment is selfishness. Charity is selfishness."
U.G. Krishnamurti.

I have a question on the last part of Krishnamurti's quote and I don't quite understand how charity or altruism can be a form of selfishness. Can you please help me answer that question for me?

I know nothing about Krishnamurti, but I expect he confuses selfishness which is taking more than you are entitled to, and is morally bad, with self-interest which is taking what you are entitled to, and is neutral, but can be a good thing to do. If there are two pieces of cake, one for me and one for my brother, and I take both, I am being selfish. If I take one, I am acting self-interestedly. If I give both to my brother, I am acting altruistically. It would be bad to grab both pieces; but there is nothing wrong with taking just my piece of cake. And, I am certainly not obligated to give my brother both pieces and leave nothing for myself.

The English 18th century philosopher and divine, Joseph Butler, went to great pains to clear up the confusion between selfishness and self-interest, and went on to argue that much of the trouble in the world was caused not by people who acted self-interestedly, but rather by people who did not act in their enlightened own best interests chiefly because they were ignorant of what was in their own self-interest, since he believed that society would be a better place if everyone knew where his own interests lay and acted to promoted them. A similar view was held by Adam Smith who believed that by everyone acting in his own self-interest, the general interests of society would be served.

So, if Krishnamurti changed his advocacy of selfishness to a more sensible advocacy of self-interest, I could agree with him. Of course, if he did, he would no longer sound paradoxically profound and probably lose some of his audience who like that sort of thing.

Kenneth Stern


Krishnamurti is distinguishing here between relative and absolute truths, a convention in his Indian tradition. He is speaking absolutely. Wanting enlightenment and charity and compassion are 'selfish' because the real meaning of the self here is universal, not relative, not psychological. The absolute meaning of self is God. The identity between self and God is found in Western tradition in the writing that man is made in the image and likeness of God. Krishnamurti is trying to awake his hearers to this truth: the identity between human and divine. Not just Christ, he believes, but each one of us, is God incarnate, but very few are awake to this 'fact' and able to realise it. In other words, God's selfishness and charity are the uttermost limits of our own, that is infinite.

Matthew Del Nevo
http://www.sicetnon.com

back

Irene asked:

Why do we have quotas and affirmative action in all businesses except professional sports? Would people watch the games if we did have this? Why isn't it required in professional sports? Why do they get a pass?

The short answer to your question is that it would be intolerable to the supporters of a soccer club, for example, that their team should lose a vital game, because the manager was not allowed to select the eleven players who, in the manager's considered judgement, were most likely to prevail against the opposition. By contrast, there is a certain degree of latitude in recruiting filing clerks, or chefs, or motor mechanics, or philosophy lecturers: the candidate has got to be good enough to do the job to the standards required. After that, other considerations may be relevant, for example, the need to promote good race relations, or religious or sexual tolerance, or the interests of women or the disabled.

That answer is not fully satisfactory. I don't think it is true in sport that winning is everything. There are sound psychological reasons why those who participate in sports at the highest level should believe that winning is everything. Self-belief, the desire to win and the belief that you can win are an essential part of the 'inner game'. It is also fully understandable that supporters and fans should go along with that belief. Of course, you want to see your team on the winners' podium.

However, it is also widely accepted that the desire to win can be taken too far. The sportsman or sportswoman enjoys a life outside of sport, a life where personal relationships are important. This is just one example of how we balance competing values. The optimal amount of practice and working out for peak performance on the playing field may not be optimal if it destroys your marriage.

If we look at the wider social context, then it is reasonable to say that although we want the quality of competition to be the very best that it can be, there are other things we value too. Groups who in the past have for political or social reasons had a hard time making progress in a particular sport deserve a fair chance. No-one would suppose that the inequality could be rectified simply by forcing clubs to recruit team members from those groups. The process has to start with the youngsters who are first attracted to the sport, by providing new opportunities for training and coaching.

Here is one notable example. In 1999, the South African rugby team, the Springboks, competed for the World Cup with a multiracial starting squad for the first time in their history. Coach Nick Mallett was quoted as saying, "There is stunning talent among our players of color. They're the future of South African rugby." Don't be fooled into thinking that this was just a happy chance. It was a result of a concerted political campaign to open up South African rugby to non-whites.

Geoffrey Klempner

back

Edith asked:

What were Socrates' comments (his definition) on "Justice"? Where would I find the appropriate reading material?

Plato sees the self (psyche) and the city (polis) analogously and his idea of justice (dikaiosyne) is part of it, like the oil in the machine which makes it function smoothly. The self has three levels. The sensual, which requires self-control or temperance (sophrosyne). The emotional, which requires fortitude (andria). And he rational, which requires wisdom or prudence (sophia or phronesis). Justice is the fourth cardinal virtue. The three parts of the self are not merely interrelated, but are modes of a unity. "Justice" is what brings about the harmony or good relations of the parts of the self. The City is the same. There are the people, the hoi poloi, the guardians and the philosopher/ rulers. The proper virtue of the people, the producers, is temperance, of the guardians, fortitude and courage, and of the philosopher/ rulers wisdom and prudence. These three classes of society maintain the balance of right relations through the medium of justice. Justice is the quality of human being that relates persons to one another and to themselves, both individually and socially.

The main texts by Plato on Justice are to found in the Republic (351d-352a; Book IV and Book X) and the Laws. In the Laws Plato's ideas are extended and enriched, although not systematically. The reader needs get the whole picture in order the see the part (justice) properly. Books I — V and VI are about human nature in its personal and civil aspects, in term of which one will grasp the elusive form of justice.

Matthew Del Nevo
http://www.sicetnon.com

back

George asked:

I have been trying to write on and understand whether disorder can be a form of order. Sometimes I feel that even though a situation may seem highly chaotic, it may be still be looked on as an organized situation. One analogy I can put to example is the case of noise vs. music. Another one I can put forward is when artists tend to create an organized mess of some sort to add to the balance and composition of a piece.

I am not too sure yet and I'm hoping some of you can help me with this.

The issue of order vs. disorder is a very nasty one, because it is involved not merely with issues of causality, but also with how we perceive of, conceive of, and symbolize pattern. Thus answering this question to any depth must involved discourses not only in physics and math but in psychology of perception, cognitive science, and aesthetics. But your question seems to have to do with art rather than physics, chaos theory, or cognition, so I'll focus on that. One thing I would recommend is that you be very clear on what you mean by "disorder" and "order". For example, to take a musical case, when jazz was first introduced it was considered highly chaotic and disordered; yet today it is seen not merely as very structured but rather old-fashioned in that regard. Has jazz changed? Of course not, so our perception of what is "ordered" in music has changed. Why is that? Surely it is because there is now so much jazz and jazz-derived music we hear, from infancy, that we learn to extract, i.e., to hear, patterns in it easily. However, since our pattern-comprehension is very flexible when we are young and becomes more difficult for us to change as we get older (because, if for no other reason, we must alter much more later than when we start), when a new style of music is introduced we have trouble hearing it coherently. Thus atonal music was a real stretch when it was introduced; now we yawn.

What about "noise vs. music"? Do you mean noise literally, i.e., "white noise": what we hear in the wind, the ocean, running water, and so forth, that is, literal random vibrations in the air? Or do you mean something heard as noise, which as we have seen above can be almost anything that we are not experienced hearing the structure in? Again, when you say that an artist creates an "organized mess" you are not referring to literal noise but to an aural structure that is not easily comprehended, right?

There are two categories of "noise", then... one being literal random vibrations, the other being pattern whose structure we do not comprehend or have trouble comprehending. You can see, then, that the "organized mess" is probably not actually a mess except for someone not familiar with that composer's idiom. You might do a little reading in gestalt psychology: see Koffka or Kohler, for example. Arnheim has a book, Art and Visual Perception, on the visual aspect of structure in art. As for sound, there's a lot on music perception these days: Music and Meaning, edited by Robinson; Music, Art and Metaphysics, by Levinson; Listening and Voice, by Ihde are just a small sample.

Steven Ravett Brown

back

Sylvia asked:

Could you explain Hume's argument that it is beyond human understanding to know the causes of events, even the everyday ones.

You seem to be somewhat confused about Hume's views on causation. It is true that, according to Hume, our beliefs about causes and effects can never be rationally proven, in the sense which leaves no possible room for doubt. Yet in his Treatise of Human Nature Hume describes eight 'Rules to judge of cause and effect' (Book I, Section xv) which we would recognize today as the application of sound scientific method. We can be wrong about causes and effects, because the cause of an event may be unobvious. The apparent cause may not be the real cause. However, this is an error which the proper application of Hume's 'Rules' allows us to rectify.

Where, famously, Hume attacked the notion of causation was in the false metaphysical assumptions which the ordinary or 'vulgar' notion implied. According to Hume, all the evidence we have for any event or connection between events consists in successions of sense data or 'impressions'. On this strict empiricist basis, Hume asks where we derive the idea of a 'necessary connection' between cause and effect; for example, one pool ball hitting another, or a stone hitting a pane of glass and breaking it. The extra factor which is the actual causal link cannot be perceived. All we see is one event, then another event. So what can we possibly mean when we say that the first event caused the second event — for example, the impact of the white ball caused the red ball to go into the corner pocket; the stone caused the glass to shatter?

Hume's answer is profound and revolutionary. The truth of a statement of the form 'A caused B' consists in the truth of a universal generalization. In all places, and at all times, a state of affairs which fits the description of B will follow from a state of affairs which fits the description of A. The impact of a red and white pool ball, or a stone and glass will always produce the same result — provided that the speeds, weights and other surrounding circumstances are the same. We may never be able to duplicate these circumstances exactly. But this is what we mean when we talk of causation. Causation is nothing but 'constant conjunction'.

There are reasons to be worried by this account. Despite the great power of the idea of a universal generalization covering indefinitely many instances, you may feel that something has been lost in the analysis. The particular relation of 'bringing about' or 'making' between one unique event and another has been replaced by a relation between general descriptions. One thing happens; then another. All that we see, according to Hume, is all that is there in reality. The extra causal link that we believe to exist between the two events, consists merely in the fact that events of the first kind are 'constantly conjoined' with events of the second. Our intuitions ought to be affronted by that claim. However, it is no easy matter finding a satisfactory alternative to Hume's analysis.

Geoffrey Klempner

back

Greg asked:

Is it possible to objectively prove religion, faith, and other subjective ideals?

The short answer, Greg, is no. Otherwise it should have been done. The reason it is not possible is that the question is too biased and narrow for the phenomenon it questions. The question raises religion in terms of the subject-object dichotomy. But semitic root religions (Judaism, Christianity, Islam) cannot be understood in psychological terms of subject-object. They can be completely misinterpreted by psychological (or Cartesian) frames of reference, and often are. Western religion is based on neither the subjective nor the objective, but on the Other. The Other is a radical category which transcends (even while altering) the subject-object split in human perception. God is Other, neither subject nor object. The ideas of 'God in Search of Man', 'He Who Comes', 'Revelation' are all enunciated in terms of what is categorically Other.

There is a stronger sense of proof than that posed by your question, which is that religion proves us. However this idea is counter-psychological. But even on the level of the question you have posed, the idea that 'proof' about anything can be 'objective' in the strongest sense, is questionable. Even 2+2=4 doesn't prove the fact — as Wittgenstein pointed out, only that for this fact there needs to exist certain prior concepts of counting and numbers, without which the sum is unprovable. And where do these concepts come from? Such a question is one addressed by religion.

Matthew Del Nevo
http://www.sicetnon.com

back

Bill asked:

How would Epicurus feel about pornography and why?

As far as Epicurus goes, his philosophy has been largely misinterpreted as being one of pursuing sensual pleasure as an end in itself. From my readings, what he actually proposed was much more sophisticated and more in line with the general attitude of the ancient Greeks: that pleasure was the result of living life in the most fulfilling way. Sensual pleasure was only one, and not the most important, source of pleasure. There was also intellectual pleasure, much more important, and various pleasures derived from social interactions. As I understand him, his notion of pleasure related to the general term eudaimonia, referring to a feeling of happiness and fulfillment coming from a life well-led, which thus would include but not be limited to sensual pleasure.

However, your question is interesting to me in that you do not define the term "pornography". That term, in the States, is usually associated with explicit sexual material. From the point of view of the ancient Greeks, as I understand it, since their notion of sexuality was radically different from our own, I suspect that what we commonly regard as pornographic would be in the main quite ordinary to them, and indeed desirable. My own take on the term "pornographic" is rather different from the norm: I take it to mean sexuality which is portrayed as bad or "dirty". In addition, sexuality which causes harm or suffering (and there's a fine line here concerning consensual sex and S&M, etc., which probably cannot be generally adjudicated, but must be judged case-by-case), however it is portrayed, I also regard as pornographic. Sex can be presented with any degree of explicitness without being pornographic, and pornography can have any degree of explicitness. Pornography, for me, implies misapprehension and misuse of sexuality. I think that this is actually more in line with the ideas of the ancient Greeks.

Steven Ravett Brown

back

Jonas asked:

I want to able to say something smart about ways to define HSM, high speed machining. I've got a hunch that there is no "what HSM really is". What would philosophy say about this?

If it is true that there is no such thing as 'what HSM really is', then the reason can only be a matter of fact, not a matter of philosophy. I suspect, however, that you have fallen under the false impression that one would be justified in asserting that there is no such thing as 'what HSM really is' in virtue of the fact that there is no precise definition of HSM. And that is patently false. (By the way, I don't think that the use of the word 'really' here cut's much ice. Just say to yourself, "There is no such thing as HSM", and listen to how that sounds.)

Even in areas of mechanical engineering and technology, many of the terms we utilize have a penumbra of vagueness, a resistance to precise definition. To take a simple example, consider what one means by saying that a component is 'flexible' or 'rigid'. How much does it have to bend to be counted as flexible? where do you draw the line? It would be absurd to draw the conclusion that no component is 'really' flexible or rigid. The terms are understood perfectly well for the purposes in hand. (Consider the exchange: "Is this rod really rigid?" "Try it and see!")

If you are looking for a philosophical background, the early paragraphs of Wittgenstein's Philosophical Investigations are excellent on this.

Suppose that a new drilling machine was designed with a special non-organic lubricant, super-hard drill bit and electronically controlled micro-wobbling movement which enabled twice the drilling speed achievable with older technology. 'HSM' would imply the use of the new process. With extra power, you might get your old drill up to higher speeds for a short while, but you would not be doing HSM, you would just be destroying your drill.

Suppose now, a couple of years down the line, someone comes up with a variation on the set up which doubles the speed again. What was 'high speed' now looks decidedly slow speed compared to the new model. Looking to the future, we might expect further dramatic speed increases. It still would not follow that there was no such thing as HSM. At any given time, you have the choice between the latest technology, and the cheaper, slower alternative. In this case, the given stage of technological development defines what we mean by 'the real thing'.

I have been trying to think of a possible scenario where one would be justified in saying that there was no such thing as HSM, or 'what HSM really is'. Maybe you can think of one. I can't.

Geoffrey Klempner

back

Xochitlquiahuitl asked:

I hope that you can help me...I'm really worried, because my teacher at school asked me to explain about philosophy and religion, and I really don't know how to do it, or how to start to explain it.

The trick here is to not explain it... Let me explain. There is no 'outside' philosophy, some 'other' vantage point which is non-philosophical from where philosophy can be observed neutrally, coolly, accurately and explained. Your teacher needs to be informed of this. Any explanation of philosophy is itself a philosophy. Is your teacher therefore expecting a fully thought-through philosophy from you? Also, the question of the unity of philosophy, that it is one thing, is questionable. There is Jewish philosophy, there is Christian philosophy, there is Anglo-American analytical and empirical philosophy and there is Continental philosophy. The question to our teacher is, therefore, which philosophy? Basically, though, philosophy is about a community of the question as I see it. The question of what, you may ask. Exactly, I reply, the question of what?

As for religion... Behind religion are philosophical questions like, Where do we come from? Where are we headed? How shall we live? Why such evil? Why such disaster and mayhem? How can I live more, fully, absolutely? One can say things about religion in terms of which of these questions it is answering, and in what way. Persons who exemplify it often start religion and it cannot survive unless it continues to be exemplified in the person of the practitioner.

Matthew Del Nevo
http://www.sicetnon.com

back

Mary asked:

What Is The Meaning Of Life?

The other day I got an e-mail out of the blue from a friend in Belfast, the e-mail consisted of a little story that had been floating about the web for a while, I think I will tell it here:

One day a Philosophy professor came to the lecture hall carrying no notes, handouts, overheads or any of the academic paraphernalia, instead what he did have was an empty mayonnaise jar and a bag of rocks.

He placed the jar on the desk for all the students to see.

"Is this jar empty?" he asked.

"Yes" agreed all the students.

Then he poured the rocks (about two inches in diameter) into the jar.

"Is the jar now full?" he asked the class.

"Yes, sure it's full" they all said.

The professor then got a hand full of smaller rocks, pebbles really, and put them in the jar, gave it a little shake about and the pebbles and rocks mixed in the jar.

"What now?" he asked, "is the jar more full?"

"Er, yes" they said, not sure where this was going.

He then got some sand and put it into the jar until the sand filled all the nooks and crannies between the rocks and pebbles, until he could get no more sand into the jar. He then asked the students, "Now will we all agree that then jar is absolutely full, we cannot get any more rocks or sand in the jar?"

"Yes" they all agreed.

"Good, Now to explain, the jar is the world, our life, the rocks are the big things in our life, the important things; love, friends, happiness. The pebbles are the other things in our life, our homes, jobs, cars. And the sand is the little things, what colour top to wear with these shoes, what to call the goldfish. Now what do you think would have happened to the jar if I had put the sand in first? You're right, there would have been no room for the rocks. What does this mean, you ask? Always take care of the big things first, there will always be time later to deal with the little things. Thank you for your time."

Just then a young student got up and approached the professor and his jar. Without a word he opened a can of beer and poured it into the jar.

The moral of the story? No matter how full your life is there is always room for BEER. I doubt this answers the question but I like the story, especially the ending.

Brian Tee
Dept of Philosophy
University of Sheffield

back

Sophi asked:

I've been reading a book about a person who has magnetic energy which he can use to heal people, is that true? Are we surrounded by energy which we can receive and transfer to others, and how can we use it for inner peace, to be more effective in all aspects of life,and what we should do to be more open and ready to receive this energy?

Last question: What is the higher self?

No philosopher would want to say that we could transmit energy to others and so cause inner peace. Philosophers like logically coherent theory and evidence, and even if philosophers were to experience this sort of thing taking place, they would not be able to account for it. However, I don't see why any philosopher should deny this sort of thing is possible. Personally, I see no reason to rule it out. However, the notion of a "higher self" is not a subject matter with which academic philosophers are concerned — academic philosophers haven't yet sorted out the concept of self.

What you seek is a life-style guru. One of the main writers and speakers on this sort of thing is Deepak Chopra who has written several books which should be easily available. Chopra is a qualified medical doctor, which may or may not be a recommendation, and is highly regarded by many people — especially in California.

Rachel Browne


I don't know what book you have been reading, but I do know that there is a long history, going back over a thousand years, of gobbledygook written about this subject. The latest manifestation of this genre is the self-development philosophies of the so-called New Age. If you want to get more of an historical perspective on 'alternative' ideas the books of Frances A. Yates are excellent and libraries often have them. The most occult books on the subject you are interested in, which might be worth reading, are Alice A. Bailey's Esoteric Healing and Treatise on Cosmic Fire. Also, the Treatise on White Magic is pretty mind-boggling. These books are available at your local Theosophical bookshop or library. Bailey didn't write these books herself, of course. She was the medium for a Tibetan Master, who, it is said, watches over humanity from a higher dimension. Bailey's books, along with The Secret Doctrine of Madame Helena Blavatsky, to which they form the sequel, are the source books for most other New Ageisms.

The higher self is not a psychological self but refers to a metaphysical notion of the human person. A simple example is from Christianity. In Christianity conscience belongs to the higher self. It is the part of you that 'knows better'. The concept of the higher self which you refer to is from Neo-Platonism in the West and Indian metaphysics, which believes in it as well. It refers to the objective ground of your personal being qua God, or that which is universally ultimate. This objective ground of your being is not a psychological ground but the ground of the psyche itself. Remember though, before you get dogmatic about higher selves. Buddhism says the higher self is empty and nothing.

Matthew Del Nevo
http://www.sicetnon.com

back

Reuben asked:

Upon which assumption do we form the belief that "Much more is known now than 50 years ago?"

Also, what factors should we use when deciding the "level of advancement" within a given culture/ regional group? What constitutes an "advanced" culture and how will it differ from a "primitive" one?

This is really two interesting questions, which are fraught with "politically correct" emotionality, not to mention a variety of post-modernist deconstructive attacks on science. I'm a real fan of the philosopher of science Paul Kitcher, who was a student of Thomas Kuhn. Kitcher wrote the book, The Advancement of Science, which (in my opinion) Kuhn should have, but did not write. So, first, read Kuhn The Structure of Scientific Revolutions, if you haven't. Then read Kitcher's book, above.

Roughly, some of the ideas in Kitcher's book are that science is not, nor should be, a huge consistent logical system, as the old philosophers of science believed (and on which many attacks are based). It is a set of practices and knowledge which attempt to find, imperfectly, truths about the world. Those truths are tested and retested, refined, revised... and in no real sense are they finished, polished, or necessarily consistent. Yet we do advance, slowly, and this is seen through various measures, such as our success at building machines, at causal explanation, at the scope and consistency of our theories, and so forth. It's not a simple nor error-free picture, and why should it be? So there is no single assumption upon which we believe that "more is known now than 50 years ago", no single nor simple test. There is a huge complex of theories, tests, experiments, ideas... various cognitive and experimental bases for concluding, not assuming, that we are making progress, in the sense of knowing more about the world. This is, actually, part of the problem that people uneducated in science have with it; in order to appreciate that, say, the "theory" of evolution is really much more than a theory, one must have knowledge in a variety of fields and in the experimental basis of science as well. It's just not simple.

See also: Shermer, Why People Believe Weird Things, Radner Science and Unreason, Hines Pseudoscience and the Paranormal.

The "culture" thing....here's the emotional issue. You might take a look at the book: Edgerton, R. B. (1992) Sick Societies: challenging the myth of primitive harmony (1st ed.) New York: The Free Press.

Edgerton really takes the bull by the horns; and just simply claims that societies in which the people are unhappy (they want to leave and do, given the chance), sick, poor (again, they leave when given the chance to make money), malnourished, etc., are societies which aren't working well. That doesn't sound unreasonable to me, but it's not very politically correct to say that, these days. You can claim that they are being exploited, etc., and that is certainly true in many cases, but the lifespan of humans, as we go back in history, decreases as a function of the time we go back, while disease and general misery increase, as far as we can tell. So it's not just the exploitation of societies in modern times that correlates with negative (and I would say that short life-span and disease, to name just two, are indeed negative) values and suffering. I'm afraid that I'm rather cynical about the "noble primitive" idea. I myself would have died of acute appendicitis at the age of 12 if it were not for modern medicine. That seems a pretty good criterion for progress to me.

So an "advanced" culture then has at least potentially a greater ability to ameliorate suffering, disease, starvation, than a primitive one. Whether and for whom such a culture actually does that is another question, isn't it? I would certainly say that the more the better....and many "advanced" cultures are certainly wanting in that respect.

Steven Ravett Brown


These are good questions. They are often dismissed as too simple to be worthy of much real attention in politics and social planning. However it is the failure to ask, never mind answer, these questions that allows society to wander so aimlessly into its future. The usual answers concern technological cleverness and economic growth, neither of which relate in any straightforward way to social improvement or genuine understanding of the world.

The real answer is that there is no agreement on these issues in general terms. It is perfectly reasonable to say that we know more 'facts' than we did 50 years ago. It is not so easy to say that we have a better understanding of the world. It could be argued either way.

Most anthropologists would say that 'advanced' and 'primitive' are not terms that we should used in relation to human society without great care. However almost everyone does use these terms, and think them, as value judgements without any care at all.

It may be unhelpful to you but I would say that until some real work is done on these issues, and some consensus is reached, your own answer will be as good as anyone else's.

Peter Jones


I am not sure about an assumption. We can certainly give examples of what we know now that we did not know 50 years ago. Here is one. The existence of DNA and its structure. There have been significant advances in the treatment of cancer such as 3D radiation, which was not even thought about 50 years ago. And these examples can be multiplied. I suppose you can be asked the question, "On what assumption do you believe that we do not know now more than we did 50 years ago?"

Technically, the difference between a primitive and a non-primitive culture is that primitive cultures do not possess writing. Of course, more loosely, advanced cultures possess science, but primitive cultures do not.

Kenneth Stern

back

Jess asked:

I am writing a paper about Kant, but specifically on how Kant would view free speech. I have been stuck on this question for about a week now and have not come up with a reasonable answer.

Underlying all of Kant's practical philosophy (ethics, politics, history) is his commitment to the paramount value of the freedom of human beings as 'ends in themselves'. Any action that is not permitted by The Categorical Imperative will be immoral because ultimately it will corrupt this freedom (of oneself and of others). Any laws that a state regulates are ones that must necessarily protect and promote this freedom.

Starting out from this position Kant distinguished innate and acquired rights. We acquire rights when situations occur when we have to enter into interpersonal agreements with others, such as property agreements and work contracts. Such circumstances necessarily require that we impinge on the freedom of others, therefore laws and regulations are needed to protect us from undue or excessive impingement's. In other words, laws are an a priori rational requirement of public interaction. (in a sense Kant has a theory of the social contract). Where no such interaction, co-operation, or common agreement is required there is no need for regulation.

Holding certain beliefs and opinions does not require any interpersonal activity and expressing those beliefs does not itself necessarily impinge on the freedom of others. Because these other people are themselves free either to ignore, disbelieve, or form there own opinions, what is said cannot affect their own freedom. Therefore, for Kant there is no need for regulation controlling free speech. Free speech is an innate right that is derived from a person's fundamental value as a free being.

Of course, there will be cases where expressing the belief will affect the freedom of others, these cases will. involve using other people as means to some end that they themselves have not agreed to, in which case Kant would agree to the regulation of expression. But it is important to make clear that Kant is not regulating against the belief that is expressed (people can still believe what they want) but the Act of expression that will result in the contamination of others' freedom.

Brian Tee
Dept of Philosophy
University of Sheffield

back

Jim asked:

There is no question that philosophy is relevant to our personal and civic lives, but the reality is this: The majority of people are cogs in the economic machines of the nations in which they live. Most people spend most of their time "making a living". So, if you are interested in promoting the study of philosophy — not only inside of, but also outside of academia — the following question must surely be of interest to you:

What specific jobs justify the time and money invested in an undergraduate degree in philosophy, or a Masters or Doctorate in philosophy?

By "justify" I mean, ideally, that the tools of a philosopher — in logic and language, in the history of philosophy and religion, in epistemology and ontology — would be indispensable tools in the work a person does. A minimal justification for studying philosophy would be, in my opinion, that training in the use of language and logic would be an indispensable tool in the work a person does.

The point of gaining a degree in philosophy is that it trains you to think rather than supplying you with a body of knowledge (even if you don't regard me as a person who thinks well, at least I know it is a relevant question). After studying philosophy for five years I took two postgraduate diplomas in law which involved cramming a lot of information. This required memorising rather than understanding. It just wasn't difficult like philosophy is and I was quite surprised.

Actually, I think the law is one career which should make the study of philosophy compulsory. The law requires an ability to apply logic, make conceptual deductions and produce persuasive arguments — this is what we are supposed to learn by studying philosophy — but law school doesn't teach it. Philosophical skills are relevant to law because when new case law is created it must be held to be truly inferred from prior case law; when statutes are interpreted analytical skills are needed; barristers need to be skilled in argument and persuasion.

I don't think we need to justify the study of philosophy. Rather we need to justify the fact that we have scientists and lawyers and other professionals who haven't studied philosophy. In a recent answer I relayed Feyerabend's views on experts and I think he would agree with this. Studying philosophy doesn't simply produce better thinkers, it puts people in a position to assess objectively what they are doing.

Rachel Browne

back

Alan asked:

Which famous philosopher promoted the idea that one should 'never draw conclusions'? I assume this proposes that it is more intelligent or enlightened to sustain an open mind rather than come to any definitive conclusion when reasoning, contemplating, reflecting, analyzing, etc.

I know of no philosopher, famous or not, who enjoined us never to draw conclusions. In order to give an argument for any view we have to "draw conclusions," otherwise there is no argument.

Perhaps you mean, that we should never jump to conclusions. That is, we should never draw conclusions from inadequate evidence. Doing that is certainly a mistake, and is, for instance, why we stereotype people by generalizing from just a few instances to an entire population.

Your qualification "definitive conclusion" makes me think that you have in mind not coming to a conclusion, but coming to a conclusion which you think is absolutely certain. In a particular sense of "certain", that is right: nothing is absolutely certain.

Kenneth Stern

back

Jeyashankar asked:

My question refers to Rene Descartes Meditations on First Philosophy:

When he asked the question, what can I know with certainty? why didn't he consider that if I stop breathing I will die?

This not only evidences one's existence but also eliminates the possibility of doubts, which exist in the mind, being a tool of the "devil".

Descartes probably did consider the possibility that if he stopped breathing he would die, but felt it was unnecessary to mention it in his works because it is overridden by a stronger kind of doubt, How does he know that if he stops breathing he will die?

It cannot be from personal experience, otherwise he would be dead and not able to ask the question, but from other people, what they say about biology, or perhaps from watching other people die. But why believe what they say, perhaps they are playing a trick on Descartes, anyway what should he even believe that these other people exist? Because he has reasons for doubting the veracity of the senses he cannot be certain that the senses are not tricking him here.

In fact how can he be certain that he IS breathing? how can he be certain that he even has a body to breathe? How can he be certain there is a link between his not breathing and his death? These are questions that Descartes did consider and found that he had no positive answer, he simply does not Know (in his sense of the word) any of these things. At least, that is not until he has finished his meditations, where he established, to his satisfaction, the existence of a God who would not allow Descartes to be deceived in such ways.

Brian Tee
Dept of Philosophy
University of Sheffield


In the 'First Meditation' Descartes' main point is that we can know nothing for certain on the basis of sense experience. But my belief that if I stop breathing, I will die is based on sense experience. Therefore, (according to Descartes) we could not know it for certain.

By the way, I do not think that Descartes thought that "doubts...are a tool of the devil." He used the hypothesis of the evil demon to raise these doubts, and he also used these doubts to argue that he could achieve certainty through the method of doubt. It was by the method of doubt that Descartes thought he had proved that God existed. So, he could hardly have thought that doubt was a "tool of the devil"

Kenneth Stern


Descartes' project was epistemological. He was seeking a foundation for knowledge. If he considered the proposition that if he stopped breathing he would die, he would only achieve the knowledge that he was essentially connected to his body. This would have been no good to him since one motivation for his project was to prove the existence of God and, as a Catholic, the soul. The breathing proposition doesn't rule out the existence of God and the soul but it is not a good premiss for Descartes' purposes.

Even if we consider the proposition that if we stop breathing we will die, there could still be an evil demon who causes this to be the case.

You can't beat the evil demon which is why Descartes is still studied.

Rachel Browne

back

Mario asked:

We are a group of I.T.E.S.M. students and we would like to get some information about the concept on "Fundamental Option", definition, application and links about the theme. We appreciate your help.

"Fundamental option", "initial choice", "fundamental project" are all various terms attached to the idea, found in Sartre of an original choice.

In Sartre's existential philosophy people are to be understood in terms of their freedom. How I as a person acknowledge engage with and face up to this freedom will explain the person that I am. Just how I will engage with this freedom is explained by Sartre in terms of the original choice. The original choice is a choice about what kind of person I will be, of what attitude I will adopt to the world, of what kind of stance I will take toward situations, other people and even myself, what projects I will pursue, what values and motives I will have. In other words, my whole world view will be encapsulated in this initial choice. An important point that Sartre makes is that while all my decisions are to be understood in terms of this fundamental option, the fundamental choice itself is made without any basis, any determination, any justification since any such support or justification is only valid, only available within a world view that one chooses. In other words such a fundamental choice is an absurd choice, there is no reason to it.

Why does Sartre think that an original choice is important? Well as I said for Sartre a person is understood in terms of her freedom, but Sartre also thinks that a person is responsible for everything she chooses, for everything that happens to her. At first sounding this sounds just crazy. Is the person being tortured responsible for the pain he is in? Is the disabled person responsible for being without legs? We usually think that people are not responsible for such situations, but Sartre thinks that they are responsible. And they are responsible in virtue of their initial choice. Here is what Sartre says: "Even this disability from which I suffer I have assumed by the very fact that I live; I surpass it toward my own projects, I make of it the necessary obstacle for my being and I cannot be crippled without choosing myself as crippled. This means that I choose the way I constitute my disability (as 'unbearable', 'humiliating, 'to be hidden', 'to be revealed to all')."

The point of this should be clear: the disability is itself nothing. It's significance is only apparent depending on what attitude I take to it, is it an obstacle in my way or an opportunity? This example should highlight the important consequences of the fundamental choice in our lives, but it is difficult to make sense of the fundamental choice itself. When exactly does it occur? don't we grow into attitudes, given our experiences and up bringing rather than consciously choose them? Do 'I' exist before a choice is made? The initial choice is supposed to influence what type of person I will be so then how do I exist before I have chosen what to be? It seems I would exist as some purely intellectual being, but this is entirely at odds with Sartre's account of a person being engaged with the world. Perhaps the most influential criticism of Sartre comes from his friend Merleau-Ponty. Merleau-Ponty argues that any choice presupposes an understanding of what is involved in the choice. In the recent General election in the UK, if I had decided to vote Labour or Conservative I must have had knowledge of what policies the parties stand for, what the differences between them are etc. In making an initial choice I must understand what options are open to me, what opportunities the world presents, my historic situation act. If this is right then there can be no initial choice in Sartre's sense, because for Sartre this is supposed to create understanding to bring it about that there is a world for me.

Nevertheless, the point that we can recognise that we have some control over our approach to life in the face of the seeming arbitrariness of the world can still strike us as a powerful idea, even if Sartre took this to an untenable length.

Brian Tee
Dept of Philosophy
University of Sheffield

back

Kelly asked:

For a college paper I am supposed to find out on the internet what Aristotle would have thought about these subjects:

  1. The death penalty
  2. Abortion
  3. Minorities
  4. Assisted Suicide
  1. Aristotle wouldn't have approved of the death penalty. He said "Punishment is inflicted for the sake of the person punished", so it should be corrective.

  2. Aristotle was against killing. He did think that pleasure and the good were things we should strive for. If a foetus was found to be so severely handicapped that it would develop into a person with no hope of pleasure he would still not be for abortion, but would hope that this person would bear its ills with as much nobility as possible.

  3. What is a minority? "Minorities", I suppose, are ethnic or sexual, but surely criminals and vegetarians should be included. You should object to this part of the question. Aristotle would think that all minority groups should be treated as fellow citizens — consistently with this we don't kill murderers but show them a way to redemption.

  4. Aristotle thought that pain is something to be avoided and so may have approved of assisted suicide — assuming the suicidal person was in pain — but it is unlikely. In Nicomachean Ethics Aristotle says that even though suicide is against the law a person who kills himself is acting contrary to the state but not acting unjustly towards himself. However, he thought that killing another being is wrong in general and would probably think the motive of kindness would not justify assisted suicide.

I hope that this is of some help. I expect you are doing other internet searches and if you come up with different results the work you have to do for your paper will become more interesting.

Rachel Browne

back

Rany asked:

Was Spinoza an atheist? If he was, in what sense (Did he not believe in God at all and was hiding it, or was he an atheist from the Jewish/Christian/ Muslim perspective only)?

Although the German poet, Lessing, called Spinoza a man "drowned in God," Spinoza was excommunicated from the Jewish Community in Amsterdam, Holland, for atheism. Although Spinoza made what he called "God" the focus of his philosophy, it is clear that his concept of God has nothing to do with any traditional concept of God. For Spinoza, "God" is identical with the world (is immanent rather than transcendent) and is impersonal. Spinoza's "God" is God in name only, and to call philosophical substance "God" as Spinoza did, is only to advance a "persuasive definition" for atheism.

Kenneth Stern

back

Tom asked:

Is there an existentialist reply to determinism? If so, what is it?

Existentialism does not so much argue against determinism as presuppose freedom, mainly because Existentialism deals with the uniquely human world, the importance of human lives in a human world, this is a different world from the one described by science.

For the existentialist, freedom is a necessary feature of human life, that is we are people, human beings because we are free. Without our freedom we would be no different from trees and dogs.

You may think that this, in a sense, is just side stepping the issue: the natural world (dogs and trees, etc) is determined, we are part of the natural world, so what distinguishes us from dogs and trees? How do we know that we are not caused to think that we are free?

When faced with these questions the existentialist has to emphasise a part of his philosophy that we have so far ignored. Existentialism grew out of a philosophy of phenomenology, a method that analyses the appearance of phenomenon, that way objects, events seem to us. Now much of existentialism retains this approach. Sartre for instance subtitled his great work Being and Nothingness "An essay in phenomenological ontology".

Sartre argued that what exists is the world (containing physical objects) and consciousness, constituted by its ability to negate or distance itself from everything else. For phenomenology as well as existentialism, consciousness just like a table can be analyzed as a phenomenal object. We can ask questions about what it is like to be conscious. Now whether or not consciousness is the outcome of brain processes it does not appear to me that way. The phenomenology of consciousness is substantially different from any scientific account (note that this means that the scientific account does not undermine or take explanatory precedence over the phenomenology).

Similarly with freedom. Getting back to the troubling question: How doe we know that we are not caused to think that we are free when in actual fact we are not? Again the existentialist can appeal to the phenomenology of freedom: whether or not my thinking is determined it does not strike me this way, rather it appears that I can think whatever I want. Much of what the existentialists are concerned with is the fact of human life that we approach the world as people having to make choices about what to do. We have this attitude whether we are determined or not. For the existentialist it does not matter about the metaphysical arguments about determinism. What matters is that we live as if we were free because this is how it seems to us regardless of any scientific account about cause and effect.

Brian Tee
Dept of Philosophy
University of Sheffield

back

Geoffrey asked:

Could you explain Richard Rorty's views on truth?

Richard Rorty believes there is no special key to catch the true reading from a text, like someone might have a secret number to open a security box. But he didn't say that we can't have in our houses a security box filled with several secret numbers. In my opinion — a opinion that I think as respecting Rorty — there are good keys and bad keys.

Let me first deal with bad reading. It appears in the hands of a kind of neomarxist or a hard analytic philosopher, or a foolish professor of Philosophy. A man or woman thinking the following: "if there are no right key to get the truth on a text then Rorty is saying that the truth is something we will not to know and then Rorty is a skeptical philosopher — this is my conclusion!" Or: "if Rorty is saying that there is not the right key to know the truth in a text then he doesn't believe in the true sentences or utterances, and this is a contradiction, a bad formulation — I needn't believe him because what he is telling doesn't make sense." Newspapers and books and universities keep a lot of people thinking in this way.

Let me deal with better readings. It is natural position of a smart professor of Philosophy. He knows: "I didn't study Rorty but what he is telling is (in the) the central debate in Philosophy nowadays: the dispute of theories of truth." He is right because of that: Rorty wants to overcome the polemic realism versus anti-realism or the conversation about the truth. But he is a philosopher and he needs to be understood among his peers. So, Rorty is seeking for a new paradigm but he is avoiding a Nietzschean hole. If he starts to talk like Nietzsche, for example "there are only interpretations", he doesn't get to make a step; nowadays he needs to choose the technical and correct language of Philosophy from the departments of Philosophy, and he has to keep waiting until someone without prejudices catches what he is saying. Shortly, Rorty talks about the debate between two theories of truth: the strong position coming from the theory of correspondence and the several positions coming from the contemporary and minimalist theories.

This is a new point, but this is not a completely unpublished point in other times. Bertrand Russell attacked William James using a theory of correspondence of truth against the pragmatist theory of truth. It was a good attack. Horkheimer did a similar thing but his text now appears boring and the words incorrect. Since that time the pragmatist walked in a inferior position. But after the Second World War Quine animated the issue. Texts written previously by Ramsey and Wittgenstein appeared. Donald Davidson became a famous philosopher. And Paul Horwith wrote a lot of text on the minimalist position. Rorty's neopragmatism in this field, as Rorty himself says, is a position swinging between an old pragmatist opinion and a minimalist opinion about truth. For Rorty sees a bridge between the old James' opinion and the possibilities of the post linguistic-turn approach made by minimalist philosophers.

I wrote several books in Portuguese where I tried to show this: O que você precisa saber em Filosofia da Educação, published by DP&A (http://www.dpa@dpa.com.br) and Richard Rorty: a filosofia do Novo Mundo em busca de mundo novos published by Vozes (http://www.vozes.com.br). Here I don't have space to explain this. So, I will just put our schema:

What are the theories of truth telling us?

  • Correspondence Theory: x is true iff x corresponds to fact.

  • Coherence Theory: x is true if, and only if x is a member of a fitted and harmonic set of sentences.

  • Pragmatist Theory: x is true if, and only if x is useful for us to believe.

  • A kind of, say, Piercean Theory: x is true if, and only if x is probable or x can pass through verification under ideal conditions.

All theories above are traditional theories. They work (or not) regarding a kind of ontological point. Of course, it is obvious in the correspondence theory, but the other cases are not completely unattached with such a point of view. That is, all theories above will go a following way: they put linguistic elements in a side and non linguistic elements on other side. And the problem is: all theories meet a gulf between both sides. The solutions for this gulf, that is a bridge or a similar thing is a trouble and they are a part of the History of Philosophy. And this History and its gulf brought the idea that there was something mysterious around the truth: what is the nature of the Truth?

Frank Ramsey said a thing before thirty years and it helped nowadays: minimalist theories. We won't talk about Ramsey — if he was or not an adept of correspondence theory. What is important is: he shows a kind of ladder (I took this idea coming from Simon Blackburn). We can say: "It is true that p", in the beginning; "It is a fact that it is true that p", in the first step; "It is really a fact that it is true that p", in the next step; and we can say after several sentences: "it corresponds to the eternal mathematical (or normative) order governing the universe that p". Ramsey taught us a thing: we use our words and sentences for a success in the communication process. It is a case of good performance and bad performance. Then the "truth" and "true" are in this to help us to have a good performance in a linguistic game. Because you and I can say "It is true that eagles can fly" and such a utterance is telling us just "eagles fly". The "true" here is a good tool for our performance in our communication. But we can give it up.

When the minimalism appeared with that card it didn't solve the gulf, but it opened a door: we can think "truth" or "true" are not mysterious things. They are linked to linguistic performance in communication process. By the way it would be the same as William James said when he put "the truth is what is good of us to believe". It is a good word, useful in a game when we talk about all things of the world. Ramsey could be read like a pragmatist philosopher. And it is the point that we need to see Rorty as a stripper. But if we take another way... oh! I believe that we will not get a naked Rorty. And we won't understand Rorty.

Paulo Ghiraldelli Jr.
Universidade Estadual Paulista
Marília City
São Paulo, Brazil
http://www.filosofia.pro.br

back

Joseph asked:

I have a two-part question:

  1. A good many years ago, I came to the conclusion that the universe is a purely natural system which creates intelligent life forms to play a functional role in its overall development, which is to say that our presence here will lead to a state of the universe or to an event of definitional significance in the scheme of things that could not occur without our being a part of the system..

  2. In terms of a specific functional role, I have come to the conclusion that we will most likely turn out to be a required, purely natural link in the reproductive process of the universe.

— I would like to know what philosophical precedents there are for such a positions.

What you have given is a pretty accurate account of the theory held by the twentieth century philosopher Samuel Alexander, in his monumental two-volume work, Space, Time and Deity (Macmillan 1920), based on his Gifford Lectures at the University of Edinburgh 1916—1918. Alexander is a deep and original thinker, and also a fine writer. I recommend that work to anyone who is serious about metaphysics. Sadly, Alexander's work came to be eclipsed by A.N. Whitehead's Process and Reality published only a few years later. Whatever chance there may have been for Alexander to influence the course of British philosophy was snuffed out by A.J. Ayer's Language, Truth and Logic and the rise of logical positivism.

I was lucky. While I was a graduate student, I came across an approving reference to Alexander in a book by Leslie Armour Logic and Reality (Van Gorcum 1972). Then not long afterwards I found the two volume Macmillan 2nd edition, dating from 1927, in an Oxford second hand shop. By that time, the work had been out of print for nearly two decades. — I have to admit it is a long time since I last read Alexander, though writing this answer has certainly whetted my appetite for giving the text a fresh look!

Alexander set the precedent, later followed by Whitehead, for a realist as opposed to an idealist theory of the nature of reality. His teacher, the idealist F.H. Bradley, author of Appearance and Reality (1893) was a strong influence. Alexander's achievement was to see that something like Bradley's Absolute could be reconstructed on realist lines: his solution was ingenious and original, as we shall see in a moment.

Incidentally, Alexander was one of the few philosophers of his day to make a serious attempt to understand Einstein's Theory of Relativity. His anti-representationalist account account of perception within a realist framework was also way ahead of its time — anticipating the work of Austin and Wittgenstein. It is fair to say though, that by the 60's, when the last copies of Space, Time and Deity left the press, metaphysical treatises that attempted to solve the great questions of Existence were out of fashion.

Now for Alexander's theory. Two key concepts for Alexander are those of an 'emergent quality' and the idea of 'evolution'. The task, as in any metaphysical theory, is to account for every aspect of existing reality on the simplest and most economical basis. Alexander's idea was to start with Space and Time each of which he regarded as inconceivable without the other. But how do we get from there?

Space-Time, the universe in its primordial form, is the stuff out of which all existents are made. It is Space-time with the characters which we have found it to reveal to experience. But it has no 'quality' save that of being spatio-temporal or motion. All the wealth of qualities which makes things precious to us belongs to existence which grow within it...It is greater than all existent finites or infinites because it is their parent...

Op. cit. Vol. I, p. 342

Out of pure space-time emerge through a process Alexander describes simply as 'motion', the stuff and matter that make up our material world. Then, at a later point in the evolution of the universe, Mind, or conscious awareness emerges from material structures that have achieved a sufficient degree of complexity. The final stage of this process has not yet been reached, however. At some point in the future history of Mind there will emerge the thing Alexander calls 'Deity', thus revealing the ultimate purpose and destiny of the universe.

Fully consistently with his revisionist theology, Alexander interprets the activity of prayer and worship as aimed at the future, rather than at something that exists in the present. Any metaphysical theory of a Deity, Alexander argues, must satisfy the requirement that the Deity so revealed should be worthy of worship. Alexander sincerely believed that his theory of the Deity who is yet to evolve satisfied that requirement.

I personally have my doubts. One can trace a direct line to another Jewish philosopher, Spinoza, who had a vision of a Deity as the one Substance in which all finite things inhere. Like Spinoza, Alexander offers a radical reinterpretation of the religious attitude which some may see as a defence of religion, while others may legitimately feel that what is essential to their faith has been thrown out in the process.

Geoffrey Klempner

back

Kevin asked:

Yesterday, I was introduced by my brother to the ideas of the Church of Scientology. I looked for the online websites, and I found first the anti- sites, who have some very disturbing things to say about them. I am not sure what to think about it. I was wondering if you could provide me with a summary of what is exactly, your opinion of its practices, and your professional opinion on the philosophical concepts on which it is based.

First, the story I heard was that Scientology was started by L. Ron Hubbard, a hack science fiction writer (known and deified within Scientology as "Elron"), on a bet, at a party, that he could fool people and start a cult. He was successful.

As you can see from the above, I believe (and have seen a lot of evidence) that this is a cult, that it is based on deceit, manipulation, and junk science. In addition, members of this cult have resorted to physical violence and intimidation against people who question it (I'm actually risking this, to some extent — but not much, at this point, because of publicity of past incidents — by responding in this way to you).

As far as I can tell (and since it's basically junk, I haven't spent much time on this), it is based on a kind of Westernization of Zen, where the goal, instead of "enlightenment" is to be "clear". This means something like being self-aware, knowing what your goals, motivations, etc., are, getting "unblocked"... all fine-sounding stuff, right? The problem is that first, the way to do this is very specific, cut and dried, and you cannot question it. Second, you must devote your life to it, basically. It's done through the use of a crude type of lie-detector, which (supposedly) indicates when you answer questions unspontaneously... you're inhibited, so you're "blocking". Ugh, that kind of thing is so ridiculously unreliable... not to mention that the construction of the questions is amazingly easy to bias toward whatever the investigator wants... junk, as I say.

There have been NO, I repeat, NO unbiased (i.e., done by people not members of this cult) studies of this that I know of that have said anything positive about it. Licensed and qualified therapists do not practice it. No legitimate schools teach it. STAY AWAY. Get your brother out of it.

If you want enlightenment, find some way that doesn't require your unquestioning devotion and entire resources. There are LOTS of legitimate teachers and schools of Zen, Yoga, etc., around.

Steven Ravett Brown

back

Berk asked:

I am working on this subject as a project and will be privileged to hear your interpretations on the following questions:

  1. The light coming from an object is transformed into electrical signals by the cells in the eye and then transmitted to the center of vision in the brain. And the electrical signals there are turned into an image. For example, you actually see this message in your brain. Then who is the one that sees and perceives the image of this message in the brain? How do you define the consciousness that can see this image in the brain without the need of an eye?

  2. Brain is a piece of flesh composed of lipids, proteins and other various molecules. Could the consciousness that sees this image be this piece of flesh? Or could the brain cells make up a consciousness that sees these electrical signals as a sea view or an e-mail message?

  3. No light penetrates the skull, which means the brain is entirely in darkness. Then how does such an illuminated, clear image is formed in this pitch-dark place? For instance how are the rays of the sun seen over the unlit brain cortex?

  4. Also no sound enters the brain. This means there is deep muteness where the brain exists. However, people listen to all different sounds inside the brain. The sound waves are turned into electrical signals inside of the ear and then transmitted to the center of hearing. And the consciousness inside the brain listens to these for instance as a melody. Then who is it that listens to the loud music aired from powerful loudspeakers and how?

  5. The image is formed inside a miniature spot in the brain. Then how is the three-dimensional image with depth is formed on this diminutive screen? For instance when I look at the horizon or the sky, how is such an image with vast depth formed at this tiny spot of center of vision just as identical to its original with the same depth and sense of distance? What is it that gives me the feeling of distance and space?

  6. When a person sees a glass of water, in fact he does not see its real form but only a copy of it in his brain. The coldness that he feels when touching the glass surface is not the real coldness of the glass but only a copy of it. This means nobody is ever able to feel that he touches the real glass. Since it is not his fingers that feel the sense of touch, but merely the sense of touch in the brain. Should we not in this case conclude that people are never able to reach the reality of objects and can never touch the reality of a glass? But not every person knows this fact. Everyone thinks they touch and see real objects. Is it not strange that people are not aware of this and they never think about this?

  7. Nothing changes when a person is hit by a bus or comes across a lion. Since, just like the image of the bus, the sense of collision or the fear while running away from a lion, all form in the brain. When I see a bus, I see it at the center of vision inside my brain. If I go and hold the door of the bus, I feel the coldness of the metal inside my brain. Then I cannot discriminate from this fact what happens when I feel pain if a bus hits or a lion bites. Then is it not very illogical when people say, "It shows I am in contact with the bus or the lion because I feel pain when the bus hits or the lion bites"?

  8. We live the entire period of our lives inside our brains. In a similar way, we also have dreams in our brain. For instance when we touch a piece of ice in our dream, we feel it is wet and cold. Or, when we smell a rose, we get the wonderful scent of it. We again sense the feelings of fear, pain, anxiety and panic in a similar fashion. Then are the dreams and the real life the same in this sense?

  9. The person's own body is also included in the images a person sees. So, a person only sees the copy of his own body. This means every person all through his life lives in the cave in his skull where he never knows what is outside, including his own body and other objects. Now think over this happening once more: Right now are you inside of the room you are present or is the room inside of you? Isn't the second alternative the right answer scientifically?

  10. Let us imagine 5 different people who look at a garden of roses. Since every one of these people see the rose garden in his own brain, then aren't there 5 different rose gardens in the brains of every one of the 5 people? Is the color red that each one sees the same with the other's perception of red? Would there be any possibility to compare these?

  11. We say that the original objects we see the copies of in our brain exist outside, but what if nothing exists outside? Because we never have the ability to test this or observe this. Then is it not dubious that the original objects are outside? At least there is a 50-50 percent possibility. Then how can we be sure that the original objects are outside? If there is no original object outside, then what is the entity that makes the images and the senses in our brain?

  12. If we are living an illusion that has the possibility of not having any reality outside, then we may be existing in a very different place. For instance is it not possible for the entire humanity to exist right over a piece of crystal? Or is it not possible that the complete history of humanity has been experienced in a place not bigger than the head of a pin? Would there be anything to stop us thinking in such a broad extent?

  13. Some people are incredibly afraid when these topics are discussed? What do you think the reason for this may be?

You don't see a message in your brain, you see what is in the world. A perceptual state is a relational state, the relata being the brain and what is perceived. It is necessary for the brain to be an informational state relating to what is perceived if there is to be consciousness or sense-experience. The fact that the brain is dark and mute is evidence for the perception being a relational state. How the brain gives rise to consciousness is a fundamental problem for philosophy and science.

You cannot "see" a message in your brain. The various transmitting mechanisms in the brain which process topographic maps of what is on the retina produce a brain state with a different description from the way we see, describe and know the environment, which is what we really see. Information from the visual environment — outside the head — will produce an image on the retina which is highly processed and abstracted from what is in the visual field. It bears no imagistic likeness and could not do so. Firstly, because the visual cortex uses information which does not come purely from retinal stimulation but from other sources, such as the tactile sense. For instance, if you see a mirage — the sun shining on the road which looks like water — you know you see a mirage because you are aware of the heat, an awareness which comes from a different neural source. Secondly, colour is the result of wavelengths of light on the retina and as coded in the brain it is not colour at all.

The "I" who perceives is the one who is conscious by virtue of undergoing a totality of information processing in the brain. For consciousness of oneself as "I", more than a brain is needed. We need to distinguish ourselves from things in the environment. You could not be a brain "looking" at some "diminutive screen" in your head and get any consciousness of yourself.

Consciousness cannot be flesh. This is the mind-body problem. The brain is essentially physical and consciousness is essentially mental and there can be no identity between these different kinds. It doesn't make sense to say consciousness is physical.

Three-dimensionality is something which we come to know through the sense of touch: I think this is Berkeley's view and it seems correct. The visual correlate, perspective, is not actually three-dimensional, although I have looked in Paul Churchland's book Matter and Consciousness and he does talk of "three-dimensional seeing" which occurs by means of "stereopsis" which is a comparison of images from each eye. You might do well to read this book.

Not many people would agree with you about reality. We are in touch with the reality of objects as they appear to us. The table may really be a bundle of moving molecules but given our perceptual equipment or the nature of our brain and its complex processing system, what we see is solid. This is our perceptual reality. A sense-experience is a feeling and when we touch something we feel a sensation in our finger. You cannot feel with your brain, your brain enables you to feel.

Dreams differ from ordinary intentional perceptual states which are relational. One of the relata — the external environment is missing in dreams so although contentful, they are not intentional. When the brain is working on its own, as in dreams, feelings and sensations appear in the same way as when awake. I'm afraid I don't know what it is that physically stimulates the brain to work on its own, but dreamt pains and fears have been shown by Freud to have some connection with real anxieties and physical conditions. For instance, you may be asleep while your body is de-hydrated and you are likely to dream of drinking water. Real life anxieties can come out in dreams but the true nature of the anxiety will be hidden. In both cases, the content but not the feeling is out of touch with reality. You are not drinking water or really anxious that, for instance, the bridge is going to collapse. Dreaming is like being in an intentional state but there is no state of affairs outside the head responsible for content.

Although it is easily suggested that each person's sensation of red is different, the way the brain codes information may have the same patterning in all persons, and neuroscience may be able to show that this is so.

If we assume that intentional states such as perception are relational then we are assuming that there is a reality outside the head — we get to this view by ordinary conceptual distinctions between states such as perception being environmentally related whereas dreams and imaginings are not so. However, we can imagine a brain in a vat being stimulated by scientists to have a sensation of red or to have a thought and it may be that is how for you or for me. The reason this is frightening to think about is that it involves a turning away from reality and you seem to be in the grip of this — yet unafraid! There are reasons for philosophers to consider possibilities such as the brain in a vat, but to really believe in possibilities rather than actuality isn't what most people want to do.

Rachel Browne


Okay... first, there are HUGE sources for information on this issue. I recommend you start here:

http://www.u.arizona.edu/~chalmers/
http://assc.caltech.edu/
http://www.phil.vt.edu/ASSC/resources.html
http://home.earthlink.net/~dravita/
http://www.phenomenologycenter.org/links.htm

That should get you started.

Now. Since there are enormous numbers of books, journals, etc., on these questions, all I'm going to do is skim them to give you some teasers instead of answers. For anything like real answers (as real as they get, anyway) you'll have to go to the sites I've given you.

"1. For example, you actually see this message in your brain."

Really? Do you? "Light" is electromagnetic radiation, and you have a subjective experience due to just one octave of the total spectrum of that radiation.

"Then who is the one that sees and perceives the image of this message in the brain?"

You see the image. One problem with your question is the term "message". It is a misconception that neural impulses are "messages"; then one starts asking, as you are, "messages to whom?" To whom, indeed. Try to conceive of an entity composed of, generated by, neural impulses, which reacts to itself, and you might have a better picture of the materialist conception of the mind.

"How do you define the consciousness that can see this image in the brain without the need of an eye?"

What do you mean by "define the consciousness"? And what good would it do, to "define" it? If all you want is a definition, try "self-awareness"; that's a perfectly good dictionary-type definition. But of course, you are not, and should not, be satisfied with that. Why not? Because what you want is not a definition but an explanation, and further, a reductive explanation. Good luck; no one has done that, but you'll find in the references above many attempts. On the other hand, if you want phenomenological analyses, check out the phenomenology links above.

Next... "see" "without the need of an eye"? You are assuming a great deal here; for a start, that there is an actual image to be seen, on a little movie screen, by a little man (a "homunculus") in our brain... but how does that little man see? Another little man in his brain? And that one? Whoops... See my discussions below.

"2. Brain is a piece of flesh composed of lipids, proteins and other various molecules. Could the consciousness that sees this image be this piece of flesh?"

"Be"? In what sense are you using that incredibly ambiguous word? Try this: consciousness is generated by and instantiated in the dynamics of the brain.

"Or could the brain cells make up a consciousness that sees these electrical signals as a sea view or an e-mail message?"

I simply don't understand this question. "Sea view"? Neurons do not receive email.

"3. No light penetrates the skull, which means the brain is entirely in darkness. Then how does such an illuminated, clear image is formed in this pitch-dark place? For instance how are the rays of the sun seen over the unlit brain cortex?"

They aren't. There is no image. You do not see the "rays of the sun". You are confusing your subjective experience of seeing the result of electromagnetic radiation, impinging on the retina, transduced into neural impulses which then become an aspect of an enormous system reacting to itself, with that objective radiation per se. Read Kant on this: we construct reality; we generate the experience of seeing. Who knows what there "really" is outside the box — the skull (more accurately, the body) — we live in. We assume that it is similar to our experiences, and, given that there is a world, we would certainly be dead if it weren't; so it is. But you must remember that what we experience, subjectively, is all a construction.

"4. Also no sound enters the brain. This means there is deep muteness where the brain exists. However, people listen to all different sounds inside the brain. The sound waves are turned into electrical signals inside of the ear and then transmitted to the center of hearing. And the consciousness inside the brain listens to these for instance as a melody. Then who is it that listens to the loud music aired from powerful loudspeakers and how?"

Same as 3 above. People do NOT listen to (objective) sounds; they construct the subjective experience of sound. As for who listens, see 1 and 3 above.

"5. The image is formed inside a miniature spot in the brain."

NO. See 3 and 4 above.

"Then how is the three-dimensional image with depth is formed on this diminutive screen? For instance when I look at the horizon or the sky, how is such an image with vast depth formed at this tiny spot of center of vision just as identical to its original with the same depth and sense of distance?"

You keep making the same mistake over and over. I'll just say it one more time: There is no image in your brain; your subjective experience bears no relationship that we know of — except the most general structural relationships (see Kant, Palmer, etc.) — to the reality "out there". There is no objective "light", no "sound", no "screen", no "depth". Zero, none, nothing. Those are subjective experiences. Neural impulses are reacting to each other, generating, we do not know how, those experiences. Period.

"What is it that gives me the feeling of distance and space?"

Now this is a better question, but one no one knows how to answer it. We do indeed have "feelings" or experiences of space, light, sound, etc. How are they generated, by neurons inter-reacting? We do not know. There are many studies showing what areas of the brain process what stimuli from various sensory inputs (to put it crudely), and you could say that those areas are "what" gives those feelings, but that's not really a good answer, is it. But it's about the best we can do right now.

"6. When a person sees a glass of water, in fact he does not see its real form but only a copy of it in his brain."

Not a copy, a construct.

"The coldness that he feels when touching the glass surface is not the real coldness of the glass but only a copy of it. This means nobody is ever able to feel that he touches the real glass."

That's better; now you're getting it.

"Since it is not his fingers that feel the sense of touch, but merely the sense of touch in the brain. Should we not in this case conclude that people are never able to reach the reality of objects and can never touch the reality of a glass?"

Good question, and some have. But if we assume that there's anything out there, we have to assume some degree of correspondence with what we construct; otherwise, as I say above, we'd be dead, because we'd be falling off cliffs, eating poison, etc, etc.

"But not every person knows this fact. Everyone thinks they touch and see real objects. Is it not strange that people are not aware of this and they never think about this?"

When you do, you're called a "philosopher" under the best circumstances, "insane" under the worst, and many in between. Also, to do this thinking to any reasonable extent, one needs leisure, food, shelter, as a minimum. If you're desperately trying to get those, you just don't have time, right? Also, it's scary... see 13 below.

"7. Nothing changes when a person is hit by a bus or comes across a lion."

Well, it sure hurts when it happens to me.

"Since, just like the image of the bus, the sense of collision or the fear while running away from a lion, all form in the brain. When I see a bus, I see it at the center of vision inside my brain. If I go and hold the door of the bus, I feel the coldness of the metal inside my brain. Then I cannot discriminate from this fact what happens when I feel pain if a bus hits or a lion bites. Then is it not very illogical when people say, 'It shows I am in contact with the bus or the lion because I feel pain when the bus hits or the lion bites'?"

This is the same question as 6, basically. However, it's not illogical, since it's based on the assumption (usually not stated) that there's a world out there. Given that assumption, the rest follows. Now if you want to question that assumption, fine... see the refs above for literature on this.

"8. We live the entire period of our lives inside our brains. In a similar way, we also have dreams in our brain. For instance when we touch a piece of ice in our dream, we feel it is wet and cold. Or, when we smell a rose, we get the wonderful scent of it. We again sense the feelings of fear, pain, anxiety and panic in a similar fashion. Then are the dreams and the real life the same in this sense?"

When I slept I dreamed I was a butterfly; now I think I'm awake... am I a butterfly dreaming I'm a prince, or a prince who just dreamed he was a butterfly? This is an old Chinese story, true, as far as I know. As you can see, you're not alone.

"9. The person's own body is also included in the images a person sees. So, a person only sees the copy of his own body."

Good inference.

"This means every person all through his life lives in the cave in his skull where he never knows what is outside, including his own body and other objects. Now think over this happening once more: Right now are you inside of the room you are present or is the room inside of you? Isn't the second alternative the right answer scientifically?"

"Scientifically" the first is correct, since science assumes that there is a world to be investigated. As to what is actually correct, see my discussions above.

"10. Let us imagine 5 different people who look at a garden of roses. Since every one of these people see the rose garden in his own brain, then aren't there 5 different rose gardens in the brains of every one of the 5 people? Is the color red that each one sees the same with the other's perception of red? Would there be any possibility to compare these?"

Now this is a much better set of questions, and one that is the subject of hot debate right now; see the refs above. First, there are indeed 5 different subjective experiences of the rose garden. Second, no one knows; but there are ways to compare certain aspects of the experiences of red: their relationship to the experience of blue, for example: how the colors fade into one another, how much light is necessary to see red. Stephen Palmer is more or less the expert on this at this point. But this is a very difficult and technical issue, and if you don't have background in cognitive science and perception, wait until you do before you tackle this literature.

"11. We say that the original objects we see the copies of in our brain exist outside, but what if nothing exists outside? Because we never have the ability to test this or observe this. Then is it not dubious that the original objects are outside? At least there is a 50-50 percent possibility. Then how can we be sure that the original objects are outside? If there is no original object outside, then what is the entity that makes the images and the senses in our brain?"

Wait, wait. "50-50 percent possibility"? No. How can you give any answer, any numbers, any definite statement of possibility on a question that you have absolutely NO data on? Answer: you cannot. See my discussion of 3, 4, 6, etc., above. This is basically the same question. Again, Kant tackled this issue head-on; many others have also. See the references above.

"12. If we are living an illusion that has the possibility of not having any reality outside, then we may be existing in a very different place. For instance is it not possible for the entire humanity to exist right over a piece of crystal? Or is it not possible that the complete history of humanity has been experienced in a place not bigger than the head of a pin? Would there be anything to stop us thinking in such a broad extent?"

Nope; go see the movie "The Matrix". Again, you're asking the same question as before. A more interesting question might be, "could humanity move into a virtual world inside a computer and be satisfied?" An interesting question indeed. I suggest that we aren't contacting alien life because they're all living in their computers, translated into conscious programs, having a much happier time in virtual universes than in the "real" one (And of course this one might be someone else's virtual universe... who knows? But for sure, it's not ours.).

"13. Some people are incredibly afraid when these topics are discussed? What do you think the reason for this may be?"

You've got to be kidding. Questioning one's entire basis for reality? For living? For believing what we see, feel, touch? The question is, why are those of us who do it not scared? Simple insanity, or what? That's part of the reason why philosophy is hard, and why people have to be introduced to it gently.

Well, have fun; you've got a lot of reading ahead...

Steven Ravett Brown


There is really one question here: how does the physical materialist account for reality? There short answer is: he does not. This kind of materialist thought scarcely begins to approximate reality, let alone question it. It is not "strange that people are not aware of this" [that reality is only "in the brain"] because for most people such questions fall short of the common sense which the grammar of ordinary language expresses and codifies. For instance, "brains" do not think and perceive in any language except the specialist technical language which belongs to areas of scientific research, which presuppose the dogma of materialism to begin with and to that extent is already sub-philosophical. It is persons who think, not "brains", not objects. And persons are not mechanisms either. Classical literature testifies to the fact that personal being, personality, is not purely mechanical or mechanistic, that thinking is not about "brains" and that "brains" do not have ideas; psychology, history, philosophy and common sense all attest to the same thing. The notions of human dignity, natural law and rights are all grounded in a sense of irreducible human spirit. The very word "spirit" evokes a whole vocabulary of words with acknowledged meanings and real sense, beyond the abstract understanding of materialist conception.

To interpret your questions Berk, as I expect other respondents will try to do, is (I would argue) to give tacit credence to the fallacies of the hard materialist position which they announce.

Matthew Del Nevo
http://www.sicetnon.com

back

Paulo asked:

Please Professor, explain me a way to teach Tarski's theory of truth for students who don't have a logical foundation.

Tarski's theory of truth without the logic? That's a tough proposition! In his famous paper, 'On the Concept of Truth in Formalized Languages', Tarski describes a potentially infinite hierarchy of 'meta-languages' each one of which is able to talk about the truth or falsity of sentences in the level just below it. The aim is to solve paradoxes like the following: You take a piece of paper and write, 'The statement on the other side of this paper is true'. Then on the other side, you write, 'The statement on the other side of this paper is false'. (See Answers 2, answer to Susan.)

I have never found this self-referential paradox particularly gripping, so I am not persuaded that Tarski's clunky contraption is really needed. However, the solution to the paradoxes about truth is not the philosophically most important part of Tarski's paper.

Tarski gives a deceptively simple test which you can apply to any proposed theory of truth to see if that theory is adequate.

First, we need to see that there are lots of things you can say about a sentence in a language. You can say that it is grammatical, or that it is poetic, or that it contains exactly eleven words, or that it is found in the King James Bible. But there is just one thing you can say about a sentence which has the remarkable effect of producing a result which is equivalent to the original sentence. And that is when you say that the sentence is true.

Let's forget about truth for the moment. We don't know what that is. Instead, Tarski says, let's just use the the letter 'T'. Suppose that someone produced a definition of a predicate, 'T', which applies to sentences of a language. And suppose that as a consequence of this definition, the following rule (Convention T) holds:

'Snow is white' is T if and only if snow is white
'Grass is green' is T if and only if grass is green
'Giraffes live on Mars' is T if and only if Giraffes live on Mars
'Freddie Starr eats hamsters' is T if and only if Freddie Starr eats hamsters...

...and so on for every sentence in the language.

Then we would be justified in saying that 'is T' is the same as 'is true'. In fact, we can say more than this. Convention T is all one could require of a theory of truth. It tells us what truth is, in the sense of distinguishing the predicate 'is true' from every other predicate that can be applied to sentences, like 'is grammatical', 'is poetic', 'contains exactly eleven words', 'is found in the King James Bible' and so on.

Just to see this working in practice, consider what is admittedly a pretty poor candidate for a theory of truth, the Biblical theory:

Biblical theory of truth: sentence x is true if and only if x occurs in the King James Bible

Does the Biblical theory satisfy Convention T? No. Consider, for example:

'Tony Blair is Prime Minister' is found in the King James Bible if, and only if Tony Blair is Prime Minister

But that is false. Nowhere in the King James Bible does there occur the sentence, 'Tony Blair is Prime Minister'. So the statement on the left hand side of 'if, and only if' is false. But Tony Blair is Prime Minister. So the statement on the right hand side of 'if and only if' is true. And the result of that is a false statement. It follows that the Biblical theory of truth does not satisfy Convention T.

Armed with Convention T, we can dispose in a similar way with the pragmatist, coherence and 'verification under ideal conditions' theories of truth. You might try it as an exercise.

Tarski, however, makes a stronger claim. He claims that Convention T is, in effect, a correspondence theory of truth. Why is that? Because in each of the examples given above, 'Snow is white', 'grass is green' etc. the left hand side talks about words and the right hand side talks about the world. If I say that 'Snow is white' is true I am talking about words. If I say that snow is white then I am talking about the world. Here we can literally see the 'correspondence' required between words and the world in order for there to be such a thing as truth.

I actually think that this is a logical mirage. No serious correspondence theorist would be satisfied with such a thin account of 'correspondence'. The real import of Tarski's Convention T is to undermine the possibility of ever giving an informative theory of truth. The proper conclusion to draw is the one that Tarski's predecessor Frege drew, and before Frege, Kant: truth is indefinable. — However, that is another story.

Geoffrey Klempner

back

Leslee asked:

What is the difference between order, balance, and harmony? I am a scientist considering a career change. Science seems to be mainly about order; the arts, which I am increasingly drawn to, seem to be more about balance and harmony. I sense the three are not mutually exclusive, but I think if I can figure out the difference between these concepts, I will be better able to design a career that meets my needs.

An interesting question... one is tempted to refer you to the Greeks, but then you'd just learn what they thought. My take on this issue, for what it's worth, is that when you refer to "order" you mean something like the marshalling and organizing of facts, data, into regularities, then possibly into laws. This is a classical picture of science, and it's probably mostly correct.

Balance and harmony... here you could be referring either to internal, i.e., subjective, feelings; or to some sort of objective standards, e.g., for the right kinds of structure in a work of art. Let's just explore the subjective, because that seems to be the general tenor of your question.

Feelings of balance and harmony come either immediately upon the "viewing" (I'm using the term more generally than just vision) of a work of art (and Kant has a lot to say about this also in the Critique of Judgement), or as a kind of general feel or orientation that one has when one does work, for example, that one loves. Now if you're referring to the second of these, this discussion is too specific for you, and my comment is merely that if doing art satisfies you and science does not, in terms of this general feeling of well-being, go do art (but I hope you have money saved up).

Let's take the first, however: the specific feeling that one has upon viewing a particular piece. How is this different from feelings that one has upon viewing a well-written, profound scientific paper? There may be no difference at all; or the difference might be due to your own preferences... we come here to either the issue in the above paragraph; or to the difference between art and science. That boundary blurs in either of their higher reaches, in my opinion, since both (again in my opinion) serve to reveal some aspect of reality to ourselves and others. But art usually emphasizes feelings, while science emphasizes objective knowledge. So what is revealed in the case of art, or better, the emphasis on what is revealed is some truth about one's own, or humanity's in general, feelings. While in the latter case what is revealed is some truth about facts... which can be very aesthetically moving, but does not deal directly with feelings, i.e., feelings are not the operators in science, although they can be the subject of scientific investigation, of course.

So it's beginning to seem as if what you might be moving toward is a desire to investigate and to manipulate feelings on a much more specific level than the general feelings of aesthetic satisfaction that you get from viewing a good paper. I've just got to throw in a plug here for the Jungian (simplistic as it is) four-part analysis of the personality; and his dictum that as we mature we move toward other areas of the psyche. This sounds to me like what's happening with you, and that's great, in my opinion.

Now... I haven't said exactly what "balance" and "harmony" are. One can talk about this and that being opposites and needing to be "evened out", so to speak; I'm afraid that I'm rather atypical in this area, since I have a rather profound skepticism about the validity of analyses purporting to dissect the human psyche (although Aristotle did this and so did Jung... and many others). But let's take the general point of feeling versus analysis, or the intellect versus the emotions... whatever categories you want to use here. There does indeed seem to be a separation here, and as you probably know, that separation is backed up by neuroanatomical data. Again, what you seem to be wanting is more emphasis on the emotions, and perhaps what you mean by "balance" refers to a very general evenness, let us say, between emotional manipulation and awareness, and intellectual operations.

The advantage of doing art is this direct manipulation of feelings. The disadvantage is that you can't really say anything specific... what does a sonata mean, or a piece of sculpture, etc.? On the other hand, the pros and cons of science are just the opposite, aren't they... you know what you mean, but not how you feel about it in any specific sense. You can say profound things about the world, but is what you're saying beautiful, not to mention how you're saying it? Not usually. Yet on the other hand, what do you get with beauty... besides very general, vague comments on the (subjective or objective) world? Yet, on the other hand, although, through science, you find out all sorts of specific things about the world... there's always the realization that there's got to be more than just the facts, right? Hesse addressed this issue rather directly in The Glass Bead Game. So far as I know, most of the literature rants and rails about the above separation, without (with a few exceptions, like Hesse) trying to synthesize art and science in any direct, specific way. Oh, there's plenty of art about science, or art that uses science in some way or another, but that's not the point, is it.

So, you can do both art and science, but remember that it takes about a decade of hard work, usually, to master something. You can continue with science, and gradually do more exploring of your feelings through viewing, reading about, etc., art, psychology, and so forth. That's the usual method. You can go the art route, take the 5-10 years to master an art form, be poor (unless you've got savings), and finally, if you're lucky and talented, find that you are exploring and manipulating feelings directly. That's the unusual method, and very few manage that... but some do. Or you can do something like become an art dealer or collector, after studying for the 5-10 years to achieve mastery. But then you're not an artist, although you're on the fringes, and being on the fringes is always a bit frustrating unless you're someone who is totally reconciled and satisfied with not being an artist, while still loving art.

Steven Ravett Brown

back

Sue asked:

What exactly was Berkeley's Doctrine of Abstract Idea? What was its implication in his overall idealism?

There was a similar question from Glouisel which you can find in Answers 11 (Answer to Glouisel).

Berkeley did not believe in "abstract ideas" and argued against them, especially in relation to Locke, in great detail at the beginning of Principles of Human Knowledge. For Berkeley an idea is a sensory item or a copy of a sensory experience in memory or imagination. Things or objects are complexes of ideas since to have an idea of a table requires that you both see it and feel it, but this is not an abstraction from sensation. Berkeley thought that Locke's idea of a triangle which is abstract and not an idea of any sort of particular triangle was absurd. Rather, for Berkeley, an abstraction of something like a triangle is a theoretical construction which belongs properly to language rather than perception. We can speak of a triangle in general but cannot have an idea of a triangle in general and language misleads us into thinking that we can imagine or conceive of some abstraction of a triangle. Language does not determine what there is but what we perceive does.

When we perceive a tree, this is a complex of ideas. We see a tree but this visual idea doesn't give us an idea of its solidity. For that we need an idea from the tactile sense, but the resistance we find on touching tree doesn't provide us with an idea of matter as substratum. Berkeley rejects Locke's distinction between primary and secondary qualities and argues that so-called primary qualities are sensible properties. A sensible property is essentially perceived, so things exist because they are perceived by God.

Rachel Browne

back

Tom asked:

I am interested in biographical and autobiographical information about philosophers. I am aware of many of the popular books of this genre, but I was wondering if there were some very detailed works that exist which have offered insights into the everyday lives of the great thinkers. I am especially interested in contemporary philosophers, although I am aware that many living philosophers are concerned with their privacy, which I can respect. I also enjoy seeing data and analysis, (the frequency of depression, musical tendencies, birth order, married or single, eccentricities, etc.). Does anything really striking come to mind?

Yes, one particular book does come to mind: Ben-Ami Scharfstein The Philosophers: Their Lives and the Nature of Their Thought (Blackwell 1980). It is an amazing book. You'll never look at the activity of philosophy in quite the same way again. The book is rich in revealing anecdotes and biographical detail. There are plenty of references to Freud. Scharfstein is not afraid to make some bold generalizations, for example, comparing different philosophical theories with various forms of madness. There is even a comparative table showing when each philosopher's father died, whether or not the philosopher married and so on. I found the section on Kierkegaard particularly poignant.

In summary, not a book that would win Professor Scharfstein many friends within the stuffy world of academic philosophy, but a great read.

Geoffrey Klempner

back

TK asked:

Mathematicians and physicists generally do their most creative work early in their careers. Do philosophers suffer the same fate? Why? What does this mean then to a senior citizen recently interested in philosophy who has taken his shoe off to dip his foot in the Philosophy Lake only to quickly withdraw it finding that he had left his sock on?

The most notable case of a philosopher who only got into his creative stride in his 50's is Immanuel Kant, who published the first edition of the Critique of Pure Reason at the age of 57. Kant is by no means an exception to examples of the productivity of philosophers in their middle to advancing years. In view of the contrast with mathematicians and physicists, this is a fact which requires explanation.

There is no doubt that the loss of mental speed that arises with the passage of the years is a handicap in any intellectual discipline. The evidence seems to suggest that the loss of speed has the greatest negative effect on the ability to do creative work in mathematics, and disciplines which rely heavily on mathematics such as physics.

Philosophers are trained to follow the logic of arguments. That does require a certain mental agility. They are also called upon to exercise their powers of judgement. Now, to take a complex set of considerations for and against and make sense of it, to see the wood rather than being lost amongst the trees, is a quality prized in Judges. Age is not considered to be a handicap on the bench. One the contrary, we would be worried if Judges were appointed in their early 20's. Why should it be so surprising, therefore, if philosophers continue their creative output well into their advancing years?

Learners of retirement age have many advantages over students fresh out of high school. They have a lifetime of experience to call upon; a greater ability to question what they are taught and refuse to have the wool pulled over their eyes; the self-confidence that comes from having successfully navigated their course through life, despite the disappointments and setbacks that each one of us must face at some time or other. You have lots of things going for you. My advice is, don't worry about getting your socks wet. Wade in with your boots on!

Geoffrey Klempner

back

Fred asked:

Science has become an integral part of many issues of public concern — medical, informational, environmental, and legal. Scientific experts frequently square off in the evening news and during legal trials. In short, because of the limits of lay knowledge, experts have quite a bit of power these days. How can philosophy help me decide on the appropriate limits of expertise? What does philosophy have to say about experts?

I was recently talking to someone who has just finished her dissertation on experts in the medical field and she told me that she no longer believes in experts because there is a subjective starting point which determines what they will eventually take to be true. Of course there is a subjective starting point, but this doesn't mean that one can person cannot know more in a particular field than others. However, more interestingly, she has discovered that many people in the medical field who are taken to be experts are not in fact so. They have passed through the proper channels and have been recognised by professionals within their own field as experts but their position as experts is not actually based on experience or knowledge but on their ability to come to the fore in their professions because of their personal qualities. So if we want to know who we should regard as experts it would be wrong to suppose that the profession itself will identify the right person. We have to be in a position to assess their work.

Your question prompted me to read Paul Feyerabend's article "Experts in a Free Society", initially printed in The Critic (Nov-Dec 1970) and reprinted in Philosophy, The Basic Issues edited by Klemke, Kline and Hollinger. Feyerabend describes experts as people who have decided to achieve supreme excellence at the expense of balanced development — he is unbelievably scathing! — you must read the paper, but I can't resist quoting from it here:

It is quite depressing to see with what fervour thousands of young people throw themselves into special subjects where they are trained and trained and trained by receiving now punishment, now a pat on the head until they are hardly distinguishable from the computers whom they want to approach in efficiency .... these inarticulate and slavish minds have convinced almost everyone that they have knowledge and insight ... that they should be able to educate our children ... Should we allow a bunch of narrow-minded and conceited slaves to tell free men how to run their society?" An example of narrow-mindedness is the physician who deals only with the physical body.

Feyerabend also criticises the experts' desire to appear professional through the use of lingo which is both absurd and can distort facts. Masters and Johnson, for instance, want to say that a male should ask a female what she wants rather than guessing, but actually say "The male will be infinitely more effective if he encourages vocalisation". The first part of the sentence is not really true and the latter part is an absurd way of talking about something as simple as "asking". Apparently, the "awful Newton" is responsible for this way of writing.

So this is how one philosopher views experts. Feyerabend also thinks that expertise is limited by following methods such as simply looking at empirical evidence and says that Galileo, who is taken as an example of a well-rounded rather than narrow-minded scientist, was more in favour of following a hunch and acting on prejudice. There is nothing intrinsically wrong with this because to agree on a method doesn't mean it is correct, but may reflect the conformism of scientists. Feyerabend also points out that Galileo worked prior to professionalism with its lingo and included personal history and rhetoric in his reports, an approach which was individualist rather than conformist and methodological as we expect the expert to be today.

Rachel Browne

back

Allen asked:

Is the Christian Doctrine of Predestination still part of Christian belief today? How do you reconcile this doctrine with the exercise of Free Will? Are there different doctrines of Predestination? If so, what are the differences?

It is a bit sweeping to say, "the Christian doctrine" of predestination. It is a Latinate doctrine if anything (rather than Greek), and a Protestant (North European rather than Mediterranean) one at that. You need a historical sense to grasp the meaning of predestination and the differences in understanding of it, since these are not basically analytic, but soteriological (belonging to the theory of salvation).

The Catholic Catechism (1995) doesn't list predestination in its detailed topical index, although masturbation is listed and there is a whole page about it. Times have changed obviously. The Magisterium is more concerned with masturbation than predestination.

Predestination is in the Old Testament, in the so-called "inter-testamental literature" especially and in particular found in the Wisdom of Solomon and the Books of Esdra and various other passages under the heading of Divine Providence. This latter is normally understood as co-operative (synergic) with human free-will, whereas, predestination is usually associated with the notion of divine foreknowledge. Are these two in contradiction you may well ask?

There are two ways that Christian philosophy thinks about the patterns of history. They are not mutually exclusive but complimentary, although at various points of human discourse one way has been over-emphasised at the expense of the other. The two ways are 'from above' (the God's eye view of history) and 'from below' (the mortal view). The former view is qua the whole, or the unity of all — the One — and the other is qua existence here. The predestinarian view was over-emphasised by Augustine in the 5th century in his dispute with Pelagius and even more famously in The City of God, his masterpiece. The Council of Orange in 529 anathematised predestination to evil, which Augustinians were preaching. In the Eastern churches Origen had taught (since the third century) that eventually all would be saved. Although not strictly orthodox, the idea of the apocatastasis as it is called, is a Christian hope, but contrary to Augustinian soteriology.

It was Calvin (and Calvinism in his wake) in The Institutes of the Christian Religion (1559) who explicitly taught predestination. "No one who wishes to be thought religious dares outright to deny predestination, by which God chooses some for the hope of life, and condemns others to eternal death." (III.xxi). The Baptist Confession of Faith (1646) speaks of the "just condemnation" of the damned (article III) and of "salvation for the elect" (article XXIII). The Catholic Council of Trent, (1545-63), while not mincing words about Original Sin, affirmed that "Man can be justified before God, by his own works" (Session VI Jan. 1547). Historically, predestination has never really been a Catholic or Orthodox teaching. It is Protestant.

Anglicans wobbled. Predestination, according to William Beveridge's authoritative interpretation the Thirty Nine Articles (in 1704), said God's predestination (a nod to Protestantism) concerned "the mystery of mysteries" (a nod to Catholicism) "which must needs be infinitely above man's apprehension" (On Article XVII 'Of Predestination and Election'). The differences in the doctrine of predestination are denominational.

Matthew Del Nevo
http://www.sicetnon.com

back

Ryan asked:

Can Realism be defended by claiming that it offers a better explanation than anti-realism about why we can affect our surroundings? It's very easy in physical terms to explain why I can lift or push an object, but if things only existed in perception, it seems that there would be no real account for this.

The Berkeleian idealist or 'immaterialist' denies the existence of objects external to the mind. However, your question also concerns a form of opposition to realism which arguably goes back to the Ancient Greek sophist Protagoras:

Of all things a measure is man — of the things that are, that they are; of the things that are not, that they are not.
Jonathan Barnes The Presocratic Philosophers §491, p. 541

On the Protagorean, or 'anti-realist' view of truth, there being things that are the case depends on the possibility of human knowledge. There is no place in the Protagorean universe for 'facts' or 'truths' which human beings are incapable of knowing to be the case. In recent times, Michael Dummett, has revived this form of anti-realism, an issue which I myself have taken a keen interest in. (See Answer to Bill, Answer to Joanna, Answer to Rute and Alan, Answer to Glouisel.) In what follows, I shall be talking about my interpretation of anti-realism, rather than Dummett's.

Idealism You say that 'it is very easy in physical terms to explain why I can lift or push an object'. What you are talking about is the physical causation involved in the movement of your hand followed by the movement of the object. Berkeley has two replies to this, however:

(a) If we are concerned merely with the lawlike behaviour of objects which we observe in the world around us, then Berkeley would say that it is just as easy to conceive of those laws applying to the world of objects inside God's mind as it is to conceive of them applying to the realist's world of 'material' objects. In this respect, realism and idealism are on a par. The laws of nature are equally 'real' whether we conceive of them applying to the realist's world of matter in space, or to the idealist's 'virtual reality' world of mental 'ideas'.

(b) Berkeley anticipated Hume in questioning whether from the mere appearance of the movement of one object, followed by the movement of another object we are able to derive any meaningful notion of causation. Berkeley believed that that it is only through acts of will that we actually perceive causation in action. So he would deny your claim that it is easy to explain how physical change takes place. It is much easier, he would argue, to grasp the real workings of causation in the world of 'ideas' than it is to grasp it in the realist's world of dumb 'matter'.

Anti-realism A version of your objection can be formulated in the context of the anti-realist view of truth. Think of all the things that might be the case, consistently with all the things we could ever know to be the case. According to the anti-realist, each of these alternatives describes a possible world, none of which carries any special metaphysical 'marker' to designate it as the actual world.

Let us suppose a historian comes across a manuscript which they have good reason to believe from the context, date and stylistic considerations was either the work of author A, or the work of author B. Suppose also that all evidence of whether it was A or B who wrote the work has been destroyed. According to the anti-realist, the substance our world, the world which we call 'actual', is made up of possible worlds, including the possible world in which A was the author and the possible world where B was the author. Both possible worlds are equally 'real'. But how can a determinate object, such as a manuscript which I actually hold in my hands, have an indeterminate cause?

This objection effectively destroys the idea that our actual world is determinate so far as we are able to uncover facts about it, and only indeterminate round the edges, beyond the limits of possible human inquiry. However, this does not have to be the anti-realist view. The alternative is to deny that there is such a thing as the manuscript, or the historian who holds it in their hands. 'The' manuscript is in reality the work of A, and at the same time is in reality the work of B (and of C, and D and all the other possible individuals who might have been its author). In other words, according to the anti-realist, the truth about our world is indeterminate all the way to the core.

Geoffrey Klempner

back

Marek asked:

I'd like to ask one significant question that is deeply dwelling my mind for some time:

What lessons do you think can be learned from the development of computers over the last twenty years?

A lot has been learnt about the workings of the human mind within the field of artificial intelligence, and the most recent development is the creation of learning robotic intelligences, such as the Abo dog. Intelligence is not simply created by programming but also by environmental interaction. Whilst we knew that this is the way human intelligence develops, the importance of man as organic is giving way to a view of man as robotic.

Also the more recent development of the internet has shown us something about human nature. People desire the ease of being pandered to, of hiding behind a screen and playing unreal games to an extent that even the existentialists didn't dream of. It is not simply that there are people who are engaging in internet relationships which need not be genuine because there is no face to face contact, and that there are bizarre phenomena like cyber-parties in which participants pretend to be socializing, but there is a real movement towards a strangely onanistic form of communication. I gather that current technological advances have enabled computer scientists in America to develop computers which respond to the sort of person you are. For instance, a male computer-user will receive spoken instructions from a soft-voiced woman because he will find this most pleasurable, but an elderly lady will be spoken to by a middle-aged man which she will find comforting as this is the sort of person she take to know best about computer operation.

Hopefully, what we will learn from this is that this not what people really want. In the words of Karl Jaspers it is likely to lead to "an emptiness ... a loneliness that hides itself" and which seeks relief in the irrational until "it leads eventually to a deep comprehension of the importance of establishing communication between man and man". Genuine communication involves compromise, transference and an understanding of what others are really like.

If we look at human nature rather than the human mind, man doesn't seem to be robotic and so perhaps we should be a bit sceptical about what we can learn from computers.

Rachel Browne

back

Valentina asked:

I'm very much interested to learn about the "poetic of space". If you could give me some tips about this then I would be grateful.

the Poetics of Space by Gaston Bachelard is one of a series of books he wrote on poetics and nothing more authoritative and inspirational is found on the subject as far as I know. Bachelard was a famous professor of the History of Science at the Sorbonne in Paris and there is a bust of him on display in one of the corridors. I believe alongside the other great luminaries of French philosophy. It was only later in life that he turned to poetics out of a profound feeling for his language. All the tips you need and much more will be found in Bachelard's book.

Matthew Del Nevo
http://www.sicetnon.com

back

Tom asked:

I am a recent graduate of a bachelor's degree in Philosophy. I studied some of the basic arguments of the free will/ determinism controversy of the great thinkers and the postings on this site. My current "allegiance" is toward a strict deterministic viewpoint because of the impression that science has given me, though I am not a professional scientist. I do not understand how so many thinkers say they are determinists and then talk about who should be punished. But more odd to me is how after just saying that everyone is under rigid causality they are now saying to not punish under certain conditions.

I think the most coherent angle is to say, "Punish...or don't..it doesn't really matter...it is the law of the universe that these things had to happen, even your attempt to talk about it, the philosophers to try to change it despite iron causality controlling them etc. It is also necessary that we should have spent time talking nonsense about it. A very great person is just as necessary as a base criminal. He has only tricked us into thinking he/she is great because we believe he/she could have done otherwise. In the material world which is us as well, there are no special people."

This seems to me the real last temptation of the philosopher, not pity as Nietzsche thought.

Let's assume for the sake of this discussion that the thesis of determinism is true. If it isn't, then we have the comfort of knowing that the future is not a foregone conclusion. But it doesn't make much difference so far as the free will debate is concerned. There is as little justification for punishing an act which has no cause, as there is for punishing an act that is determined by the agent's prior state.

In response to this challenge, the theory of punishment goes through several dialectical spirals:

Level 1 If it is determined that the agent acts, then it is also determined that we respond with sanctions. The force of calling this response 'punishment' can be explained in terms of its intended effects, as can the development of rules for deciding when it is, or is not, appropriate to punish. In the vocabulary of determinism, we learn to discriminate cases where punishment is effective from cases where punishment is not effective. Punishment is not effective for automobiles which fail to start, for infants who lose control of their bladders, for adults who are not in possession of their faculties, or for adults who, though in full possession of their faculties, did what they did as the result of physical coercion.

That primitive theory of punishment is refuted by the example cited by F.H. Bradley, of the Master of Hounds who gives his dogs a good thrashing before they go out on a hunt "just to show them who's boss". According to the Level 1 theory, there is no reason why we should not 'punish' those who have not committed any wrongdoing, but might possibly be deterred from doing so by being given a proactive thrashing.

Level 2 As an institution, the practice of punishment gains enormously in effectiveness from being combined with the natural, though in deterministic terms irrational, human desire to see the wrongdoer 'paid back' for their wrongdoing. This provides the motive for distinguishing cases where punishment would be effective, but not deserved, from cases where punishment would be effective and deserved. Rather than undermine this powerful institution, we forbear punishment of the innocent, even in cases where it would otherwise be very effective as a deterrent.

The problem with this theory is that distinguishing between the 'guilty' who deserve to be punished from the 'innocent' who do not can only be effective so long as those on whom punishment is administered do not reflect on the implications of the rationale supplied for making this distinction. For, if they do, if they realize that it was not possible, given the initial conditions, that either the 'guilty' or the 'innocent' should have chosen to have acted otherwise, they will refuse to accept that there is any real difference between the two cases.

Level 3 Punishment has to be seen within a wider framework of our interpersonal attitudes. The seminal paper is P.F. Strawson's British Academy lecture 'Freedom and Resentment' (reprinted in several places, including his collection Freedom and Resentment and Other Essays). For example, although you profess determinism, you submitted your question in the hope of a reasoned response. In pressing the 'submit' button you did not merely intend to bring about a certain effect (as would be the case, e.g. if the answers for 'Ask a Philosopher' were not produced by humans at all but by a AI program running on a supercomputer). The human world brought into being by our inter-personal attitudes — a world where arguments are 'valid' or 'invalid', where actions are 'right' or 'wrong', where punishment is 'deserved' or 'undeserved' — cannot be dismissed as an illusion merely on the grounds that it cannot be perceived from the fully objective standpoint. On those grounds, the passage of time, the world of our sense experience, consciousness itself are mere 'illusions'. An excellent book to read on these issues is Thomas Nagel The View from Nowhere.

That is not the last word on the subject, however. Even though we may accept the impossibility of discarding this 'double vision' of the world and our place in it — the world seen from the subjective and and from objective viewpoints — there remain reasons for metaphysical disquietude. We praise or punish people for the choices they have made. It is part of what it is to see a person as a person and not a thing that we do not try to 'get behind' the person to see the chain of causes leading back to the moment of their birth that accounts for their being the way they are, or for choosing the way they chose. Yet at the same time, we feel a keen sense of cosmic injustice that one human being has been 'chosen' by the world to do good, while another person has been 'chosen' to do evil.

However, cosmic injustice is the one injustice that cannot be righted by human efforts. To think that we can right it by refusing to make judgements 'good' or 'bad' is the illusion which, I suspect, continues to grip you.

Geoffrey Klempner

back

Aisha asked:

My question is twofold:

Music is coming to the stage where composers are continuously trying to find unique orchestration and instrumentation, to the point where the music no longer expresses either program music (music created for a certain emotion or situation) or absolute music (music for the sake of music). Instead, we are left with composers simply trying to be more unique than the next. This does not include those who actually are able to create music that the majority can relate to or view as an important work of art. Do you believe that the limitations of our western music simply do not allow any longer for any more pieces of as high caliber as that which was written by Bach or Beethoven?

Now, let's say that a child is left in a room with a piano, and this child has an innate talent for music (absolute pitch, and a natural ability to create tunes). If this child has no previous knowledge of the world of western music — not even that of simple tunes such as Mary Had a Little Lamb — do you think that he would "recreate" this piece? What about other pieces? In other words, are the songs that are a part of western culture integral to the way we think, or would he begin to compose songs that were, in our eyes, obscure and atonal?

I cannot think of what limitations on western music would restrict the creation of works of high calibre today, since if these are purely musical limitations, they would have existed in Beethoven's time. Furthermore, there is no reason to suppose that all great works have been written. As you say, there is now a search for unique instrumentation. In the past composers will have written with only classical instruments in mind but now composers can create with a wider choice. So really, classical composers were more limited than musicians today and perhaps the limitations on the kinds of externally produceable sound forces greater creative effort. With only a pencil the visual artist needs to be highly skilled, but with paints and textures he need not be.

Music, like visual art, is shaped by the nature of the times. Artists can display something surprising in a gallery — famous cases being a Duchamps' urinal and Tracey Emin's bed — and it becomes "art". As a result of this, it is now held that there is no such thing as real art: Art is something defined by what the institution stipulates it to be. However, there are still brilliant artists around and, as you say, there are excellent musicians, and only some seek to be unique. It is difficult to know why this is so but it isn't a phenomenon confined to music and art, it is also found in literature and film. I think one reason is that it is part of our idea today that to be creative is to be innovative and to move beyond the traditional and classical forms — this true of all art forms. Visual art has been intellectualised and it is no longer important that the artist displays skill in his work as long as it "signifies" something — and perhaps this idea is being taken up by musicians, but hopefully not!

We can only know what the child would do by empirical experiment. However, we do know that the child would be restricted in the sort of tunes it created by the piano which has a determinate range of notes recognised by Western culture. If it were a stringed instrument, there would be a possibility of creating new notes not recognised by us. Given the limiting factor of the piano, I think it is possible that the child would create music which we would find natural rather than atonal. The child will experiment with small phrases and will settle on a sound which resonates with him which, at this initial stage, will reflect tones and rhythms of his spoken language, as the sound of language probably underlies basic tunes. The foundation for this assumption is that Western speech, tones, notes and music differs totally from, for instance, Chinese speech, tones, notes and music. Beyond simple tunes, musical creations do seek to imitate, express or represent experiences or emotions and stories. We could say that initially music represents mere sound based on the rhythms and tones of speech, and because of the different nature of speech in Western and Eastern culture, the origins and hence the development of music is different. However, what is expressed, represented or evoked would not necessarily be different since most experiences and emotions are universal rather than cultural.

I don't think music is out there to be found and it is unlikely the child would re-create a tune we know.

Rachel Browne

back

Junaid asked:

What philosophical questions arise from prayer and why do you think God has ordered us to pray? If He is All-knowing and All-Seeing why must we pray?

Prayer is not so much an order as an expectation. In the Judeo-Christian tradition represented by the Bible man is shown negotiating with God. The Hebrew prophets continually do this. Abraham saved his nephew Lot and his Lot's family from Sodom, which God wanted to destroy, by bargaining with God (Gen. 18:23ff. "Will you destroy the righteous with the wicked?" etc.) Moses, Isaiah and Jeremiah all thought God had chosen the wrong man for the task but God gives each of them the power of prophetic speech and the courage of His convictions. Jonah refuses to follow God's lead in the destruction of Ninevah, but when God changes his mind, having seen that Jonah is probably right in shying away from the task, Jonah is quite put out. ("Should I not spare Nineveh that great city?" God asks him in a rhetorical question at the end.) Job forces God to meet his demands (that he is innocent and his fate unjust) over and above the best the Devil can do. Jesus prays in Gethsemane for the inevitable not to happen, but his prayer is not granted.

What philosophical questions arise? Is that which is ultimate qua Being not abstract but active? Can one communicate with the Absolute? Can the Absolute communicate with us and does it? When? How? Has our world history been formed out of this communication? To what extent? Is this a belief or an experience? Given this exchange with the Absolute that has gone on for so long, what does it mean for the question, 'how are we to live?' Is the essential existential? etc etc.

That God is All-knowing and All-seeing should be understood in context. "God is All-knowing and All-seeing" is not a proposition in the analytical sense and is misunderstood if it is taken as such (as is philosophy of religion). The context is one of language (in an ontological not merely linguistic sense) and myth. God is All-knowing and All-seeing like a Father over his children. The meaning is an analogous, not a totalising interpretation.

Matthew Del Nevo
http://www.sicetnon.com


There are many different kinds of prayer that the faithful may practice: Prayer for Forgiveness, Petitionary prayer, prayer that glorifies God and others. Perhaps the most philosophically interesting is petitionary prayer. We can ask two questions; 1. What is petitionary prayer supposed to do? 2. How does petitionary prayer work?

When someone performs a petitionary prayer he is asking God for something he believes to be good and worthwhile asking for. A petition is to ask God to intervene in the world. But if this is the intention of pray we can see that it is pointless. Because God is omniscient and omnibenevolent. He is also supposed to be self sufficient, i.e. independent of any and all influence. If God did respond to the prayer this would mean that God could be effected by something which seems impossible so prayer is ineffective. If on the other hand God could be effected and did respond he would be responding to a call to bring something Good into the world, but if he is omnibenevolent wouldn't He have already created every possible good?

Perhaps not, a faithful person may say that there are some goods that can only be created when asked for, prayer then would be the vehicle that these goods are instigated. But this sounds suspicious to me. It sounds as if by praying we can tempt God to give us more goods than we would otherwise have. But what does it mean to say that we can tempt God? In his book The Star of Redemption Franz Rosenzweig actually talks about a twofold temptation. On the one hand God tempts human to pray with the promise of eternal life. On the other hand we tempt God to provide us with a good life with our promise that we will be obedient and faithful.

But now rather than a loving relation between God and man we have a barter system between 'spiritual capitalists'. (Note that I do not mean to imply that this is Rosenzweig's idea. I mention him as a possible reference if you are interested.) So it is not at all. clear what prayer is supposed to be, nor is it clear how prayer could work. First, since God cannot be effected by any cause outside himself, prayer can have no effect on God. Second, since people pray of different things, often conflicting things God could not possibly fulfil all prayers. For example imagine that some one is seriously ill. One person prays that they die quickly another prays that they hang on to life, both want something the other does not. How can God answer these types of prayer?

Of course these are the tough technical issues of prayer, perhaps none of these arise for those who are faithful, those who often simply want to talk to God, to share a personal relation with God. This may be all that prayer could amount to given the ingrained problems we have surveyed and while it would restore the loving relation between man and God it may leave prayer itself uninteresting for the philosopher.

Brian Tee
Dept of Philosophy
University of Sheffield